*NURSING > TEST BANK > TEST BANK LEHNE’S PHARMACOTHERAPEUTICS FOR ADVANCED PRACTICE NURSES AND PHYSICIAN ASSISTANTS 2ND E (All)

TEST BANK LEHNE’S PHARMACOTHERAPEUTICS FOR ADVANCED PRACTICE NURSES AND PHYSICIAN ASSISTANTS 2ND EDITION ROSENTHAL ,100% CORRECT

Document Content and Description Below

Chapter 1: Prescriptive Authority Test Bank Multiple Choice 1. An APRN works in a urology clinic under the supervision of a physician who does not restrict the types of medications the APRN is a... llowed to prescribe. State law does not require the APRN to practice under physician supervision. How would the APRN’s prescriptive authority be described? 2. Which factors increase the need for APRNs to have full prescriptive authority? 3. Which factors could be attributed to limited prescriptive authority for APRNs? Select all that apply. 4. Which aspects support the APRN’s provision for full prescriptive authority? Select all that apply. 5. Which aspects support the APRN’s provision for full prescriptive authority? Select all that apply. 6. A family nurse practitioner practicing in Maine is hired at a practice across state lines in Virginia. Which aspect of practice may change for the APRN? Rosenthal: Lehne's Pharmacotherapeutics for Advanced Practice Providers, 2nd Ed. Chapter 2: Rational Drug Selection and Prescription Writing Test Bank Multiple Choice 7. How can collaboration with a pharmacist improve positive outcomes for patients? Select all that apply. 8. A patient presents with delirium tremens requiring Ativan administration. The provider of care is not in the facility. Which action by the nurse is most appropriate? 9. A patient with chronic pain calls the provider’s office to request a refill on theiroxycontin. Which action is most appropriate? 10. A patient prescribed amoxicillin for streptococcal pharyngitis reports new onset of a flat, itchy red rash on the chest and neck. Which action is most important? 11. A patient taking three medications for hypertension is diagnosed with COPD. Which action should be taken prior to prescribing medications to treat COPD? 12. A patient with diabetes reports losing their job and an inability to purchase required medications. Which action is most appropriate? 13. A patient recently prescribed hydrocodone calls to report theyare unable to fill the prescription. Which factors could contribute to the inability to fill the prescription? Select all that apply. Rosenthal: Lehne's Pharmacotherapeutics for Advanced Practice Providers, 2nd Ed. Chapter 3: Promoting Positive Outcomes of Drug Therapy Test Bank Multiple Choice 14. A patient reports that a medication prescribed for recurrent migraine headaches is not working. Which action should be taken first? 15. A patient is prescribed metronidazole for bacterial vaginosis. Which patient history finding would be most concerning? 16. A patient is using a metered-dose inhaler containing albuterol for asthma. The medication label instructs the patient to administer “two puffs every 4 hours as needed for coughing or wheezing.” The patient reports feeling jittery sometimes when taking the medication, and she doesn’t feel that the medication is always effective. Which action is most appropriate? 17. A patient newly diagnosed with diabetes is to be discharged from the hospital. Which action should be taken first during medication education? 18. The drug manual states that older adult patients are at increased risk for hepatic side effects. Which action is most important when prescribing this medication to an 80-year-old patient? 19. A patient recently diagnosed with HIV is prescribed several medications to treat the condition. Which factors could impact the patient’s adherence to the treatment regimen? Select all that apply. 20. A patient diagnosed with bipolar disorder is prescribed daily lithium. Which action is most important to determine if the therapeutic level is maintained? Rosenthal: Lehne's Pharmacotherapeutics for Advanced Practice Providers, 2nd Ed. Chapter 4: Pharmacokinetics, Pharmacodynamics, and Drug Interactions Test Bank Multiple Choice 21. The nurse administers naloxone [Narcan] to a patient who has received a toxic dose of morphine sulfate. The nurse understands that the naloxone is effective because of which action? 22. A patient is taking drug X and receives a new prescription for drug Y, which is listed as an inducing agent. The nurse caring for this patient understands that this patient mayrequire doses of drug . 23. The nurse is preparing to administer penicillin G intramuscularly to a child. The child’s parents ask why the drug cannot be given in an oral liquid form. What is the nurse’s reply? 24. Which statement about food and drug interactions is true? AnalysisREF: p. 39TOP: Nursing Process: Assessment MSC: NCLEX Client Needs Category: Physiologic Integrity: Reduction of Risk Potential 25. A nurse is teaching a patient about a drug that induces P-glycoprotein. The nurse will explain that this drug may cause which effect on other drugs? 26. A patient claims to get better effects with a tablet of Brand X of a drug than with a tablet of Brand Y of the same drug. Both brands contain the same amount of the active ingredient. What does the nurse know to be most likely? 27. Two nurses are discussing theories of drug-receptor interaction. Which statements are true regarding the affinity of a drug and its receptor? Select all that apply. 28. A patient receives a drug that has a narrow therapeutic range. The nurse administering this medication will expect to do what? 29. What occurs when a drug binds to a receptor in the body? 30. A patient is receiving intravenous gentamicin. A serum drug test reveals toxic levels. The dosing is correct, and this medication has been tolerated bythis patient in the past. Which could be a probable cause of the test result? 31. A patient reports becoming “immune” to a medication because it no longer works to alleviate symptoms. The nurse recognizes that this decreased effectiveness is likely caused by: 32. A patient who is taking morphine for pain asks the nurse how a pain medication can also cause constipation. What does the nurse know about morphine? 33. The nurse is administering morning medications. The nurse gives a patient multiple medications, two of which compete for plasma albumin receptor sites. As a result of this concurrent administration, the nurse can anticipate that what might occur? Select all that apply. 34. When administering medications to infants, it is important to remember which of the following? Select all that apply. Rosenthal: Lehne's Pharmacotherapeutics for Advanced Practice Providers, 2nd Ed. Chapter 5: Adverse Drug Reactions and Medication Errors Test Bank Multiple Choice 35. A nursing student is preparing to give a medication that has a boxed warning. The student asks the nurse what this means. What will the nurse explain about boxed warnings? 36. A nurse is preparing to administer a drug. Upon reading the medication guide, the nurse notes that the drug has been linked to symptoms of Parkinson disease in some patients. What will the nurse do? 37. Which patients are at increased risk for adverse drug events? Select all that apply. 38. A nurse provides teaching to a patient who will begin taking a drug with a known risk of hepatotoxicity. Which statement by the patient indicates a need for further teaching? 39. A nurse is reviewing a medication administration record before administering medications. Which order will the nurse implement? 40. A patient is given a new medication and reports nausea within an hour after taking the drug. The nurse consults the drug information manual and learns that nausea is not an expected adverse effect of this drug. When the next dose is due, what will the nurse do? 41. A nurse is preparing to give an antibiotic to a patient who reports being allergic to antibiotics. Before giving the medication, what will the nurse do first? 42. A patient is taking sertraline [Zoloft] for depression, and the provider ordersazithromycin [Zithromax] to treat an infection. What will the nurse do? 43. A patient is given a drug for the first time and develops shortness of breath. The patient’s heart rate is 76 beats/minute, the respiratory rate is 20 breaths/minute, and the blood pressure is 120/70 mm Hg. The nurse checks a drug administration manual to make sure the correct dose was given and learns that some patients taking the drug experience shortness of breath. The nurse will contact the provider to report what? 44. Which are effective ways to help prevent medication errors? Select all that apply. 45. A patient is taking a drug that has known toxic side effects. What will the nurse do? 46. A patient is being discharged after surgery. During the admission history, the nurse learned that the patient normally consumes two or three glasses of wine each day. The prescriber has ordered hydrocodone with acetaminophen [Lortab] for pain. What will the nurse do? 47. Which actions occur in 90% of fatal medication errors? Select all that apply. Rosenthal: Lehne's Pharmacotherapeutics for Advanced Practice Providers, 2nd Ed. Chapter 6: Individual Variation in Drug Responses Test Bank Multiple Choice 48. A nurse is caring for a woman with breast cancer who is receiving tamoxifen. A review of this patient’s chart reveals a deficiency of the CYP2D6 gene. The nurse will contact the provider to suggest: 49. Which groups of people are especially sensitive to medication effects? Select all that apply. 50. A post-operative patient who is worried about pain control will be discharged several days after surgery. The nurse providing discharge teaching tells the patient that the prescribed Lortab is not as strong as the morphine the patient was given in the immediate post-operative period. Which response is the patient likely to experience? 51. A patient has been taking narcotic analgesics for chronic pain for several months. The nurse caring for this patient notes that the prescribed dose is higher than the recommended dose. The patient has normal vital signs, is awake and alert, and reports mild pain. What does the nurse recognize about this patient? 52. A patient asks a nurse why a friend who is taking the same drug responds differently to that drug. The nurse knows that the most common variation in drug response is due to differences in each patient’s: 53. The nurse is assessing a newly admitted older patient who has recently lost 15 pounds. The nurse notes that the patient is taking warfarin (Coumadin). Which laboratory tests willthe nurse discuss with this patient’s provider? 54. A nurse is preparing to care for a patient who is receiving digoxin. When screening for potential adverse effects fromthis drug, the nurse will review which of this patient’s laboratoryresults? 55. A nurse administers the same medication in the same preparation in the same dose to several patients and notes that some patients have a better response to the drug than others. What is the most likely explanation for this phenomenon? 56. A nurse is teaching a group of women about medications. The women want to know why so many drugs have unpredictable effects in women. The nurse will tell them that: 57. The U.S. Food and Drug Administration (FDA) recommends genetic testing of patients receiving certain medications. Genetic testing helps prescribers: Chapter 7 Genetic and Genomic Considerations in Pharmacotherapeutics 1. Drugs do not metabolize the same way in all people. For what patient would a nurse expect to assess for an alteration in drug metabolism? 2. A patient presents to the emergency department with a drug level of 50 units/mL. The half-life of this drug is 1 hour. With this drug, concentrations above 25 units/mL are considered toxic and no more drug is given. How long will it take for the blood level to reach the non-toxic range? 3. A patient has recently moved from Vermont to Southern Florida. The patient presents to the clinic complaining of dizzy spells and weakness. While conducting the admission assessment, the patient tells the nurse that he have been on the same antihypertensive drug for 6 years and had stable blood pressures and no adverse effects. Since his move, he has been having problems and he feels that the drug is no longer effective. The clinic nurse knows that one possible reason for the change in the effectiveness of the drug could be what? 4. An important concept taught by the nurse when providing medication teaching is the need to provide a complete list of medications taken to health care providers to avoid what? 5. A pharmacology student asks the instructor what an accurate description of a drug agonist is. What is the instructors best response? 6. A nurse is caring for a patient who has been receiving a drug by the intramuscular route but will receive the drug orally after discharge. How does the nurse explain the increased dosage prescribed for the oral dose? 7. A nurse is working as a member of a research team involved in exploring the unique response to drugs each individual displays based on genetic make-up. What is this area of study is called? 8. The nurse uses what term to describe the drug level required to have a therapeutic effect? 9. A nurse is caring for a patient who is supposed to receive two drugs at the same time. What is the nurses priority action? 10. The nurse is talking with a group of nursing students who are doing clinical hours on the unit. A student asks if all intramuscular (IM) drugs are absorbed the same. What factor would the floor nurse tell the students to affect absorption of the IM administration of drugs? 11. The patient is taking a drug that affects the body by increasing cellular activity. Where does this drug work on the cell? 12. Several processes enable a drug to reach a specific concentration in the body. Together they are called dynamic equilibrium. What are these processes? (Select all that apply.) 13. A nurse is administering digoxin to a patient. To administer medications so that the drug is as effective as possible, the nurse needs to consider what? Pharmacoeconomics includes all costs involved in drug therapy. Chapter 8: Drug Therapy During Pregnancy and Breast-Feeding Test Bank Multiple Choice 58. Which types of drugs taken by a pregnant patient are more likely to have effects on afetus? 59. A patient in her second trimester of pregnancy tells the nurse she is worried that a medication she took before knowing she was pregnant might have harmed the fetus. What will the nurse do? 60. A patient who has just learned she is pregnant has stopped using a prescription medication that she takes for asthma because she does not want to harm her baby. What will the nurse tell her? 61. A pregnant patient in active labor is admitted to the emergency department. A toxicology screen and a physical assessment reveal that the patient is an active heroin addict. The nurse who cares for the neonate after delivery should anticipate which clinical manifestations? 62. A pregnant patient asks the nurse about the safe use of medications during the third trimester. What will the nurse tell her about drugs taken at this stage? 63. A patient has just given birth to a baby boy with a cleft palate. The nurse will review the patient’s medication history with special emphasis on drugs taken during which period? 64. A nurse is caring for a patient and her newborn immediately after delivery. The patient’s medication history includes prenatal vitamins throughout pregnancy, one or two glasses of wine before knowing she was pregnant, occasional use of an albuterol inhaler in her last trimester, and intravenous morphine during labor. What will the nurse expect to do? 65. A woman who is breastfeeding her infant must take a prescription medication for 2 weeks. The medication is safe, but the patient wants to make sure her baby receives as little of the drug as possible. What will the nurse tell the patient to do? 66. A nursing student asks the nurse why more is not known about the teratogenic effects of maternal medication ingestion during pregnancy. Which response by the nurse is correct? 67. A nurse is teaching a class to a group of pregnant patients. The nurse correctly teaches that the highest risk of teratogen-induced gross malformations exists during which time? Rosenthal: Lehne's Pharmacotherapeutics for Advanced Practice Providers, 2nd Ed. Chapter 9: Drug Therapy in Pediatric Patients Test Bank Multiple Choice 1. A nurse is caring for an infant after a surgical procedure. After ensuring that the ordered dose is appropriate for the infant’s age and weight, the nurse administers a narcotic analgesic intravenously. When assessing the infant 15 minutes later, the nurse notes respirations of 22 breaths/minute and a heart rate of 110 beats/minute. The infant is asleep in the parent’s arms and does not awaken when vital signs are assessed. The nurse understands that these findings are the result of: 2. A child will receive 750 mg of an antibiotic for 10 days. The child attends day care. The drug may be dosed in several ways and is available in two concentrations. Which dosing regimen will the nurse discuss with the child’s provider? 3. Parents ask the nurse why an over-the-counter cough suppressant with sedative side effects is not recommended for infants. Which response by the nurse is correct? 4. A nurse caring for a 5-year-old child notes that the child has discoloration of several teeth. When taking a medication history, the nurse will ask about which group of medications? 5. An infant has allergies and often develops a pruritic rash when exposed to allergens. The infant’s parents ask the nurse about using a topical antihistamine. What should the nurse tell them? 6. An infant is receiving a medication that has a narrow therapeutic range. The nurse reviews the medication information and learns that the drug is excreted by the kidneys. When giving the medication, the nurse will assess the infant for: 7. A pediatric nurse is teaching nursing students to calculate medication doses for childrenusing a formula based on body surface area. Which statement by a nursing student indicates understanding of the teaching? 8. A pediatric nurse is teaching nursing students about medication administration in children. Which statement by a student indicates an understanding of the teaching? 9. A nurse is teaching nursing students about pediatric medication administration. What will the nurse include when discussing pediatric drug research? 10. A prescriber has ordered medication for a newborn. The medication is eliminated primarily by hepatic metabolism. The nurse expects the prescriber to: 11. The parents of a child with asthma ask the nurse why their child cannot use oral corticosteroids more often, because they are so effective. The nurse will offer which information that is true for children? Rosenthal: Lehne's Pharmacotherapeutics for Advanced Practice Providers, 2nd Ed. Chapter 10: Drug Therapy in Geriatric Patients Test Bank Multiple Choice 12. A nurse is obtaining a drug history from an older adult patient who is taking multiple medications prescribed by different providers. Which two medications taken together create a reason for concern? 13. A nurse is teaching a group of nursing students about administering medications to older adult patients. Which statement by a student indicates a need for further teaching? 14. A nurse is preparing to give medications to four older patients who are all taking multiple medications. Which patient is most likely to have an adverse drug reaction related to increased drug effects? 15. A nurse is concerned about renal function in an 84-year-old patient who is taking several medications. What will the nurse assess? 16. A nurse is preparing to teach a forgetful older adult patient about a multiple drug regimen to follow after discharge from the hospital. To help promote adherence, what will the nurse do? 17. Based on changes in hepatic function in older adult patients, which adjustment should the nurse expect for oral medications that undergo extensive first pass metabolism? 18. A thin older adult woman is admitted to the hospital after several days of vomiting, diarrhea, and poor intake of foods and fluids. She has not voided since admission. In preparing to care for this patient, the nurse will look for what laboratory values to help guide medication administration? Select all that apply. 19. A nurse is reviewing an older adult patient’s chart before giving medications. Which patient information is of most concern? 20. A nurse is caring for an older adult patient during the immediate postoperative period after a total hip replacement. The surgeon has ordered meperidine [Demerol] for severe pain. What will the nurse do? 21. The nurse is teaching a group of nursing students about adherence to medication in older adults. Which statement by a student indicates understanding of the teaching? 22. A nurse is making a home visit to an older adult woman who was recently discharged home from the hospital with a new prescription. The nurse notes that a serum drug level drawn the day before was subtherapeutic. What will the nurse do next? 23. An older adult patient is admitted to the hospital for treatment of an exacerbation of a chronic illness. Admission laboratory work reveals an extremely low serum drug level of the drug used to treat this condition. The patient has brought the medication to the hospital, along with other medications taken. The patient’s renal and hepatic function tests are normal. What might the nurse suspect as a likely cause of this finding? Rosenthal: Lehne's Pharmacotherapeutics for Advanced Practice Providers, 2nd Ed. Chapter 11: Basic Principles of Neuropharmacology Test Bank Multiple Choice 24. A nursing student asks about drugs that interfere with the termination of transmitter action. Which statement by the nurse is correct? 25. A nurse is teaching a group of nursing students about neuropharmacology. Which statement by a student about peripheral nervous system (PNS) drugs indicates a need for further teaching? 26. A nurse is preparing to administer a medication and learns that it is a nonselective agonist drug. What does the nurse understand about this drug? 27. A patient has allergies and takes an antihistamine. The patient wants to know how the drug works. The nurse understands that antihistamines work because they are what? 28. A patient receiving botulinum toxin injections to control muscle spasticity asks how the drug works. The nurse knows that this drug affects the transmitter acetylcholine by: 29. A nurse is administering drug X to a patient. The drug information states that the drug acts by activating receptors in the peripheral nervous system by increasing transmitter synthesis. The nurse understands that the effect of this drug is to: 30. A patient receives morphine and shows signs of toxicity. The prescriber orders naloxone [Narcan] to reverse the effects of the morphine. The nurse understands that the naloxone reverses morphine toxicity by which action on morphine receptor sites? 31. A nurse learns about a drug that interferes with transmitter storage in the PNS. The transmitter affected by this drug causes an increased heart rate. What response will the nurse expect to see when this drug is administered? Rosenthal: Lehne's Pharmacotherapeutics for Advanced Practice Providers, 2nd Ed. Chapter 12: Physiology of the Peripheral Nervous System Test Bank Multiple Choice 32. What is the target organ when a beta1 agonist is administered? 33. A nurse is teaching a group of nursing students about neurotransmitters. Which statement by a student about acetylcholine indicates a need for further teaching? 34. A nurse is administering an agonist drug that acts on postganglionic neurons of the sympathetic nervous system. Which response will the nurse expect to see? 35. A nurse is teaching a patient about a medication that alters sympathetic nervous system functions. To evaluate understanding, the nurse asks the patient to describe which functions the sympathetic nervous system regulates. Which answer indicates the need for further teaching? 36. A patient is wheezing and short of breath. The nurse assesses a heart rate of 88 beats/minute, a respiratory rate of 24 breaths/minute, and a blood pressure of 124/78 mm Hg. The prescriber orders a nonspecific beta agonist medication. Besides evaluating the patient for a reduction in respiratory distress, the nurse will monitor for which side effect? 37. Many medications list side effects that include dry mouth, constipation, and urinary retention. What kinds of effects are these? 38. A patient is to receive a beta agonist. Before administration of this medication, which assessment finding would most concern the nurse? 39. A nurse is explaining activation of beta2 receptors to nursing students during a clinical rotation at the hospital. Which statement by a student demonstrates a need for further teaching? 40. A pregnant patient is in premature labor. Which class of drug will she be given? Rosenthal: Lehne's Pharmacotherapeutics for Advanced Practice Providers, 2nd Ed. Chapter 13: Muscarinic Agonists 41. A prescriber has ordered bethanechol [Urecholine] for a postoperative patient who has urinary retention. The nurse reviews the patient’s chart before giving the drug. Which part of the patient’s history would be a contraindication to using this drug? 42. Bethanechol [Urecholine] is used to treat urinary retention but is being investigated for use in which other condition? 43. An older adult patient who lives alone and is somewhat forgetful has an overactive bladder (OAB) and reports occasional constipation. The patient has tried behavioral therapy to treat the OAB without adequate results. Which treatment will the nurse anticipate for this patient? 44. A nurse is helping a nursing student who is administering a medication to a patient with myasthenia gravis. Which statement by the student indicates the need for further teaching? 45. A patient is experiencing toxic side effects from atropine, including deliriumand hallucinations. Which medication will the nurse expect to administer? 46. A prescriber has ordered pilocarpine [Pilocar]. A nurse understands that the drug stimulates muscarinic receptors and would expect the drug to have which action? 47. A nurse is caring for a patient who has myasthenia gravis. The prescriber has ordered neostigmine [Prostigmin]. An important initial nursing action before administration of the medication includes assessing: 48. A patient who has myasthenia gravis will be taking neostigmine [Prostigmin]. What will the nurse emphasize when teaching this patient about the medication? 49. A patient who has esophageal cancer is experiencing dry mouth and the provider orders oral pilocarpine to treat this symptom. What will the nurse expect to teach this patient about this medication? Chapter 14 Muscarinic Antagonists 1. The nurse administers an adrenergic blocking agent in order to prevent release of what neurotransmitter? 2. What medication, if ordered for an 8-year-old patient, should the nurse question? (Select all that apply.) 3. A nurse is working with a patient who is taking an adrenergic blocking agent. While assessing the patients medication history, the nurse discovers that the patient takes several alternative therapies. What herb is the nurse concerned may interact with the adrenergic blocking agent and affect the patients blood glucose level? 4. A priority nursing assessment for a patient who is to receive an alpha- or beta- adrenergic blocking agent would be what? 5. Bisoprolol (Zebeta) would be the drug of choice for which patient with a diagnosis of hypertension? 6. What would be the teaching priority for a diabetic patient being treated with a nonselective beta-blocker? 7. The nurse is caring for a patient who is receiving an adrenergic blocking agent. While writing the care plan for this patient what nursing diagnoses would be most appropriate concerning comfort? 8. A nurse is providing discharge instructions to a patient who is taking atenolol (Tenormin) to treat hypertension. What would the nurse teach the patient regarding a possible drugdrug interaction? 9. A busy patient with many responsibilities is to have a medication ordered to treat her hypertension. To increase compliance with drug therapy, what drug would be a good choice for this patient? 10. The nurse provides patient teaching for a patient who has a new order for nadolol (Corgard) to treat hypertension. What statement by the patient concerning nadolol (Corgard) would indicate that the teaching has been effective? Rosenthal: Lehne's Pharmacotherapeutics for Advanced Practice Providers, 2nd Ed. Chapter 15: Adrenergic Agonists Test Bank Multiple Choice 50. A nursing student asks the nurse about receptor specificity of adrenergic agonist medications. What will the nurse say? 51. A nurse is teaching parents how to use an Epi-Pen for their child, who has a peanut allergy. Which statement by the parents indicates understanding of the teaching? 52. A nursing student asks the nurse why epinephrine, and not other adrenergic agonists, is used to treat anaphylactic shock. What will the nurse tell the student? 53. Dopamine is administered to a patient who has been experiencing hypotensive episodes. Other than an increase in blood pressure, which indicator would the nurse use to evaluate a successful response? 54. A nursing student asks why albuterol, which is selective for beta2 receptors, causes an increased heart rate in some patients. How should the nurse respond? 55. Because they cause vasoconstriction, alpha1-adrenergic agonists are especially useful for: 56. A nurse is teaching a nursing student about the two classes of adrenergic agonist drugs. Which statement by the nursing student indicates understanding of the teaching? 57. A patient with asthma uses albuterol [Ventolin] for wheezing. The nurse assesses the patient and notes vital signs of HR, 96 beats/minute; RR, 18 breaths/minute; and BP, 116/78 mmHg. The patient has clear breath sounds and hand tremors. What will the nurse do? 58. A nurse is administering intravenous dopamine [Intropin] to a patient in the intensive care unit. Which assessment finding would cause the most concern? 59. A patient is receiving dobutamine [Dobutrex] as a continuous infusion in the immediate postoperative period. The patient also is receiving a diuretic. What adverse drug reactions are possible in this patient? Select all that apply. 60. A patient brought to the emergency department requires sutures. The prescriber orders a local anesthetic with epinephrine. The nurse understands that epinephrine is ordered to: 61. A patient is admitted to the intensive care unit for treatment of shock. The prescriber orders isoproterenol [Isuprel]. The nurse expects this drug to increase tissue perfusion in this patient by activating: Rosenthal: Lehne's Pharmacotherapeutics for Advanced Practice Providers, 2nd Ed. Chapter 16: Adrenergic Antagonists Test Bank Multiple Choice 62. A patient with pheochromocytoma is admitted for surgery. The surgeon has ordered analpha- blocking agent to be given preoperatively. What does the nurse understand about this agent? 63. A patient with type 1 diabetes is taking NPH insulin, 30 units every day. A nurse notes that the patient is also taking metoprolol [Lopressor]. What education should the nurse provide to the patient? 64. A nurse is caring for a newborn 1 day after delivery. The infant’s mother used betaxolol during pregnancy. The nurse will expect to monitor this infant for which condition? 65. A patient with migraines is started on a beta blocker. The nurse explains the benefits oftaking the medication for migraines. Which statement by the patient indicates an understanding of the medication’s effects? 66. A nurse is discussing phentolamine [OraVerse] with a nursing student. Which statement bythe student indicates the need for further teaching? 67. The nurse assesses a patient who has been given phentolamine [OraVerse] to treat pheochromocytoma. The nurse notes a blood pressure of 76/52 mm Hg and a heart rate of 90 beats/minute. Which action by the nurse is correct? 68. Which are conditions that may be treated using beta blockers? Select all that apply. 69. Which are adverse effects of alpha blockade? Select all that apply. 70. The nurse is discussing home management with a patient who will begin taking an alpha- adrenergic antagonist for hypertension. Which statement bythe patient indicates understanding of the teaching? 71. A nurse is teaching nursing students about the use of alpha-adrenergic antagonists. Which statement by a student indicates the need for further teaching? 72. A patient taking a beta blocker complains of shortness of breath. The patient has respirations of 28 breaths/minute, a blood pressure of 162/90 mm Hg, and a pulse of 88 beats/minute. The nurse auscultates crackles in all lung fields. The nurse understands that these assessments are consistent with: 73. A male patient is being treated for benign prostatic hyperplasia and has stopped taking his alpha-adrenergic antagonist medication because of ejaculatory difficulties. Which medication does the nurse expect the provider to prescribe? 74. A patient is taking a beta-adrenergic antagonist medication for angina pectoris and asks the nurse how the drug works to relieve the discomfort associated with this condition. Which statement bythe patient after the nurse’s teaching indicates understanding of the drug’s effects? 75. A nurse prepares to administer propranolol [Inderal] to a patient recovering from acute myocardial infarction. The patient’s heart rate is 52 beats/minute, and the rhythm is regular. What action should the nurse take next? 76. A patient will begin taking propranolol [Inderal] for hypertension. Which statement by the nurse is important when teaching this patient about the medication? Rosenthal: Lehne's Pharmacotherapeutics for Advanced Practice Providers, 2nd Ed. Chapter 17: Indirect-Acting Antiadrenergic Agents Test Bank Multiple Choice 77. A patient who has been taking clonidine [Catapres] for severalweeks complains of drowsiness and constipation. What will the nurse do? 78. A prescriber has ordered methyldopa for a patient with hypertension. The nurse teaches the patient about drug actions, adverse effects, and the ongoing blood tests necessary with this drug. The nurse is correct to tell the patient what? 79. A prescriber has ordered clonidine [Catapres] for a patient who has hypertension. The nurse teaches the patient about side effects of this drug. Which statement by the patient indicates understanding of the teaching? 80. A prescriber has ordered methyldopa for a female patient with hypertension. The nurse understands that which laboratory tests are important before beginning therapy with this drug? Select all that apply. 81. A nurse is teaching nursing students about the pharmacology of methyldopa. Which statement by a student indicates the need for further teaching? 82. A patient complains to the nurse that the clonidine [Catapres] recently prescribed for hypertension is causing drowsiness. Which response by the nurse to this concern is appropriate? 83. A prescriber orders clonidine [Kapvay] ER tablets for a 12-year-old child. The nurse understands that this drug is being given to treat which condition? 84. A patient with hypertension has a previous history of opioid dependence. Which medication would the nurse question? 85. Clonidine is approved for the treatment of which conditions? Select all that apply. .DIF: Cognitive Level: ApplicationREF: pp. 133TOP: Nursing Process: Assessment MSC: 86. A prescriber orders transdermal clonidine [Catapres TTS] for a patient with hypertension. What will the nurse teach this patient? 87. A nurse is teaching a patient about a new prescription for reserpine [Serpasil] for hypertension. Which statement by the patient indicates the need for further teaching? a. Practice Providers, 2nd Ed. Chapter 18: Introduction to Central Nervous System Pharmacology Test Bank Multiple Choice 88. An infant who receives a drug that does not produce CNS side effects in adults exhibits drowsiness and sedation. The nurse understands that this is because of differences in which physiologic system in infants and adults? a. Blood-brain barrier b. First-pass effect c. Gastrointestinal absorption d. Renal filtration ANS: A The blood-brain barrier is not fully developed at birth, making infants much more sensitive to CNS drugs than older children and adults. CNS symptoms may include sedation and drowsiness. The first-pass effect and GI absorption affect metabolism and absorption of drugs, and renal filtration affects elimination of drugs, all of which may alter drug levels.DIF: Cognitive Level: ComprehensionREF: pp. 139TOP: Nursing Process: N/A MSC: NCLEX Client Needs Category: Physiologic Integrity: Physiologic Adaptation 89. Which monoamines act as neurotransmitters in the central nervous system? Select all that apply. a. Acetylcholine b. Norepinephrine c. Serotonin d. Dopamine e. Epinephrine f. Histamine ANS: B , C , D , E Acetylcholine and histamines are not monoamines.DIF: Cognitive Level: ComprehensionREF: pp. 140TOP: Nursing Process: Assessment MSC: NCLEX Client Needs Category: Physiologic Integrity: Pharmacologic and Parenteral Therapies 90. A patient asks a nurse to explain what drug tolerance means. The nurse responds by telling the patient that when tolerance occurs, it means the patient: a. has developed a psychologic dependence on the drug. b. may need increased amounts of the drug over time. c. will cause an abstinence syndrome if the drug is discontinued abruptly. d. will have increased sensitivity to drug side effects. ANS: B When tolerance develops, a dose increase may be needed, because a decreased response mayoccur with prolonged use. Psychologic dependence involves cravings for drug effects and does not define tolerance. Physical dependence occurs when the drug becomes necessary for the brain to function “normally,” meaning the patient should be weaned from the drug slowly to prevent an abstinence syndrome. Patients may have a decreased sensitivity to drug side effects over time as the brain adapts to the medication.DIF: Cognitive Level: ComprehensionREF: pp. 140TOP: Nursing Process: N/A MSC: NCLEX Client Needs Category: Physiologic Integrity: Physiologic Adaptation 91. A group of nursing students asks a nurse to explain the blood-brain barrier. The nursewould be correct to say that the blood-brain barrier: a. prevents some potentially toxic substances from crossing into the central nervoussystem. b. causes infants to be less sensitive to CNS drugs and thus require larger doses. c. allows only ionized or protein-bound drugs to cross into the central nervous system. d. prevents lipid-soluble drugs from entering the central nervous system. ANS: A The blood-brain barrier can prevent some drugs and some toxic substances from entering the CNS. The blood-brain barrier in infants is not fully developed, so infants are more sensitive to CNS drugs and often require lower doses. The blood-brain barrier prevents highly ionized and protein- bound drugs from crossing into the CNS and allows lipid-soluble drugs and those that can cross via specific transport systems to enter.DIF: Cognitive Level: AnalysisREF: pp. 139TOP: Nursing Process: Evaluation MSC: NCLEX Client Needs Category: Physiologic Integrity: Pharmacologic and Parenteral Therapies 92. A nurse is teaching a group of students about how CNS drugs are developed. Which statement by a student indicates a need for further teaching? a. “Central nervous systemdrug development relies on observations of their effects on human behavior.” b. “Studies of new central nervous system drugs in healthy subjects can produce paradoxical effects.” c. “Our knowledge of the neurochemicaland physiologic changes that underlie mental illness is incomplete.” d. “These drugs are developed based on scientific knowledge of CNS transmittersand receptors.” ANS: D The deficiencies in knowledge about how CNS transmitters and receptors work make systematic development of CNS drugs difficult. Testing in healthy subjects often leads either to no effect or to paradoxical effects. Medical knowledge of the neurochemical and physiologic changes underlying mental illness is incomplete. The development of CNS drugs depends less on knowledge of how the CNS functions and how these drugs affect that process and more on how administering one of these agents leads to changes in behavior.DIF: Cognitive Level: AnalysisREF: pp. 139TOP: Nursing Process: Assessment MSC: NCLEX Client Needs Category: Physiologic Integrity: Pharmacologic and Parenteral Therapies 93. A nurse is teaching a group of nursing students how the CNS adapts to psychotherapeutic medications. Which statement by a nursing student indicates a need for further teaching? a. “Adaptation can lead to tolerance of these drugs with prolonged use.” b. “Adaptation helps explain how physical dependence occurs.” c. “Adaptation often must occur before therapeutic effects develop.” d. “Adaptation results in an increased sensitivity to side effects over time.” ANS: D With adaptation of the central nervous system to prolonged exposure to CNS drugs, many adverse effects diminish and therapeutic effects remain. Adaptation helps explain how tolerance and physical dependence occur, as the brain adapts to the presence of the drug. Therapeutic effects can take several weeks to manifest, because they appear to work by initiating adaptive changes in the brain.DIF: Cognitive Level: AnalysisREF: pp. 140TOP: Nursing Process: Evaluation MSC: NCLEX Client Needs Category: Physiologic Integrity: Pharmacologic and ParenteralTherapies 94. A psychiatric nurse is teaching a patient about an antidepressant medication. The nurse tells the patient that therapeutic effects may not occur for several weeks. The nurse understands that this is likely the result of: a. changes in the brain as a result of prolonged drug exposure. b. direct actions of the drug on specific synaptic functions in the brain. c. slowed drug absorption across the blood-brain barrier. d. tolerance to exposure to the drug over time. ANS: A It is thought that beneficial responses to central nervous system (CNS) drugs are delayed because they result from adaptive changes as the CNS modifies itself in response to prolonged drug exposure, and that the responses are not the result of the direct effects of the drugs on synaptic functions. The blood-brain barrier prevents protein-bound and highly ionized drugs from crossing into the CNS, but it does not slow the effects of drugs that can cross the barrier. Tolerance is a decreased response to a drug after prolonged use.DIF: Cognitive Level: ApplicationREF: pp. 140TOP: Nursing Process: Implementation MSC: NCLEX Client Needs Category: Physiologic Integrity: Pharmacologic and Parenteral Therapies 95. Which are medical applications of central nervous systemdrugs? Select all that apply. a. Analgesia b. Anesthesia c. Depression d. Euphoria e. Seizure control ANS: A , B , E CNS drugs have medical uses for pain management, anesthesia, and seizure control. Depression and euphoria are side effects that can contribute to abuse of these drugs.DIF: Cognitive Level: ComprehensionREF: pp. 139TOP: Nursing Process: Assessment MSC: NCLEX Client Needs Category: Physiologic Integrity: Pharmacologic and Parenteral Therapies Rosenthal: Lehne's Pharmacotherapeutics for Advanced Practice Providers, 2nd Ed. Chapter 19: Drugs for Parkinson's Disease Test Bank Multiple Choice 96. A nursing student wants to know how carbidopa can be effective for treating Parkinson disease if it prevents the conversion of levodopa to dopamine. The nurse explains that carbidopa: a. can be taken with high-protein meals. b. does not cross the blood-brain barrier. c. has dopamine-like effects of its own. d. reduces abrupt loss of effect. ANS: B Carbidopa inhibits decarboxylation of levodopa in the intestine and peripheral tissues, leading to more levodopa in the CNS. Carbidopa cannot cross the blood-brain barrier, so it does not have this action in the CNS. Carbidopa is not given with high-protein meals. Carbidopa does not have dopamine-like effects. Carbidopa does not affect abrupt loss of effect.DIF: Cognitive Level: AnalysisREF: p. 149TOP: Nursing Process: Evaluation MSC: NCLEX Client Needs Category: Physiologic Integrity: Pharmacologic and Parenteral Therapies 97. A 25-year-old patient has been newly diagnosed with Parkinson disease, and the prescriber is considering using pramipexole [Mirapex]. Before beginning therapy with this drug, the nurse will ask the patient about: a. any history of alcohol abuse or compulsive behaviors. b. any previous history of hypertension. c. difficulty falling asleep or staying asleep. d. whether any family members have experienced psychoses. ANS: A Pramipexole has been associated with impulse control disorders, and this risk increases in patients with a history of alcohol abuse or compulsive behaviors. Pramipexole increases the risk of hypotension and sleep attacks, so a history of hypertension or insomnia would not be cautionary. Unlike with levodopa, the risk of psychoses is not increased.DIF: Cognitive Level: ApplicationREF: p. 153TOP: Nursing Process: Planning MSC: NCLEX Client Needs Category: Physiologic Integrity: Reduction of Risk Potential 98. A patient with Parkinson disease is taking levodopa/carbidopa [Sinemet] and reports occasional periods of loss of drug effect lasting from minutes to several hours. The nurse questions the patient further and discovers that these episodes occur at different times related to the medication administration. The nurse will contact the provider to discuss: a. administering a catechol-O-methyltransferase (COMT) inhibitor, such as entacapone. b. adding the DA-releasing agent amantadine to the regimen. c. giving a direct-acting dopamine agonist. d. shortening the dosing interval of levodopa/carbidopa. ANS: A This patient is describing abrupt loss of effect, or the “off” phenomenon, which is treated with entacapone or another COMT inhibitor. Amantadine is used to treat dyskinesias. A direct-acting dopamine agonist is useful for gradual loss of effect, which occurs at the end ofthe dosing interval as the dose is wearing off. Shortening the dosing interval does not help with abrupt loss of effect.DIF: Cognitive Level: ApplicationREF: p. 154TOP: Nursing Process: Assessment MSC: NCLEX Client Needs Category: Physiologic Integrity: Pharmacologic and Parenteral Therapies 99. A nurse is discussing motor symptoms with a patient with Parkinson disease who has been taking levodopa/carbidopa [Sinemet] for 9 months and who is now having regular tics. Which statement by the patient indicates understanding of this symptom? a. “I may need to try a lower dose of Sinemet to reduce my tics.” b. “My provider may order clozapine to treat these tics.” c. “These tics are an indication that my dose of Sinemet is too low.” d. “This means I will have to have surgery to stop the symptoms.” ANS: A Levodopa can cause movement disorders, generally within the first year of therapy. If they occur, a lower dose of levodopa may be required to alleviate them. Clozapine is an antipsychotic used to treat levodopa-induced psychoses. Movement disorders generally occur as the dose of levodopa increases. Surgery is a last option for treating movement disorders, after amantadine fails.DIF: Cognitive Level: ApplicationREF: p. 148TOP: Nursing Process: Implementation MSC: NCLEX Client Needs Category: Physiologic Integrity: Pharmacologic and Parenteral Therapies 100. A hospitalized patient with Parkinson disease who is receiving apomorphine to treat “off” episodes develops nausea and vomiting. The nurse will discuss the use of which medication with the patient’s provider? a. Levodopa [Dopar] b. Ondansetron [Zofran] c. Prochlorperazine [Compazine] d. Trimethobenzamide [Tigan] ANS: D Trimethobenzamide can be used as an antiemetic in patients treated with apomorphine. Serotonin receptor agonists (e.g., ondansetron) and dopamine receptor antagonists (e.g., prochlorperazine) cannot be used, because they increase the risk of serious postural hypotension. Levodopa only increases nausea and vomiting.DIF: Cognitive Level: ApplicationREF: p. 154TOP: Nursing Process: Planning MSC: NCLEX Client Needs Category: Physiologic Integrity: Pharmacologic and Parenteral Therapies 101. A patient has been diagnosed with Parkinson disease (PD) and begins treatment with levodopa/carbidopa [Sinemet]. After several months of therapy, the patient reports no change in symptoms. The nurse will expect the provider to: a. add a dopamine agonist. b. discuss the “on-off” phenomenon. c. increase the dose of Sinemet. d. reevaluate the diagnosis. ANS: D Patients beginning therapy with levodopa/carbidopa should expect therapeutic effects to occur after several months of treatment. Levodopa is so effective that a diagnosis of PD should be questioned if the patient fails to respond in this time frame. Adding a dopamine agonist is not indicated. The “on-off” phenomenon occurs when therapeutic effects are present. Increasing the dose of levodopa/carbidopa is not indicated.DIF: Cognitive Level: ApplicationREF: p. 146TOP: Nursing Process: Implementation MSC: NCLEX Client Needs Category: Physiologic Integrity: Pharmacologic and Parenteral Therapies 102. A nursing student wants to know why a patient who has been taking levodopa [Dopar] for years will now receive levodopa/carbidopa [Sinemet]. The nurse explains the reasons that levodopa as a single agent is no longer available. Which statement by the student indicates a need for further education? a. “Carbidopa increases the availability of levodopa in the central nervous system.” b. “Carbidopa reduces the incidence of nausea and vomiting.” c. “Combination products reduce peripheral cardiovascular side effects.” d. “Combination products cause fewer dyskinesias and decrease psychosis.” ANS: D Adding carbidopa to levodopa does not reduce the incidence of dyskinesias or psychosis. In fact, carbidopa can increase the intensity and the speed of onset of these effects. Carbidopa inhibits decarboxylation of levodopa in the intestine and peripheral tissues, leading to more levodopa in the CNS. Carbidopa cannot cross the blood-brain barrier, so it does not have this action in the CNS. Peripheral side effects are reduced, including nausea, vomiting, and cardiovascular effects.DIF: Cognitive Level: AnalysisREF: p. 149TOP: Nursing Process: Evaluation MSC: NCLEX Client Needs Category: Physiologic Integrity: Pharmacologic and Parenteral Therapies 103. A patient has taken levodopa [Dopar] for Parkinson disease for 2 weeks but reports no improvement in the symptoms. Which response by the nurse is correct? a. “Another agent will be needed to manage your symptoms.” b. “Double the dose to see whether an effect occurs.” c. “It may take several months for a response to occur.” d. “The prescriber may need to change your drug regimen.” ANS: C A full therapeutic response with levodopa maytake several months to develop. Until the true effect of the dose is seen, it is not necessary to change to another drug, increase the dose, or change the drug regimen.DIF: Cognitive Level: ApplicationREF: p. 148TOP: Nursing Process: Implementation MSC: NCLEX Client Needs Category: Physiologic Integrity: Pharmacologic and Parenteral Therapies 104. A patient with Parkinson disease is taking levodopa/carbidopa [Sinemet]. The prescriber orders bromocriptine [Parlodel] to treat dyskinesias. The nurse notesthat the patient is agitated, and the patient reports having frequent nightmares. The nurse will contact the provider to discuss: a. adding an antipsychotic medication. b. changing from bromocriptine to cabergoline [Dostinex]. c. reducing the dose of bromocriptine. d. reducing the dose of levodopa/carbidopa. ANS: C Bromocriptine is used to treat levodopa-induced dyskinesias and has dose-dependent psychologic side effects. The nurse should suggest reducing the dose of this drug to minimize these side effects. Adding an antipsychotic medication is not indicated. Cabergoline is not approved for this use. Reducing the dose of levodopa/carbidopa is not indicated.DIF: Cognitive Level: ApplicationREF: nurse is teaching a group of nurses about Parkinson medications. The nurse is correct to state that one side effect associated with pramipexole [Mirapex] that is less likely to occur with other dopamine agonists is: a. sleep attacks. b. dizziness. c. hallucinations. d. dyskinesias. ANS: A A few patients taking pramipexole have experienced sleep attacks, or an overwhelming and irresistible sleepiness that comes on without warning. Dizziness, hallucinations, and dyskinesias are listed as side effects of pramipexole and other dopamine agonists.DIF: Cognitive Level: ApplicationREF: p.153TOP: Nursing Process: Implementation MSC: NCLEX Client Needs Category: Physiologic Integrity: Pharmacologic and Parenteral Therapies Rosenthal: Lehne's Pharmacotherapeutics for Advanced Practice Providers, 2nd Ed. Chapter 20: Drugs for Alzheimer's Disease Test Bank Multiple Choice nurse is caring for an older adult man who has Alzheimer disease (AD). The patient’s daughter wants to know if testing can be done to determine her risk for developing the disease. What will the nurse tell her? a. Female gender is known to increase the risk. b. Genetic testing can provide a definitive measure of the risk. c. Patients with the apolipoprotein E2 gene (apoE2) are more likely to develop thedisease. d. Advancing age and family history are known risk factors. ANS: D Advancing age and a positive family history are the only two known risk factors. Female gender is not a known risk; the increased incidence among females may be the result of women living longer than men. No definitive genetic tests are available. The presence of apoE2 seems to be protective.DIF: Cognitive Level: ApplicationREF: pp. 162TOP: Nursing Process: Assessment MSC: NCLEX Client Needs Category: Physiologic Integrity: Physiologic Adaptation patient will begin taking a cholinesterase inhibitor for early Alzheimer disease. The nurse is teaching the patient’s spouse about the medication. Which statement by the spouse indicates a need for further teaching? 108. The spouse of a patient who is newly diagnosed with Alzheimer disease asks the nurse if medications will prevent the need for nursing home care. Which response by the nurse is correct? 109. A nurse is caring for an older adult patient who has Alzheimer disease. The patient is taking a cholinesterase inhibitor drug. Which side effects would concern the nurse? 110. An older adult patient with Alzheimer disease is admitted to the hospital. The patient’s spouse reports that the patient is often confused and gets lost walking to the store, which is three blocks from their home. That evening, the nurse observes the patient pacing the hall and screaming. What will the nurse do? 111. A nurse is preparing to administer memantine [Nemanda] to a patient and notes a slight elevation in the patient’s creatinine clearance level. What will the nurse expect the provider to order for this patient? 112. The spouse of a patient with Alzheimer disease asks a nurse for more information about the rivastigmine [Exelon] transdermal patch that is being used. Which statement by the spouse indicates a need for further explanation? 113. A patient is worried about the risk of developing Alzheimer disease, because both parents had the disease. The nurse will tell this patient that known risk factors include what? Select all that apply. a. Advanced age b. Alcoholism c. Family history d. Gender e. Obesity ANS: A , C The major known risk factor for AD is advancing age; the only other known risk factor is a family history of AD. Alcoholism, gender, and obesity are not known risk factors.DIF: Cognitive Level: ComprehensionREF: pp. 162TOP: Nursing Process: Assessment MSC: NCLEX Client Needs Category: Physiologic Integrity: Physiologic Adaptation 114. A nurse is teaching a group of nursing students about the use of memantine [Namenda] for Alzheimer disease. Which statement by a student indicates understanding of the teaching? a. “Memantine is indicated for patients with mild to moderate Alzheimer disease.” b. “Memantine modulates the effects of glutamate to alter calcium influx into neurons.” c. “Memantine prevents calcium from leaving neurons, which improves their function.” d. “Memantine and donepezil combined may stop progression of Alzheimer disease.” ANS: B Memantine modulates the effects of glutamate, which is involved in calcium influx into neuronal cells. Memantine is used for patients with moderate to severe AD. Memantine does not prevent calcium from leaving cells; it only affects the influx of calcium. In studies, although the effects of memantine and donepezil appear to be synergistic or may confer independent benefits, they only demonstrate improvement in cognitive function and not a stop in disease progression.DIF: Cognitive Level: AnalysisREF: pp. 165TOP: Nursing Process: Implementation MSC: NCLEX Client Needs Category: Physiologic Integrity: Pharmacologic and Parenteral Therapies 115. An older adult patient has confusion, memory loss, and disorientation in familiar surroundings. The patient has been taking donepezil [Aricept] 10 mg once daily for 6 months. The patient’s symptoms have begun to worsen, and the patient’s spouse asks if the medication dose can be increased. What will the nurse tell the spouse? a. The dose can be increased, because the patient has been taking the drug for longer than 3 months. b. The dose can be increased to twice daily dosing instead of once daily dosing. c. The increase in symptoms is the result of hepatotoxicity from the medication’s side effects. d. The patient must take the drug for longer than 1 year before the dose can be increased. ANS: A Donepezil is given for mild, moderate, and severe AD, and dosing may be increased, although it must be titrated up slowly. For patients with moderate to severe AD who have taken 10 mg once daily for at least 3 months, the dose can be increased to 23 mg once daily. Donepezil is not given twice daily. Donepezil does not cause hepatotoxicity; hepatotoxicity occurs with tacrine, the first acetylcholinesterase (AChE) inhibitor, which now is rarely used. Dosing is increased after 3 months, not 1 year.DIF: Cognitive Level: ApplicationREF: pp. 164TOP: Nursing Process: Planning MSC: NCLEX Client Needs Category: Physiologic Integrity: Pharmacologic and Parenteral Therapies Rosenthal: Lehne's Pharmacotherapeutics for Advanced Practice Providers, 2nd Ed. Chapter 21: Drugs for Seizure Disorders Test Bank Multiple Choice 116. A nurse is assessing a patient who becomes motionless and seems to stare at the wall andthen experiences about 60 seconds of lip smacking and hand wringing. What should the nurse do? a. Ask the patient about a history of absence seizures. b. Contact the provider to report symptoms of a complex partial seizure. c. Notify the provider that the patient has had a grand mal seizure. d. Request an order for intravenous diazepam [Valium] to treat status epilepticus. ANS: B This patient showed signs of a complex partial seizure, characterized by impaired consciousness beginning with a period of motionlessness with a fixed gaze, followed by a period ofautomatism. The entire episode generally lasts 45 to 90 seconds. Absence seizures are characterized by loss of consciousness for a brief period (about 10 to 30 seconds) and may involve mild, symmetric motor activity or no motor signs. A grand mal seizure is characterized by jaw clenching and rigidity followed by alternating muscle relaxation and contraction and then periods of cyanosis, all with a loss of consciousness. Status epilepticus is a seizure that persists for 30 minutes or longer.DIF: Cognitive Level: ApplicationREF: p. 167TOP: Nursing Process: Assessment MSC: NCLEX Client Needs Category: Physiologic Integrity: Physiologic Adaptation 117. A nurse counsels a patient who is to begin taking phenytoin [Dilantin] for epilepsy. Which statement by the patient indicates understanding of the teaching? a. “I should brush and floss my teeth regularly.” b. “Once therapeutic blood levels are reached, they are easy to maintain.” c. “I can consume alcohol in moderation while taking this drug.” d. “Rashes are a common side effect but are not serious.” ANS: A Gingival hyperplasia occurs in about 20% of patients who take phenytoin. It can be minimized with good oral hygiene, so patients should be encouraged to brush and floss regularly. Because small fluctuations in phenytoin levels can affect response, maintaining therapeutic levels is not easy. Patients should be cautioned against consuming alcohol while taking phenytoin. Rashes can be serious and should be reported immediately.DIF: Cognitive Level: ApplicationREF: p. 174TOP: Nursing Process: Planning MSC: NCLEX Client Needs Category: Physiologic Integrity: Pharmacologic and Parenteral Therapies 118. A nurse is caring for a patient who has been taking an antiepileptic drug for several weeks. The nurse asks the patient if the therapy is effective. The patient reports little change in seizure frequency. What will the nurse do? a. Ask the patient to complete a seizure frequency chart for the past few weeks. b. Contact the provider to request an order for serum drug levels. c. Reinforce the need to take the medications as prescribed. d. Request an order to increase the dose of the antiepileptic drug. ANS: B If medication therapy is not effective, it is important to measure serum drug levels of the medication to determine whether therapeutic levels have been reached and to help monitor patient compliance. Patients should be asked at the beginning of therapy to keep a seizure frequencychart to help deepen their involvement in therapy; asking for historical information is not helpful. Until it is determined that the patient is not complying, the nurse should not reinforce the need to take the medication. Until the drug level is known, increasing the dose is not indicated.DIF: Cognitive Level: ApplicationREF: p. 172TOP: Nursing Process: Evaluation MSC: NCLEX Client Needs Category: Physiologic Integrity: Pharmacologic and Parenteral Therapies 119. A patient is to begin taking phenytoin [Dilantin] for seizures. The patient tells the nurse that she is taking oral contraceptives. What will the nurse tell the patient? a. She may need to increase her dose of phenytoin while taking oral contraceptives. b. She should consider a different form of birth control while taking phenytoin. c. She should remain on oral contraceptives, because phenytoin causes birth defects. d. She should stop taking oral contraceptives, because they reduce the effectiveness of phenytoin. ANS: B Because phenytoin can reduce the effects of oral contraceptive pills (OCPs) and because avoiding pregnancy is desirable when taking phenytoin, patients should be advised to increase the dose of oral contraceptives or use an alternative method of birth control. Increasing the patient’s dose of phenytoin is not necessary; OCPs do not affect phenytoin levels. Phenytoin is linked to birth defects; OCPs have decreased effectiveness in patients treated with phenytoin, and the patient should be advised to increase the OCP dose or to use an alternative form of birth control. OCPs do not alter the effects of phenytoin.DIF: Cognitive Level: ApplicationREF: p. 175TOP: Nursing Process: Planning MSC: NCLEX Client Needs Category: Physiologic Integrity: Pharmacologic and Parenteral Therapies 120. A patient with a form of epilepsy that may have spontaneous remission has been taking an AED for a year. The patient reports being seizure free for 6 months and asks the nurse when the drug can be discontinued. What will the nurse tell the patient? a. AEDs must be taken for life to maintain remission. b. Another AED will be substituted for the current AED. c. The provider will withdraw the drug over a 6- to 12-week period. d. The patient should stop taking the AED now and restart the drug if seizures recur. ANS: C The most important rule about withdrawing AEDs is that they should be withdrawn slowly over 6 weeks to several months to reduce the risk of status epilepticus (SE). AEDs need not be taken for life if seizures no longer occur. Substituting one AED for another to withdraw AED therapy is not recommended. Stopping an AED abruptly increases the risk of SE.DIF: Cognitive Level: ApplicationREF: p. 172TOP: Nursing Process: Implementation MSC: NCLEX Client Needs Category: Physiologic Integrity: Pharmacologic and Parenteral Therapies 121. A patient who has a seizure disorder is admitted to the hospital after an increase in seizure frequency, and the prescriber orders carbamazepine [Tegretol] 100 mg twice daily to be added to the patient’s medication regimen. The nurse reviewing the patient’s medical history notes that the patient is already taking lamotrigine [Lamictal] 375 mg twice daily. The nurse will contact the provider to discuss which action? a. Reducing the carbamazepine dose to 50 mg twice daily b. Reducing the lamotrigine dose to 225 mg twice daily c. Increasing the carbamazepine dose to 200 mg twice daily d. Increasing the lamotrigine dose to 500 mg twice daily ANS: D Carbamazepine induces hepatic drug-metabolizing enzymes and can increase the rate at which lamotrigine and other drugs are metabolized; therefore, patients taking any of these drugs would need an increased dose. Reducing the dose of either drug is not indicated. Increasing the dose of carbamazepine may be necessary but only after serum drug levels have been checked.DIF: Cognitive Level: AnalysisREF: p. 188TOP: Nursing Process: Assessment MSC: NCLEX Client Needs Category: Physiologic Integrity: Reduction of Risk Potential 122. A patient who is taking oral contraceptives begins taking valproic acid [Depakote] for seizures. After a week of therapy with valproic acid, the patient tells the nurse that she is experiencing nausea. What will the nurse do? a. Ask the patient if she is taking the valproic acid with food, because taking the drug on an empty stomach can cause gastrointestinal side effects. b. Contact the provider to request an order for a blood ammonia level, because hyperammonemia can occur with valproic acid therapy. c. Suggest that the patient perform a home pregnancy test, because valproic acid can reduce the efficacy of oralcontraceptives. d. Suspect that hepatotoxicity has occurred, because this is a common adverse effect of valproic acid. ANS: A Gastrointestinal effects, including nausea, vomiting, and indigestion, are common with valproic acid and can be minimized by taking the drug with food or using an enteric-coated product. Hyperammonemia can occur when valproic acid is combined with topiramate. Signs ofpregnancy usually do not occur within a week, so this is less likely. Hepatotoxicity is a rare adverse effect.DIF: Cognitive Level: ApplicationREF: p. 189TOP: Nursing Process: Evaluation MSC: NCLEX Client Needs Category: Physiologic Integrity: Pharmacologic and Parenteral Therapies 123. A nurse is discussing partial versus generalized seizures with a group of nursing students. Which statement by a student indicates understanding of the teaching? a. “Febrile seizures are a type of generalized tonic-clonic seizure.” b. “Generalized seizures are characterized by convulsive activity.” c. “Partial seizures do not last as long as generalized seizures.” d. “Patients having partial seizures do not lose consciousness.” ANS: A Febrile seizures typically manifest as a tonic-clonic seizure of short duration and are a type of generalized seizure. Generalized seizures may be convulsive or nonconvulsive. Partial seizures may last longer than some types of generalized seizures. Patients with complex partial seizures and secondarily generalized seizures, which are types of partial seizures, may lose consciousness.DIF: Cognitive Level: AnalysisREF: p. 168TOP: Nursing Process: Implementation MSC: NCLEX Client Needs Category: Physiologic Integrity: Pharmacologic and Parenteral Therapies 124. A nurse provides teaching for a patient with a newly diagnosed partial complex seizure disorder who is about to begin therapy with antiepileptic drugs (AEDs). Which statement by the patient indicates understanding of the teaching? a. “Even with an accurate diagnosis of my seizures, it may be difficult to find an effective drug.” b. “I will soon know that the drugs are effective by being seizure free for several months.” c. “Serious side effects may occur, and if they do, I should stop taking the medication.” d. “When drug levels are maintained at therapeutic levels, I can expect to be seizure free.” ANS: A Even with an accurate diagnosis of seizures, many patients have to try more than one AED to find a drug that is effective and well tolerated. Unless patients are being treated for absence seizures, which occur frequently, monitoring of the clinical outcome is not sufficient for determining effectiveness, because patients with convulsive seizures often have long seizure-free periods. Serious side effects may occur, but withdrawing a drug precipitously can induce seizures. Not all patients have seizure control with therapeutic drug levels, because not all medications work for all patients.DIF: Cognitive Level: ApplicationREF: p. 170TOP: Nursing Process: Planning MSC: NCLEX Client Needs Category: Physiologic Integrity: Pharmacologic and Parenteral Therapies Rosenthal: Lehne's Pharmacotherapeutics for Advanced Practice Providers, 2nd Ed. Chapter 22: Drugs for Muscle Spasm and Spasticity Test Bank Multiple Choice 125. A nurse is teaching the parent of a child with spastic quadriplegia about intrathecal baclofen [Lioresal]. Which statement by the parent indicates a need for further teaching? a. “I can expect my child to be more drowsy when receiving this medication.” b. “I should not notice any change in my child’s muscle strength.” c. “I will contact the provider if my child is constipated or cannot urinate.” d. “If my child has a seizure, I should stop giving the medication immediately.” ANS: D Seizures may occur if oral baclofen is withdrawn abruptly; seizures are not an adverse effect of baclofen. If intrathecal baclofen is stopped abruptly, patients can experience life-threatening effects, so parents should be advised not to stop the drug abruptly. The central nervous system effects of baclofen include drowsiness and lethargy, so these effects are expected. Baclofen does not reduce muscle strength. It can cause constipation and urinary retention, and patients should be advised to contact their provider so that these conditions can be treated.DIF: Cognitive Level: ApplicationREF: pp. 197TOP: Nursing Process: Planning MSC: NCLEX Client Needs Category: Physiologic Integrity: Reduction of Risk Potential 126. A nurse is admitting a patient to the hospital. The patient reports taking oral baclofen [Lioresal] but stopped taking the drug the day before admission. The nurse would be correct to anticipate which adverse effects? a. Weakness and dizziness b. Fatigue and drowsiness c. Seizures and hallucinations d. Respiratory depression and coma ANS: C Abrupt discontinuation of baclofen is associated with visual hallucinations, paranoid ideation, and seizures. Central nervous system effects of baclofen include weakness, dizziness, fatigue, and drowsiness. Respiratory depression is a result of overdose of baclofen.DIF: Cognitive Level: ApplicationREF: pp. 195TOP: Nursing Process: Diagnosis MSC: NCLEX Client Needs Category: Physiologic Integrity: Reduction of Risk Potential 127. A patient with cerebral palsy has severe muscle spasticity and muscle weakness. The patient is unable to take anything by mouth. The nurse is correct to anticipate that which medication will be ordered for home therapy? a. Baclofen [Lioresal] b. Dantrolene [Dantrium] c. Diazepam [Valium] d. Metaxalone [Skelaxin] ANS: A Baclofen is used to treat muscle spasticity associated with multiple sclerosis, spinal cord injury, and cerebral palsy. It does not reduce muscle strength, so it will not exacerbate this patient’s muscle weakness. It can be given intrathecally, via an implantable pump, and therefore is a good choice for patients who cannot take medications by mouth. Dantrolene must be given by mouth or intravenously and so would not be a good option for this patient. It also causes muscle weakness. Diazepam is not the first-line drug of choice. Alternative routes to PO administration are IM, IV, or by rectum. Metaxalone is used to treat localized muscle spasms caused by injury and is not used for cerebral palsy.DIF: Cognitive Level: ApplicationREF: pp. 196TOP: Nursing Process: Planning MSC: NCLEX Client Needs Category: Physiologic Integrity: Reduction of Risk Potential 128. A patient has localized muscle spasms after an injury. The prescriber has ordered tizanidine [Zanaflex] to alleviate the spasms. When obtaining the patient’s health history, the nurse should be concerned about which possible reason for considering another drug? a. Concomitant use of aspirin b. A history of hepatitis c. A history of malignant hyperthermia d. Occasional use of alcohol ANS: B Hepatotoxicity is a serious potential problem in a patient receiving tizanidine, because the drug can cause liver damage. Baseline liver enzymes should be obtained before dosing and periodically thereafter. Analgesic anti-inflammatory drugs commonly are used in conjunction with centrally acting muscle relaxants, so using aspirin is not a concern. This drug does not contribute to malignant hyperthermia. Patients should be advised to avoid alcohol when taking this drug, but a history of occasional alcohol use is not a contraindication.DIF: Cognitive Level: ApplicationREF: pp. 193TOP: Nursing Process: Diagnosis MSC: NCLEX Client Needs Category: Physiologic Integrity: Pharmacologic and Parenteral Therapies 129. A patient with multiple sclerosis needs pharmacologic treatment for spasticity to begin strengthening exercises to improve walking ability. The nurse anticipates that which medication will be ordered for spasticity? a. Baclofen [Lioresal] b. Dantrolene [Dantrium] c. Diazepam [Valium] d. Metaxalone [Skelaxin] ANS: A Baclofen is used to treat spasms associated with multiple sclerosis. It has no direct muscle relaxant effects, so it does not reduce muscle strength. Dantrolene works well to reduce spasms, but it also has significant effects on muscle strength. Diazepam is not the first-line drug of choice, but it could be used because it does not reduce muscle strength. Metaxalone is not indicated to treat spasms caused by multiple sclerosis.DIF: Cognitive Level: ApplicationREF: pp. 196TOP: Nursing Process: Planning MSC: NCLEX Client Needs Category: Physiologic Integrity: Reduction of Risk Potential 130. A patient with cerebral palsy who has been receiving baclofen [Lioresal] via gastrostomy tube for 3 months is admitted to the hospital for evaluation of new-onset seizures. What may the nurse suspect to be the cause of these seizures? a. Baclofen toxicity b. Common adverse effect of baclofen c. Idiopathic causes related to disease process d. Missed doses of baclofen ANS: D Baclofen does not appear to cause physical dependence, but abrupt discontinuation has been associated with adverse reactions. Abrupt withdrawal of oral baclofen can cause visual hallucinations, paranoid ideation, and seizures and should be considered when a patient develops these symptoms. Seizures are not a symptom of baclofen toxicity.DIF: Cognitive Level: ApplicationREF: pp. 197TOP: Nursing Process: Evaluation MSC: NCLEX Client Needs Category: Physiologic Integrity: Reduction of Risk Potential 131. Which drugs are used to treat spasticity? Select all that apply. a. Baclofen [Lioresal] b. Dantrolene [Dantrium] c. Diazepam [Valium] d. Metaxalone [Skelaxin] e. Tizanidine [Flexeril] ANS: A , B , C Three drugs—baclofen, dantrolene, and diazepam—are used to treat spasticity. Baclofen and diazepam act in the CNS, whereas dantrolene acts directly on skeletal muscles. With the exception of diazepam, drugs used for muscle spasm, such as metaxalone and tizanidine, are not effective for treating spasticity.DIF: Cognitive Level: ComprehensionREF: pp. 193TOP: Nursing Process: Assessment MSC: NCLEX Client Needs Category: Physiologic Integrity: Pharmacologic and Parenteral Therapies 132. A patient who has a lower back injury exhibits muscle spasms. The provider orders cyclobenzaprine [Flexeril] 10 mg three times a day. What will the nurse include when teaching this patient about this drug? a. “This drug carries some risk of developing hallucinations and psychotic symptoms.” b. “This medication may cause your urine to turn brown, black, or dark green.” c. “You may experience blurred vision, dry mouth, or constipation.” d. “You will need to have liver function tests performed while taking this medication.” ANS: C Cyclobenzaprine has significant anticholinergic effects and patients should be warned about dry mouth, blurred vision, and constipation. Tizanidine can cause hallucinations and psychotic symptoms. Methocarbamol mayturn urine brown, black, or green, which is a harmless side effect. Tizanidine and metaxalone can cause liver toxicity and require monitoring.DIF: Cognitive Level: ApplicationREF: pp. 193TOP: Nursing Process: Implementation MSC: NCLEX Client Needs Category: Physiologic Integrity: Pharmacologic and Parenteral Therapies 133. Which patient should receive dantrolene [Dantrium] with caution? a. A 20-year-old woman with a spinal cord injury b. A 45-year-old man with a history of malignant hyperthermia c. A 55-year-old woman with multiple sclerosis d. An 8-year-old child with cerebral palsy ANS: C Dose-related liver damage is the most serious adverse effect of dantrolene and is most common in women older than 35 years. Dantrolene is used to treat spasticity associated with multiple sclerosis, cerebral palsy, and spinal cord injury, so all of these patients would be candidates for this agent. Dantrolene also is used to treat malignant hyperthermia.DIF: Cognitive Level: ApplicationREF: pp. 196TOP: Nursing Process: Assessment MSC: NCLEX Client Needs Category: Physiologic Integrity: Reduction of Risk Potential 134. A nurse is caring for a patient receiving intrathecal baclofen [Lioresal]. The patient is unresponsive. After asking a coworker to contact the provider, the nurse anticipates performing which intervention? a. Preparing to support respirations b. Administering an antidote to baclofen c. Administering diazepam to prevent seizures d. Obtaining an electrocardiogram ANS: A An overdose of baclofen can produce coma and respiratory depression, so the nurse would be correct to suspect overdose in this patient. Respiratory support is essential to prevent a fatal outcome. There is no antidote for baclofen overdose. Diazepam would not be indicated, because seizures are not a result of baclofen overdose and may further depress respirations. An electrocardiogram is not indicated for this patient.DIF: Cognitive Level: ApplicationREF: pp. 195TOP: Nursing Process: Implementation MSC: NCLEX Client Needs Category: Physiologic Integrity: Reduction of Risk Potential Rosenthal: Lehne's Pharmacotherapeutics for Advanced Practice Providers, 2nd Ed. Chapter 23: Local Anesthetics Test Bank Multiple Choice 135. A nurse is caring for a patient in the immediate postoperative period after surgery in which a spinal anesthetic was used. The patient has not voided and complains of headache. The patient has a pulse of 62 beats/minute, a respiratory rate of 16 breaths/minute, and a blood pressure of 92/48 mm Hg. Which action by the nurse is appropriate? a. Contact the anesthetist to request an order for ephedrine. b. Have the patient sit up to relieve the headache pain. c. Lower the head of the bed to a 10- to 15-degree head-down position. d. Obtain an order for a urinary catheter for urinary retention. ANS: C Spinal anesthetics have several adverse effects, but the most significant is hypotension caused by the venous dilation that occurs from blockade of sympathetic nerves. The result is decreased blood return to the heart, which causes reduced cardiac output and a drop in blood pressure. The first step in treating this is to put the patient in a 10- to 15-degree head-downposition to promote venous return to the heart. Ephedrine or phenylephrine is used if the first measure fails. Spinal headaches are common; the intervention for this is to have the patient assume a supine position. Urinary retention can occur secondary to autonomic blockade; it is a concern if the patient has not voided for 8 hours after the procedure, but not in the immediate postoperative period.DIF: Cognitive Level: ApplicationREF: pp. 199TOP: Nursing Process: Implementation MSC: NCLEX Client Needs Category: Physiologic Integrity: Reduction of Risk Potential 136. A nurse is assisting a physician who is preparing to suture a superficial laceration on a patient’s leg. The physician asks the nurse to draw up lidocaine with epinephrine. The nurse understands that epinephrine is used with the lidocaine to: a. allow more systemic absorption to speed up metabolism of the lidocaine. b. increase the rate of absorption of the lidocaine. c. improve perfusion by increasing blood flow to the area. d. prolong anesthetic effects and reduce the risk of systemic toxicity from lidocaine. ANS: D Epinephrine causes vasoconstriction, which reduces local blood flow and delays systemic absorption of lidocaine, which prolongs local anesthetic effects and reduces the risk of systemic toxicity. Epinephrine slows the rate of absorption. Epinephrine delays systemic absorption of lidocaine, so metabolism is slowed and the effects are prolonged in the periphery. Epinephrine does not increase local blood flow.DIF: Cognitive Level: ApplicationREF: pp. 198TOP: Nursing Process: Planning MSC: NCLEX Client Needs Category: Physiologic Integrity: Pharmacologic and Parenteral Therapies 137. A nurse is preparing a patient to go home from the emergency department after receiving sutures for a laceration on one hand. The provider used lidocaine with epinephrine as a local anesthetic. Which symptom in this patient causes the most concern? a. Difficulty moving the fingers of the affected hand b. Inability to feel pressure at the suture site c. Nervousness and tachycardia d. Sensation of pain returning to the wound ANS: C Absorption of the vasoconstrictor can cause systemic effects, including nervousness and tachycardia. If severe, alpha- and beta-adrenergic antagonists can be given. Local anesthetics are nonselective modifiers of neuronal function and also can block motor neurons, so it is expected that patients may have difficulty with movement. The sensation of pressure also is affected and is an expected effect. As the local anesthetic wears off, the sensation of pain will return.DIF: Cognitive Level: ApplicationREF: pp. 198TOP: Nursing Process: Evaluation MSC: NCLEX Client Needs Category: Physiologic Integrity: Pharmacologic and Parenteral Therapies 138. A nurse is teaching a group of nursing students about local anesthetics. Which statement by a student reflects an understanding of the teaching? a. “Local anesthetics affect large myelinated neurons first.” b. “Local anesthetics affect motor and sensory nerves.” c. “Local anesthetics do not block temperature perception.” d. “Local anesthetics do not cause systemic effects.” ANS: B Local anesthetics are nonselective modifiers of neuronal function. They block actions in motor and sensory nerves. They affect small myelinated neurons first. They block temperature, pressure, and pain sensation. When absorbed into the systemic circulation, they can cause systemic effects.DIF: Cognitive Level: ApplicationREF: pp. 197TOP: Nursing Process: Implementation MSC: NCLEX Client Needs Category: Physiologic Integrity: Pharmacologic and Parenteral Therapies 139. Which are important considerations when spinal anesthesia is being administered? Select all that apply. a. Encouraging the patient to remain supine after the procedure b. Having epinephrine available in case of adverse effects c. Injecting a local anesthetic into the sacral region d. Placing the agent into the spinal column outside the dura mater e. Using an anesthetic solution from a single-dose vial ANS: A , B , E Spinal anesthesia frequently causes headache; these headaches are posture-dependent and can be relieved by having the patient assume a supine position. Epinephrine is used if significant hypotension occurs as an adverse effect of spinal anesthesia and should be kept nearby. All anesthesia used in spinal anesthesia must be preservative free, so it would be drawn up from a single-dose vial. Spinal anesthesia is produced by injecting local anesthetic into the subarachnoid space with injection made in the lumbar region below the termination of the cord. Epidural anesthesia is injected into the spinal column outside the dura mater.DIF: Cognitive Level: ComprehensionREF: pp. 199TOP: Nursing Process: Implementation MSC: NCLEX Client Needs Category: Physiologic Integrity: Pharmacologic and Parenteral Therapies 140. A nurse is discussing the use of cocaine as a local anesthetic with a nursing student. Which statement by the student indicates understanding of this agent? a. “Anesthetic effects develop slowly and persist for several hours.” b. “Cocaine is a local anesthetic administered by injection.” c. “Vasoconstrictors should not be used as adjunct agents with this drug.” d. “When abused, cocaine causes physical dependence.” ANS: C Cocaine should not be combined with epinephrine or other vasoconstrictors, because it causes vasoconstriction itself, and the combination could precipitate severe hypertension. Cocaine has a rapid onset of effects, which last about 1 hour. It is used only topically for anesthesia. Although subject to widespread abuse with profound psychologic dependence, it does not cause substantial physical dependence.DIF: Cognitive Level: AnalysisREF: pp. 200TOP: Nursing Process: Evaluation MSC: NCLEX Client Needs Category: Physiologic Integrity: Reduction of Risk Potential 141. A nurse is assisting a physician who is performing a circumcision on a newborn. The physician asks the nurse to prepare lidocaine and epinephrine for injection to provide anesthesia. What will the nurse do? a. Ask the provider why an injectable anesthetic is being used for this procedure. b. Draw up the medication as ordered and prepare the infant for the procedure. c. Make sure that seizure precautions are in place. d. Question the use of the epinephrine for this procedure. ANS: D The physician is preparing to use infiltration anesthesia by injecting the local anesthetic directly into the immediate area of surgery. Epinephrine can be used in some cases but should never be used in areas supplied by end arteries, such as the penis, toes, fingers, nose, or ears, because restriction of blood flow in these areas can result in gangrene. Injectable agents are appropriate for this procedure. The nurse should not draw up the medication as requested, because the combination of agents can harm the patient. Seizure precautions are not necessary.DIF: Cognitive Level: ApplicationREF: pp. 198TOP: Nursing Process: Planning MSC: NCLEX Client Needs Category: Physiologic Integrity: Reduction of Risk Potential 142. A nurse is assisting the physician during a procedure in which a local anesthetic is administered. Within a few minutes of administration of the anesthetic, the patient has a pulse of 54 beats/minute, respirations of 18 breaths/minute, and a blood pressure of 90/42 mm Hg. The nurse should monitor the patient for further signs of: a. heart block. b. anaphylaxis. c. central nervous system excitation. d. respiratory depression. ANS: A When absorbed in a sufficient amount, local anesthetics can affect the heart and blood vessels. These drugs suppress excitability in the myocardium and conduction system and can cause hypotension, bradycardia, heart block, and potentially cardiac arrest. Anaphylaxis would be manifested by hypotension, bronchoconstriction, and edema of the glottis. Central nervous system excitation would be manifested by hyperactivity, restlessness, and anxiety and may be followed by convulsions. No evidence indicates respiratory depression; this patient’s respirations are within normal limits.DIF: Cognitive Level: ApplicationREF: pp. 199TOP: Nursing Process: Implementation MSC: NCLEX Client Needs Category: Physiologic Integrity: Reduction of Risk Potential 143. A nurse is teaching a patient who has a second-degree burn on one arm about the use of a topical anesthetic for pain. Which statement by the patient indicates understanding of the teaching? a. “I will apply a thin layer of the medication to a small area of skin.” b. “I will cover the burn with a dressing after applying the medication.” c. “I will make sure to apply the medication to the entire burn area.” d. “I will use the medication only on the most painful, blistered areas.” ANS: A Topical anesthetics can be absorbed in sufficient amounts to cause serious and even life- threatening systemic toxicity, so they should be applied in the smallest amount needed to as small an area as possible. Covering the site increases the skin’s temperature, which increases absorption, so this should be avoided. Applying the medication to a large area increases systemic absorption. Applying the medication to broken skin increases systemic absorption.DIF: Cognitive Level: ApplicationREF: pp. 200TOP: Nursing Process: Implementation MSC: NCLEX Client Needs Category: Physiologic Integrity: Reduction of Risk Potential 144. Vasoconstrictors are combined with local anesthetics for whichreasons? Select all that apply. a. To enhance absorption b. To reduce the risk of toxicity c. To prevent bradycardia d. To shorten the duration of action e. To prolong anesthesia ANS: B , E Vasoconstrictors, when combined with local anesthetics, reduce the risk of toxicity and prolong the anesthetic effects. Vasoconstrictors, when combined with local anesthetics, do not speed up the absorption process, prevent bradycardia, or shorten the duration of action.DIF: Cognitive Level: ApplicationREF: pp. 198TOP: Nursing Process: Diagnosis MSC: NCLEX Client Needs Category: Physiologic Integrity: Pharmacologic and Parenteral Therapies 145. A patient receives an epidural anesthetic during labor and delivery. The nurse caring for the newborn in the immediate postpartum period will observe the infant for: a. bradycardia. b. hypoglycemia. c. jitteriness. d. tachypnea. ANS: A Local anesthetics can cross the placenta, causing bradycardia and central nervous system (CNS) depression in the infant. They do not affect blood glucose. Jitteriness is a sign of CNS excitation. Increased respirations are not an adverse effect in the newborn.DIF: Cognitive Level: ApplicationREF: pp. 199TOP: Nursing Process: Evaluation MSC: NCLEX Client Needs Category: Physiologic Integrity: Reduction of Risk Potential Rosenthal: Lehne's Pharmacotherapeutics for Advanced Practice Providers, 2nd Ed. Chapter 24: Opioid Analgesics, Opioid Antagonists, and Nonopioid Centrally Acting Analgesics Test Bank Multiple Choice 146. A patient who has had abdominal surgery has been receiving morphine sulfate via a patient- controlled analgesia (PCA) pump. The nurse assesses the patient and notes that the patient’s pupils are dilated and that the patient is drowsy and lethargic. The patient’s heart rate is 84 beats/minute, the respiratoryrate is 10 breaths/minute, and the blood pressure is 90/50 mm Hg. What will the nurse do? a. Discuss possible opiate dependence with the patient’s provider. b. Encourage the patient to turn over and cough and take deep breaths. c. Note the effectiveness of the analgesia in the patient’s chart. d. Prepare to administer naloxone and possibly ventilatory support. ANS: D Opioid toxicity is characterized by coma, respiratory depression, and pinpoint pupils. Although pupils are constricted initially, they may dilate as hypoxia progresses, which also causes blood pressure to drop. This patient has a respiratory rate of fewer than 12 breaths/minute, dilated pupils, and low blood pressure; the patient also is showing signs of central nervous system (CNS) depression. The nurse should prepare to give naloxone and should watch the patient closely for respiratory collapse. Patients with opioid dependence show withdrawal symptoms when the drug is discontinued. When postoperative patients have adequate analgesia without serious side effects, encouraging patients to turn, cough, and breathe deeply is appropriate. This patient is probably relatively pain free, but providing emergency treatment is the priority.DIF: Cognitive Level: ApplicationREF: pp. 207TOP: Nursing Process: Implementation MSC: NCLEX Client Needs Category: Physiologic Integrity: Reduction of Risk Potential 147. A patient asks the nurse what can be given to alleviate severe, chronic pain of several months’ duration. The patient has been taking oxycodone [OxyContin] and states that it is no longer effective. The nurse will suggest discussing which medication with the provider? a. Fentanyl [Duragesic] transdermal patch b. Hydrocodone [Vicodin] PO c. Meperidine [Demerol] PO d. Pentazocine [Talwin] PO ANS: A Transdermal fentanyl is indicated only for persistent, severe pain in patients already opioid tolerant. Hydrocodone, a combination product, has actions similar to codeine and is not used for severe, chronic pain. Meperidine is not recommended for continued use because of the risk of harm caused by the accumulation of a toxic metabolite. Pentazocine is an agonist-antagonist opioid and is less effective for pain; moreover, when given to a patient who is already opioid tolerant, it can precipitate an acute withdrawal syndrome.DIF: Cognitive Level: ApplicationREF: pp. 208TOP: Nursing Process: Planning MSC: NCLEX Client Needs Category: Physiologic Integrity: Pharmacologic and Parenteral Therapies 148. Which side effects of opioid analgesics can have therapeutic benefits? Select all that apply. a. Biliary colic b. Cough suppression c. Suppression of bowel motility d. Urinary retention e. Vasodilation ANS: B , C , E Individual effects of morphine may be beneficial, detrimental, or both. Cough suppression is usually beneficial; suppression of bowel motility and vasodilation can be either beneficial or detrimental. Biliary colic and urinary retention are always detrimental side effects.DIF: Cognitive Level: AnalysisREF: pp. 204TOP: Nursing Process: Assessment MSC: NCLEX Client Needs Category: Physiologic Integrity: Pharmacologic and Parenteral Therapies 149. A patient with cancer has been taking an opioid analgesic four times daily for severalmonths and reports needing increased doses for pain. What will the nurse tell the patient? a. PRN dosing of the drug may be more effective. b. The risk of respiratory depression increases over time. c. The patient should discuss increasing the dose with the provider. d. The patient should request the addition of a benzodiazepine to augment pain relief. ANS: C This patient is developing tolerance, which occurs over time and is evidenced by the need for a larger dose to produce the effect formerly produced by a smaller dose. This patient should be encouraged to request an increased dose. PRN dosing is less effective than scheduled, around-the- clock dosing. The risk ofrespiratory depression decreases over time as patients develop tolerance to this effect. Benzodiazepines are CNS depressants and should not be given with opioids, because they increase the risk of oversedation.DIF: Cognitive Level: ApplicationREF: pp. 206TOP: Nursing Process: Planning MSC: NCLEX Client Needs Category: Physiologic Integrity: Reduction of Risk Potential 150. A patient will receive buprenorphine [Butrans] as a transdermal patch for pain. What is important to teach this patient about the use of this drug? a. Avoid prolonged exposure to the sun. b. Cleanse the site with soap or alcohol. c. Remove the patch daily at bedtime. d. Remove hair by shaving before applying the patch. ANS: A Patients using the buprenorphine transdermal patch should be cautioned against heat, heating pads, hot baths, saunas, and prolonged sun exposure. The skin should be cleaned with water only. The patch should stay on for 7 days before a new patch is applied. Patients should remove hair by clipping, not shaving.DIF: Cognitive Level: ApplicationREF: pp. 214TOP: Nursing Process: Implementation MSC: NCLEX Client Needs Category: Physiologic Integrity: Reduction of Risk Potential 151. A patient with moderate to severe chronic pain has been taking oxycodone [OxyContin] 60 mg every 6 hours PRN for several months and tells the nurse that the medication is not as effective as before. The patient asks if something stronger can be taken. The nurse will contact the provider to discuss: a. administering a combination opioid analgesic/acetaminophen preparation. b. changing the medication to a continued-release preparation. c. confronting the patient about drug-seeking behaviors. d. withdrawing the medication, because physical dependence has occurred. ANS: B Oxycodone is useful for moderate to severe pain, and a continued-release preparation may give more continuous relief. Dosing is every 12 hours, not PRN. A combination product is not recommended with increasing pain, because the nonopioid portion of the medication cannot be increased indefinitely. This patient does not demonstrate drug-seeking behaviors. Physical dependence is not an indication for withdrawing an opioid, as long as it is still needed; it indicates a need for withdrawing the drug slowly when the drug is discontinued.DIF: Cognitive Level: ApplicationREF: pp. 212TOP: Nursing Process: Evaluation MSC: NCLEX Client Needs Category: Physiologic Integrity: Pharmacologic and Parenteral Therapies 152. A woman in labor receives meperidine [Demerol] for pain. The nurse caring for the infant will observe the infant closely for: a. congenital anomalies. b. excessive crying and sneezing. c. respiratory depression. d. tremors and hyperreflexia. ANS: C Use of morphine or other opioids during delivery can cause respiratory depression in the neonate, because the drug crosses the placenta. Infants should be monitored for respiratory depression and receive naloxone if needed. Opioids given during delivery do not contribute to birth defects in the newborn. Excessive crying and sneezing and tremors and hyperreflexia are signs of neonatal opioid dependence, which occurs with long-term opioid use by the mother during pregnancy and not with short-term use of these drugs during labor.DIF: Cognitive Level: ApplicationREF: pp. 207TOP: Nursing Process: Evaluation MSC: NCLEX Client Needs Category: Physiologic Integrity: Reduction of Risk Potential 153. A patient has been taking methadone [Dolophine] for 5 months to overcome an opioid addiction. The nurse should monitor the patient for which of the following electrocardiographic changes? a. Prolonged QT interval b. Prolonged P-R interval c. AV block d. An elevated QRS complex ANS: A Methadone prolongs the QT interval. It does not prolong the P-R interval, cause AV block, or produce an elevated QRS complex.DIF: Cognitive Level: ApplicationREF: p. 211TOP: Nursing Process: Assessment MSC: NCLEX Client Needs Category: Physiologic Integrity: Pharmacologic and Parenteral Therapies 154. A patient who has developed opioid tolerance will experience which effect? a. Decreased analgesic effect b. Decreased constipation c. Increased euphoria d. Increased respiratory depression ANS: A Patients who develop tolerance to opioids will develop tolerance to its analgesic, euphoric, and sedative effects and will also develop tolerance to respiratory depression. Very little tolerance develops to constipation.DIF: Cognitive Level: ApplicationREF: p. 206TOP: Nursing Process: Assessment MSC: NCLEX Client Needs Category: Physiologic Integrity: Pharmacologic and Parenteral Therapies 155. A nurse is preparing a pediatric patient for surgery and is teaching the patient and the child’s parents about the use of the patient-controlled analgesia pump. The parents voice concern about their child receiving an overdose of morphine. What will the nurse do? a. Instruct the parents not to activate the device when their child is sleeping. b. Reassure the parents that drug overdose is not possible with PCA. c. Suggest that the child use the PCA sparingly. d. Tell the patient that the pump can be programmed for PRN dosing only. ANS: A The nurse should instruct parents not to activate the PCA when their child is sleeping because that can lead to drug overdose. Postoperative pain should be treated appropriately with medications that are effective. Nonopioid medications are not sufficient to treat postoperative pain. Patients should be encouraged to use the PCA as needed so that pain can be controlled in a timely fashion. PRN dosing is not as effective as dosing that is continuous, so a basal dose should be given as well as a PRN dose.DIF: Cognitive Level: ApplicationREF: p. 206TOP: Nursing Process: Planning MSC: NCLEX Client Needs Category: Physiologic Integrity: Reduction of Risk Potential 156. A patient is brought to the emergency department by friends, who report finding the patient difficult to awaken. The friends report removing two fentanyl transdermal patches from the patient’s arm. On admission to the emergency department, the patient has pinpoint pupils and a respiratory rate of 6 breaths/minute. A few minutes after administration of naloxone, the respiratory rate is 8 breaths/minute and the patient’s pupils are dilated. The nurse recognizes these symptoms as signs of: a. a mild opioid overdose. b. decreased opioid drug levels. c. improved ventilation. d. worsening hypoxia. ANS: D The classic triad of symptoms of opioid overdose are coma, respiratory depression, and pinpoint pupils. The pupils may dilate as hypoxia worsens, and this symptom, along with continued respiratory depression (fewer than 12 breaths/minute), indicates worsening hypoxia. Fentanyl is a strong opioid, so this is not likely to be a mild overdose, because the patient was wearing two patches. Fentanyl continues to be absorbed even after the patches are removed because of residual drug in the skin, so the drug levels are not likely to be decreasing. The patient does not have improved ventilation, because the respiratory rate is still fewer than 12 breaths/minute.DIF: Cognitive Level: ApplicationREF: pp. 207TOP: Nursing Process: Evaluation MSC: NCLEX Client Needs Category: Physiologic Integrity: Reduction of Risk Potential 157. A patient with chronic pain has been receiving morphine sulfate but now has decreased pain. The prescriber changes the medication to pentazocine [Talwin]. The nurse will monitor the patient for: a. euphoria. b. hypotension. c. respiratorydepression. d. yawning and sweating. ANS: D Pentazocine is an agonist-antagonist opioid, and when given to a patient who is physically dependent on morphine, it can precipitate withdrawal. Yawning and sweating are early signs of opioid withdrawal. Pentazocine does not produce euphoria, hypotension, or respiratory depression.DIF: Cognitive Level: ApplicationREF: pp. 206TOP: Nursing Process: Planning MSC: NCLEX Client Needs Category: Physiologic Integrity: Pharmacologic and Parenteral Therapies 158. A nurse is administering morphine sulfate to a postoperative patient. Which are appropriate routine nursing actions when giving this drug? Select all that apply. a. Counting respirations before and after giving the medication b. Encouraging physical activity and offering increased fluids c. Monitoring the patient’s blood pressure closely for hypertension d. Palpating the patient’s lower abdomen every 4 to 6 hours e. Requesting an order for methylnaltrexone [Relistor] to prevent constipation ANS: A , B , D Respiratory depression, constipation, and urinary retention are common adverse effects of opioid analgesics. It is important to count respirations before giving the drug and periodically thereafter to make sure that respiratory depression has not occurred. Increased physical activity, increased fluid intake, and increased fiber help alleviate constipation. It is important to assess the patient’s abdomen and palpate the bladder to make sure that urinary retention has not occurred. Patients taking morphine often experience hypotension, not hypertension. Methylnaltrexone is given as a last resort to treat constipation, because it blocks mu receptors in the intestine.DIF: Cognitive Level: ApplicationREF: pp. 204TOP: Nursing Process: Evaluation MSC: NCLEX Client Needs Category: Physiologic Integrity: Pharmacologic and Parenteral Therapies 159. A patient who has biliary colic reports a pain level of 8 on a 1-to-10 pain scale with 10 being the most severe pain. The patient has an order for ibuprofen as needed for pain. Which action by the nurse is correct? a. Administer the ibuprofen as ordered. b. Contact the provider to discuss nonpharmacologic pain measures. c. Request an order for meperidine [Demerol]. d. Request an order for morphine sulfate. ANS: C Opioids can induce spasm of the common bile duct and can cause biliary colic. For patients with existing biliary colic, morphine may intensify the pain. It is important to treat pain, however, and certain opioids, such as meperidine, which cause less smooth muscle spasm, may be given. Ibuprofen is used for mild-to-moderate pain and is not appropriate for this patient. Nonpharmacologic methods are appropriate when used as adjunctive therapy with an opioid.DIF: Cognitive Level: ApplicationREF: pp. 210TOP: Nursing Process: Implementation MSC: NCLEX Client Needs Category: Physiologic Integrity: Pharmacologic and Parenteral Therapies Rosenthal: Lehne's Pharmacotherapeutics for Advanced Practice Providers, 2nd Ed. Chapter 25: Drugs for Headache Test Bank Multiple Choice 160. A young woman with migraine headaches who has recently begun taking sumatriptan [Imitrex] calls the nurse to report a sensation of chest and arm heaviness. The nurse questions the patient and determines that she feels pressure and not pain. What will the nurse do? a. Ask the patient about any history of hypertension or coronary artery disease. b. Determine whether the patient might be pregnant. c. Reassure the patient that this is a transient, reversible side effect of sumatriptan. d. Tell the patient to stop taking the medication immediately. ANS: C Some patients taking sumatriptan experience unpleasant chest symptoms, usually described as “heavy arms” or “chest pressure.” These symptoms are transient and are not related to heart disease. Patients experiencing angina-like pain when taking sumatriptan, as a result of coronary vasospasm, should be asked about hypertension or coronary artery disease (CAD); they should not take sumatriptan if they have a history of either of these. The symptoms this patient describes are not characteristics of pregnancy. There is no need to stop taking the medication.DIF: Cognitive Level: ApplicationREF: pp. 225TOP: Nursing Process: Assessment MSC: NCLEX Client Needs Category: Physiologic Integrity: Pharmacologic and Parenteral Therapies 161. A patient who has migraine headaches has been using sumatriptan [Imitrex] with goodresults but reports frequent migraine recurrence 24 hours later. Which medication will the nurse expect the provider to order for this patient? a. Aspirin b. Ergotamine [Ergomar] c. Naratriptan [Amerge] d. Zolmitriptan [Zomig] ANS: C Naratriptan has effects that persist longer than other triptans, and the 24-hour recurrence rate may be reduced when taking this formulation. Aspirin has a shorter half-life. Ergotamine and zolmitriptan do not have a long duration.DIF: Cognitive Level: ApplicationREF: pp. 225TOP: Nursing Process: Planning MSC: NCLEX Client Needs Category: Physiologic Integrity: Pharmacologic and Parenteral Therapies 162. A patient arrives in the emergency department complaining of numbness in the extremities. The nurse notes that the patient’s hands and feet are cool and pale. When conducting a health history, the nurse learns that the patient has a history of migraine headaches. The nurse recognizes this patient’s symptoms as: a. ergotamine withdrawal. b. ergotism. c. severe migraine symptoms. d. sumatriptan side effects. ANS: B Ergotism is a serious toxicity caused by acute or chronic overdose of ergotamine. The toxicity results in ischemia, causing the extremities to become cold, pale, and numb. Symptoms associated with ergotamine withdrawal include headache, nausea, vomiting, and restlessness. These are not symptoms of a severe migraine or side effects of sumatriptan.DIF: Cognitive Level: ApplicationREF: pp. 226TOP: Nursing Process: Assessment MSC: NCLEX Client Needs Category: Physiologic Integrity: Pharmacologic and Parenteral Therapies 163. Which drugs are used to prevent migraine headaches? Select all that apply. a. Divalproex [Depakote] b. Amitriptyline [Elavil] c. Timolol d. Ergotamine [Ergomar] e. Acebutolol ANS: A , B , C Preferred drugs for prophylaxis include propranolol, divalproex, and amitriptyline. Timolol is a beta blocker and can be used instead of propranolol. Acebutolol possesses intrinsic sympathomimetic activity and is not effective for migraine prophylaxis. Ergotamine is used for abortive therapy.DIF: Cognitive Level: ComprehensionREF: pp. 227TOP: Nursing Process: Implementation MSC: NCLEX Client Needs Category: Physiologic Integrity: Pharmacologic and Parenteral Therapies 164. Which medications are used to treat menstrually associated migraine (MAM)? Select all that apply. a. Amitriptyline [Elavil] b. Estrogen c. Ergotamine [Ergomar] d. Frovatriptan [Frova] e. Naproxen ANS: B , D , E Menstrual migraines may be treated with estrogen, some perimenstrual triptans, such as frovatriptan, and Naproxyn.DIF: Cognitive Level: ComprehensionREF: pp. 228TOP: Nursing Process: Planning MSC: NCLEX Client Needs Category: Physiologic Integrity: Pharmacologic and Parenteral Therapies 165. A patient who has a history of asthma experiences three or four migraine headaches each month. The patient uses sumatriptan [Imitrex] as an abortive medication and has developed medication-overuse headaches. The patient asks the nurse what can be done to prevent migraines. The nurse will suggest that the patient discuss which preventive medication with the provider? a. Botulinum toxin b. Meperidine [Demerol] c. Timolol d. Topiramate [Topamax] ANS: D Topiramate can be used for migraine prophylaxis, and its benefits appear equal to those ofthe first- line beta blockers. Botulinum toxin can be used for migraine prophylaxis in patients who have 15 or more headaches a month. Meperidine may be used as abortive therapy but has addictive potential. Timolol is a beta blocker; this patient has asthma, and because beta blockers cause bronchoconstriction, these agents are not recommended.DIF: Cognitive Level: ApplicationREF: pp. 228TOP: Nursing Process: Planning MSC: NCLEX Client Needs Category: Physiologic Integrity: Pharmacologic and Parenteral Therapies 166. A prescriber orders sumatriptan [Imitrex] for a patient for a migraine headache. Before administration of this drug, it would be most important for the nurse to assess whether the patient: a. has a family history of migraines. b. has taken acetaminophen in the past 3 hours. c. has taken ergotamine in the past 24 hours. d. is allergic to sulfa compounds. ANS: C Sumatriptan, other triptans, and ergot alkaloids all cause vasoconstriction and should not be combined, or excessive and prolonged vasospasm could result. Sumatriptan should not be used within 24 hours of an ergot derivative and another triptan. A family history is important, but it is not vital assessment data as it relates to this scenario. Acetaminophen has no drug-to-drug interaction with sumatriptan. Sulfa is not a component of sumatriptan and therefore is not relevant.DIF: Cognitive Level: ApplicationREF: pp. 225TOP: Nursing Process: Assessment MSC: NCLEX Client Needs Category: Physiologic Integrity: Reduction of Risk Potential 167. A patient on the unit complains of cluster headaches. A new graduate nurse is asked to differentiate between a migraine headache and cluster headaches. The graduate nurse is correct to state that manifestations and/or risk factors for a patient with cluster headaches include what? Select all that apply. a. Female gender b. Male gender c. Complaints of nausea and vomiting d. Short duration (15 min to 2 hr) e. Auras before the onset of headache pain f. Throbbing, sometimes piercing pain ANS: B , D , F Cluster headaches are more common in males, are short in duration, and present as throbbing and piercing pain. Migraine headaches are more common in females and are manifested by nausea and vomiting and the presence of an aura before the onset of headache pain.DIF: Cognitive Level: ApplicationREF: p. 228TOP: Nursing Process: Assessment MSC: NCLEX Client Needs Category: Physiologic Integrity: Physiologic Adaptation 168. A patient who has migraine headaches is prescribed sumatriptan [Imitrex] 5 mg unit-dose nasal spray. The patient has administered two sprays at 1400, 1600, and 1800 and calls to report little relief from headache pain. What will the nurse instruct the patient to do? a. Administer two sprays at 2000 and call the provider if no relief. b. Continue using two sprays every 2 hours as needed to relieve discomfort. c. Contact the provider to ask about using an ergot alkaloid medication. d. Use three sprays at the next dose to increase the dose. ANS: A Sumatriptan nasal spray may be used every 2 hours with a maximum 24-hour dose of 40 mg. The patient has already used 30 mg, and another two-spraydose will bring the total dose to 40 mg. The patient should be instructed to contact the provider if the next dose is not effective. Ergot alkaloids should not be given within 24 hours of a triptan. Increasing the next dose to 15 mg would exceed the 24-hour dose maximum.DIF: Cognitive Level: AnalysisREF: pp. 224TOP: Nursing Process: Implementation MSC: NCLEX Client Needs Category: Physiologic Integrity: Pharmacologic and Parenteral Therapies 169. A woman with moderate migraine headaches asks a nurse why the provider has ordered metoclopramide [Reglan] as an adjunct to aspirin therapy, because she does not usually experience nausea and vomiting with her migraines. The nurse will tell her that the metoclopramide is used to: a. help induce sleep. b. improve absorption of the aspirin. c. prevent gastric irritation caused by the aspirin. d. prolong the effects of the aspirin. ANS: B Besides reducing nausea and vomiting, metoclopramide also reverses gastric stasis and improves absorption of oral antimigraine drugs. When aspirin-like analgesics are combined with metoclopramide, they may work as well as sumatriptan. Metoclopramide is not used to induce sleep. It does not prevent gastric irritation or prolong the effects of the aspirin.DIF: Cognitive Level: ApplicationREF: pp. 222TOP: Nursing Process: Implementation MSC: NCLEX Client Needs Category: Physiologic Integrity: Pharmacologic and Parenteral Therapies 170. Supplemental oxygen has been shown to help reduce symptoms for which type of headache? a. Cluster b. Menstrual migraine c. Migraine d. Tension-type ANS: A Cluster headaches can be treated with 100% oxygen inhalation. Oxygen therapy is not used to treat other types of headaches.DIF: Cognitive Level: ComprehensionREF: pp. 229TOP: Nursing Process: Planning MSC: NCLEX Client Needs Category: Physiologic Integrity: Pharmacologic and Parenteral Therapies 171. A patient who has occasional migraine headaches tells a nurse that the abortive medication works well, but she would like to do more to prevent the occurrence of these headaches. The nurse will suggest that the patient: a. ask the provider about an adjunct medication, such as prochlorperazine. b. discuss the use of prophylactic medications with the provider. c. keep a headache diary to help determine possible triggers. d. take the abortive medication regularly instead of PRN. ANS: C Keeping a headache diary to try to identify triggers to migraines can be helpful when a patient is trying to prevent them and is the first step in managing headaches. Prochlorperazine is an antiemetic and does not prevent or abort migraine headaches. Prophylactic medications are used when headaches are more frequent. To prevent medication-overuse headache, abortive medications should not be used more than 1 to 2 days at a time.DIF: Cognitive Level: ApplicationREF: pp. 222TOP: Nursing Process: Implementation MSC: NCLEX Client Needs Category: Physiologic Integrity: Pharmacologic and Parenteral Therapies 172. A patient who has recurrent migraine headaches is prescribed sumatriptan [Imitrex]. Which aspect of this patient’s history is of concern when taking this drug? a. Asthma b. Coronary arterydisease c. Diabetes d. Renal disease ANS: B Serotonin receptor agonists can cause vasoconstriction and coronary vasospasm and should not be given to patients with coronary artery disease, current symptoms of angina, or uncontrolled hypertension. There is no contraindication for asthma, diabetes, or renal disease.DIF: Cognitive Level: ApplicationREF: pp. 225TOP: Nursing Process: Assessment MSC: NCLEX Client Needs Category: Physiologic Integrity: Reduction of Risk Potential Rosenthal: Lehne's Pharmacotherapeutics for Advanced Practice Providers, 2nd Ed. Chapter 26: Antipsychotic Agents and Their Use in Schizophrenia Test Bank Multiple Choice 173. A patient with schizophrenia receives a dose of risperidone [Risperdal Consta] IM. The nurse teaching this patient about this medication will make which statement? a. “You will experience therapeutic levels of this drug in 1 to 2 weeks.” b. “You will need injections of this drug every 6 weeks.” c. “You will need to take an oral antipsychotic drug for 3 weeks.” d. “You probably will not have extrapyramidal symptoms with this drug.” ANS: C Risperidone given intramuscularly is a depot preparation used for long-term therapy. Significant release of the drug does not occur until 2 to 3 weeks after injection; therefore, patients must take an oral antipsychotic medication until drug levels are raised. Therapeutic levels are reached 4 to 6 weeks after injection. Patients need injections every 2 weeks. With IM dosing, the incidence of extrapyramidal symptoms is substantial.DIF: Cognitive Level: ApplicationREF: pp. 240TOP: Nursing Process: Planning MSC: NCLEX Client Needs Category: Physiologic Integrity: Pharmacologic and Parenteral Therapies 174. A patient with schizophrenia shows suicidal behaviors, and the provider orders clozapine [Clozaril]. The nurse teaches the family about the medication and its side effects. Which statement by a family member indicates a need for further teaching about this drug? a. “Blood counts are necessary for several weeks after discontinuation of thedrug.” b. “Fever, sore throat, and sores in the mouth should be reported immediately.” c. “If the ANC is less than 3000, the drug will be discontinued permanently.” d. “Use of this drug requires weekly evaluation of blood work.” ANS: C Clozapine can cause agranulocytosis. If the absolute neutrophil count (ANC) drops below 1000/mcL, the drug must be discontinued permanently. Blood counts must be evaluated weekly, and this evaluation should be continued for several weeks after withdrawal of the drug. Fever, sore throat, and mouth ulcers are symptoms of agranulocytosis and should be reported immediately.DIF: Cognitive Level: ComprehensionREF: pp. 240TOP: Nursing Process: Assessment MSC: NCLEX Client Needs Category: Physiologic Integrity: Pharmacologic and Parenteral Therapies 175. A patient in whom drug therapy has failed several times in the past is readmitted to a hospital to begin therapy for schizophrenia. What will the nurse do to help improve adherence? a. Encourage the patient to take responsibility for medication management. b. Teach the patient about drug side effects and how to manage them. c. Tell the patient that an abstinence syndrome will occur if the drug is stopped. d. Tell the patient that the drug may be taken as needed to control symptoms. ANS: B One way to promote adherence to a medication regimen is to teach patients about drug side effects and how to minimize undesired responses. Family members should be encouraged to oversee medication management for outpatients, because patients themselves may fail to appreciate the need for therapy or may be unwilling to take prescribed medications. It is not true that an abstinence syndrome occurs when these drugs are withdrawn. These drugs are not used PRN; they must be given on a regular basis.DIF: Cognitive Level: ApplicationREF: pp. 245TOP: Nursing Process: Implementation MSC: NCLEX Client Needs Category: Physiologic Integrity: Pharmacologic and Parenteral Therapies 176. A nurse provides teaching for a patient about to begin taking an FGA drug for schizophrenia. Which statement by the patient indicates a need for further teaching about side effects of these drugs? a. “Dry mouth and constipation are uncommon with this medication.” b. “I may experience gynecomastia and galactorrhea.” c. “I may feel lightheaded or dizzy and should sit or lie down if this occurs.” d. “Sedation may occur initially, but will subside in 1 to 2 weeks.” ANS: A Anticholinergic effects are common with FGAs, so this statement indicates a need for further teaching. Neuroendocrine effects, orthostatic hypertension, and sedation can occur with FGAs.DIF: Cognitive Level: ApplicationREF: pp. 236TOP: Nursing Process: Implementation MSC: NCLEX Client Needs Category: Physiologic Integrity: Pharmacologic and Parenteral Therapies 177. A patient taking an FGA medication develops severe parkinsonism and is treated with amantadine [Symmetrel]. The amantadine is withdrawn 2 months later, and the parkinsonism returns. The nurse will expect the provider to: a. give anticholinergic medications. b. make a diagnosis of idiopathic parkinsonism. c. resume the amantadine indefinitely. d. try a second-generation antipsychotic (SGA). ANS: D Neuroleptic-induced parkinsonism is treated with some of the same drugs used for idiopathic parkinsonism, such as amantadine. If parkinsonism is severe, switching to an SGA may help, because the risk of parkinsonism is much lower with these drugs. An anticholinergic medication may be used initially. A recurrence of parkinsonism when the drug is withdrawn doesnot indicate idiopathic parkinsonism. These drugs should not be used indefinitely.DIF: Cognitive Level: AnalysisREF: pp. 235TOP: Nursing Process: Evaluation MSC: NCLEX Client Needs Category: Physiologic Integrity: Pharmacologic and Parenteral Therapies 178. What are negative symptoms of schizophrenia? Select all that apply. a. Delusions b. Disordered thinking c. Poor judgment d. Poor self-care e. Poverty ofspeech ANS: C , D , E Poor judgment, poor self-care, and poverty of speech are all negative symptoms of schizophrenia. Delusions and disordered thinking are positive symptoms.DIF: Cognitive Level: ComprehensionREF: pp. 231TOP: Nursing Process: Assessment MSC: NCLEX Client Needs Category: Physiologic Integrity: Pharmacologic and Parenteral Therapies 179. A patient is taking an FGA for schizophrenia. The nurse notes that the patient has trouble speaking and chewing and observes slow, wormlike-movements of the patient’s tongue. The nurse recognizes which adverse effect in this patient? a. Acute dystonia b. Akathisia c. Parkinsonism d. Tardive dyskinesia ANS: D Tardive dyskinesia can occur in patients during long-term therapy with FGAs. This patient shows signs of this adverse effect. Acute dystonia is characterized by severe spasm of muscles in the face, tongue, neck, or back, and by opisthotonus. Akathisia is characterized by constant motion. Parkinsonism is characterized by bradykinesia, drooling, tremor, rigidity, and a shuffling gait.DIF: Cognitive Level: ComprehensionREF: pp. 236TOP: Nursing Process: Assessment MSC: NCLEX Client Needs Category: Physiologic Integrity: Pharmacologic and Parenteral Therapies 180. A nurse and a nursing student are discussing the plan of care for a patient with schizophrenia. The patient, who has been taking a high-potency FGA for 2 months, has become restless and constantly needs to be in motion. Which statement by the student indicatesa need for further education? a. “Anticholinergic medications may help control these symptoms.” b. “Because this may be an exacerbation of psychosis, the provider may increase the dose of the FGA.” c. “The provider may try a low-potency FGA instead of the high-potencyFGA.” d. “This patient may need to take a benzodiazepine or a beta blocker.” ANS: B The patient is showing signs of akathisia, which can resemble an exacerbation of psychosis. If the two are confused and the provider orders more of the FGA, the symptoms may actually increase. Anticholinergic medications may be used, a low-potency FGA may be ordered, or a benzodiazepine or beta blocker may be prescribed.DIF: Cognitive Level: AnalysisREF: pp. 236TOP: Nursing Process: Evaluation MSC: NCLEX Client Needs Category: Physiologic Integrity: Pharmacologic and Parenteral Therapies 181. A patient who is taking a first-generation antipsychotic (FGA) drug for schizophrenia comes to the clinic for evaluation. The nurse observes that the patient has a shuffling gait and mild tremors. The nurse will ask the patient’s provider about which course of action? a. Administering a direct dopamine antagonist b. Giving an anticholinergic medication c. Increasing the dose of the antipsychotic drug d. Switching to a second-generation antipsychotic drug ANS: B The patient is showing signs of parkinsonism, an extrapyramidal effect associated with antipsychotic medications. Anticholinergic medications are indicated. A direct dopamine antagonist would counter the effects of the antipsychotic and remove any beneficial effect it has. Increasing the dose of the antipsychotic medication would only worsen the extrapyramidal symptoms. A second-generation antipsychotic medication may be used if parkinsonism is severe, since the risk of parkinsonism is lower than with the FGA. This patient is exhibiting mild symptoms, so this is not necessary at this point.DIF: Cognitive Level: AnalysisREF: pp. 235TOP: Nursing Process: Implementation MSC: NCLEX Client Needs Category: Physiologic Integrity: Reduction of Risk Potential 182. A patient with schizophrenia has been taking an oral FGA for 1 week. The patient has been taking the drug daily in two divided doses. The individual complains of daytime drowsiness. The patient’s family reports a decrease in the person’s hostility and anxiety but states that the patient remains antisocial with disordered thinking. What will the nurse tell the patient and the family? a. An increased dose of the drug may be needed. b. Intramuscular dosing may be needed. c. Some symptoms take months to improve. d. The entire dose may be taken at bedtime. ANS: C When patients begin therapy with antipsychotic medications, some symptoms resolve sooner than others. During the first week, agitation, hostility, anxiety, and tension may resolve, but other symptoms may take several months to improve. It is not necessary to increase the dose in the first week. IM dosing is indicated for patients with severe, acute schizophrenia and for long-term maintenance. Sedation is normal, and once an effective dose has been determined, the entire dose can be taken at bedtime, but not in the initial days of therapy.DIF: Cognitive Level: ApplicationREF: pp. 245TOP: Nursing Process: Evaluation MSC: NCLEX Client Needs Category: Physiologic Integrity: Pharmacologic and Parenteral Therapies 183. A patient with schizophrenia has been taking an antipsychotic drug for several days. The nurse enters the patient’s room to administer a dose of haloperidol [Haldol] and finds the patient having facial spasms. The patient’s head is thrust back, and the patient is unable to speak. What will the nurse do? a. Administer the haloperidol as ordered. b. Discuss increasing the haloperidol dose with the provider. c. Request an order to give diphenhydramine. d. Request an order to give levodopa. ANS: C An early reaction to antipsychotic drugs is acute dystonia. Initial treatment consists of an anticholinergic medication, such as diphenhydramine. Administering more antipsychotic medication would increase the symptoms and could be life threatening. Levodopa is not given for extrapyramidal symptoms, because it could counteract the beneficial effects of antipsychotic treatment.DIF: Cognitive Level: AnalysisREF: pp. 235TOP: Nursing Process: Implementation MSC: NCLEX Client Needs Category: Physiologic Integrity: Reduction of Risk Potential 184. A parent reports being afraid that a child may have schizophrenia because of disorganized speech and asocial behaviors. The nurse will tell this parent that which of the following must also be present to make a diagnosis? Select all that apply. a. A decrease in self-care, job, or school function b. A history of substance abuse c. A 1-month duration of active phase symptoms d. Continuous signs of disturbance for longer than 6 months e. The presence of manic episodes ANS: A , C , D Patients must have at least two symptoms with 1-month duration of active symptoms. One symptom must be delusions, hallucinations, or disordered speech. Patients must have continuous signs of disturbance for longer than 6 months. A history of substance abuse and manic episodes are not associated with schizophrenia.DIF: Cognitive Level: ApplicationREF: pp. 231TOP: Nursing Process: Assessment MSC: NCLEX Client Needs Category: Physiologic Integrity: Pharmacologic and Parenteral Therapies 185. Which side effects are more common in second-generation antipsychotic medications than in first-generation antipsychotic medications? Select all that apply. a. Agranulocytosis b. Anticholinergic effects c. Extrapyramidal symptoms d. Metabolism by CYP3A4 e. Prolactin elevation ANS: A , B , D SGAs are more likely than FGAs to cause agranulocytosis and anticholinergic effects and are metabolized by CYP3A4 enzymes. They are not more likely to cause extrapyramidal effects or prolactin elevation.DIF: Cognitive Level: ApplicationREF: pp. 231TOP: Nursing Process: Assessment MSC: NCLEX Client Needs Category: Physiologic Integrity: Pharmacologic and Parenteral Therapies 186. A nurse in a mental health hospital finds a patient with schizophrenia who takes haloperidol [Haldol] lying rigid in bed with a temperature of 41.3°C. A cardiac monitor shows cardiac dysrhythmias. What will be included in the treatment of this patient? Select all that apply. a. Anticholinergic medications b. Beta blockers c. Dantrolene d. Intravenous fluids e. Withdrawal of haloperidol ANS: C , D , E Neuroleptic malignant syndrome is characterized by “lead pipe” rigidity, sudden high fever, and autonomic instability. Treatment requires supportive measures, drug therapy, and immediate withdrawal of the antipsychotic medication. Dantrolene is used to relax muscles and reduce heat production. Intravenous fluids are used to maintain hydration. Anticholinergic medications and beta blockers are not helpful.DIF: Cognitive Level: ApplicationREF: pp. 236TOP: Nursing Process: Assessment MSC: NCLEX Client Needs Category: Physiologic Integrity: Pharmacologic and Parenteral Therapies 187. A patient who has diabetes mellitus is diagnosed with schizophrenia and the provider orders thioridazine. The patient asks the nurse why the provider has not ordered olanzapine [Zyprexa], which the patient has seen advertised on television. Which response by the nurse is the most important reason that this patient is not receiving olanzapine? a. “Olanzapine is more expensive than thioridazine.” b. “Olanzapine causes more metabolic side effects than thioridazine.” c. “Thioridazine has fewer side effects than olanzapine.” d. “Thioridazine has a faster onset of action than olanzapine.” ANS: B Olanzapine is an SGA and, although it has fewer extrapyramidal side effects than the FGA the provider has ordered, it has an increased risk of metabolic side effects, which is contraindicated in patients with diabetes. It is more expensive, but this is not the most important reason for not prescribing it. Thioridazine has more side effects than olanzapine, but the side effects caused by olanzapine are more critical for this patient. Thioridazine does not have a faster onset of action.DIF: Cognitive Level: AnalysisREF: pp. 240TOP: Nursing Process: Implementation MSC: NCLEX Client Needs Category: Physiologic Integrity: Pharmacologic and Parenteral Therapies Rosenthal: Lehne's Pharmacotherapeutics for Advanced Practice Providers, 2nd Ed. Chapter 27: Antidepressants Test Bank Multiple Choice 188. A patient who has been taking a monoamine oxidase inhibitor (MAOI) for depression for several months tells the provider that the medication has not helped with symptoms. The provider plans to switch the patient to an SSRI. The nurse will teach this patient to: a. start taking the SSRI and stop the MAOI when symptoms improve. b. start taking the SSRI and then gradually withdraw the MAOI. c. stop taking the MAOI and wait 5 weeks before starting the SSRI. d. stop taking the MAOI 2 weeks before starting the SSRI. ANS: D MAOIs increase 5-HT availability, thus greatly increasing the risk of serotonin syndrome. MAOIs should be withdrawn at least 14 days before an SSRI is started. An SSRI should never be given at the same time as an MAOI. It is not necessary to wait 5 weeks before starting an SSRI.DIF: Cognitive Level: ApplicationREF: pp. 252TOP: Nursing Process: Planning MSC: NCLEX Client Needs Category: Physiologic Integrity: Reduction of Risk Potential 189. An older adult patient who is to begin taking imipramine [Tofranil] asks the nurse when the drug should be taken. The nurse will instruct the patient to: a. divide the daily dose into two equal doses 12 hours apart. b. take the entire dose at bedtime to minimize the sedative effects. c. take the medication once daily in the late afternoon. d. take the medication once daily in the morning. ANS: A For many patients, taking the entire dose of a TCA at bedtime is advantageous for facilitating adherence, minimizing daytime sedation, and promoting sleep. However, older adult patients are at greater risk for cardiotoxicity and may experience intolerable effects on the heart if the entire dose is taken at once; therefore, twice-daily dosing is recommended in the elderly.DIF: Cognitive Level: ApplicationREF: pp. 258TOP: Nursing Process: Planning MSC: NCLEX Client Needs Category: Physiologic Integrity: Pharmacologic and Parenteral Therapies 190. A young adult patient has been taking an antidepressant medication for several weeks and reports having increased thoughts of suicide. The nurse questions further and learns that the patient has attempted suicide more than once in the past. The patient identifies a concreteplan for committing suicide. The nurse will contact the provider to discuss: a. changing the medication to another drug class. b. discontinuing the medication immediately. c. hospitalizing the patient for closer monitoring. d. requiring more frequent clinic visits for this patient. ANS: C Patients with depression often think of suicide, and during treatment with antidepressants, these thoughts often increase for a time. Patients whose risk of suicide is especially high should be hospitalized. All antidepressants carry this risk, so changing medication is not recommended. Discontinuing the medication is not recommended. More frequent clinic visits are recommended for patients with a low to moderate risk of suicide.DIF: Cognitive Level: AnalysisREF: pp. 248TOP: Nursing Process: Evaluation MSC: NCLEX Client Needs Category: Physiologic Integrity: Reduction of Risk Potential 191. A patient has been taking an SSRI antidepressant for major depression and reports having headaches and jaw pain. What will the nurse tell the patient? a. This represents an irreversible extrapyramidal side effect. b. Discuss discontinuing the antidepressant with the provider. c. Discuss these symptoms with a dentist. d. Try stress-relieving methods and relaxation techniques. ANS: C Bruxism is a side effect of SSRIs and can result in headache and jaw pain. Patients who experience these signs should be evaluated for bruxism by a dentist, who can determine whether the patient may benefit from the use of a mouth guard. Headache and jaw pain are not signs of extrapyramidal side effects. Discontinuing the antidepressant is not indicated, because depression may return. Stress-relieving methods and relaxation techniques are not recommended, because these symptoms occur during sleep.DIF: Cognitive Level: ApplicationREF: pp. 252TOP: Nursing Process: Evaluation MSC: NCLEX Client Needs Category: Physiologic Integrity: Pharmacologic and Parenteral Therapies 192. A neonate is born to a patient who reports taking venlafaxine [Effexor XR]. The nurse caring for the infant will observe the infant for: a. irritability, tremor, and respiratory distress. b. poor appetite and disturbed sleeping patterns. c. serotonin syndrome. d. sustained mydriasis. ANS: A Use of venlafaxine late in pregnancy can result in a neonatal withdrawal syndrome characterized by irritability, abnormal crying, tremor, respiratory distress, and possibly seizures. Poor appetite and disturbed sleep are not part of this withdrawal syndrome. Serotonin syndrome is not likely. Sustained mydriasis occurs as an adverse effect in patients taking the drug.DIF: Cognitive Level: AnalysisREF: pp. 254TOP: Nursing Process: Assessment MSC: NCLEX Client Needs Category: Physiologic Integrity: Reduction of Risk Potential 193. A patient is brought to the emergency department after taking a handful of TCA pills. The nurse will expect to provide what when caring for this patient? Select all that apply. a. Cardiac monitoring b. Cholinesterase inhibitors c. Gastric lavage and activated charcoal d. Sedative medications e. Procainamide ANS: A , B , C Patients who overdose with a TCA should have cardiac monitoring, because cardiac side effects can occur. Cholinesterase inhibitors are given to counteract anticholinergic side effects. Gastric lavage followed by activated charcoal can reduce absorption of the TCA. Sedative drugs would only increase the sedative effects of the TCA. Procainamide causes cardiac depression and is not recommended to treat TCA dysrhythmias.DIF: Cognitive Level: ApplicationREF: pp. 257TOP: Nursing Process: Implementation MSC: NCLEX Client Needs Category: Physiologic Integrity: Pharmacologic and Parenteral Therapies 194. A patient is diagnosed with major depression with severe symptoms and begins taking an antidepressant medication. Three weeks after beginning therapy, the patient tells the nurse that the drug is not working. The nurse will counsel this patient to ask the provider about: a. adding a second medication to complement this drug. b. changing the medication to one in a different drug class. c. increasing the dose of this medication. d. using nondrug therapies to augment the medication. ANS: D Patients with severe depression benefit more from a combination of drug therapy and psychotherapy than from either component alone, so this patient should ask the provider about nondrug therapies. Once a drug has been selected for treatment, it must be used for 4 to 8 weeks before its efficacy can be assessed. Until a drug has been used at least 1 month without success, it should not be considered a failure. Adding a second medication, changing to a different medication, and increasing the dose of this medication should all be reserved until the current drug is deemed to have failed after at least 4 weeks.DIF: Cognitive Level: AnalysisREF: pp. 247TOP: Nursing Process: Evaluation MSC: NCLEX Client Needs Category: Physiologic Integrity: Pharmacologic and Parenteral Therapies 195. Which patients are candidates for MAOIs? Select all that apply. a. Patients who have not responded to SSRIs and TCAs b. Patients with atypical depression c. Patients with bulimia nervosa d. Patients with hypotension e. Patients with postpartumdepression ANS: A , B , C Patients who have not responded to SSRIs or TCAs, patients with atypical depression, and patients with bulimia nervosa are candidates for MAOIs. MAOIs contribute to hypotension and therefore are contraindicated in patients with hypotension. MAOIs are not recommended for the treatment of postpartum depression.DIF: Cognitive Level: ComprehensionREF: pp. 258TOP: Nursing Process: Implementation MSC: NCLEX Client Needs Category: Physiologic Integrity: Pharmacologic and Parenteral Therapies 196. A patient whose spouse has died recently reports feeling down most of every day for the past 2 months. On further questioning, the nurse learns that the patient has quit participating in church and social activities, has difficulty falling asleep, and has lost 5 pounds. The patient reports feeling tired and confused all the time but does not have suicidal thoughts. What does the nurse suspect? a. Grief and sadness b. Hypomania c. Major depression d. Situational depression ANS: C This patient has symptoms of major depression, which include depressed mood, loss of pleasure in usual activities, insomnia, weight loss, and feelings of fatigue. For a diagnosis of major depression, these symptoms must be present most of the day, nearly every day, for at least 2 weeks. Grief and sadness and situational depression are common responses to the death of a loved one, but this patient’s symptoms go beyond this normal response. This patient does not show signs of hypomania.DIF: Cognitive Level: ApplicationREF: pp. 247TOP: Nursing Process: Assessment MSC: NCLEX Client Needs Category: Psychosocial Integrity: Grief and Loss 197. A patient taking an MAOI is seen in the clinic with a blood pressure of 170/96 mm Hg. What will the nurse ask this patient? a. Whether any antihypertensive medications are used b. Whether the patient drinks grapefruit juice c. To list all foods eaten that day d. Whether SSRIs are taken in addition to the MAOI ANS: C Patients taking an MAOI should be counseled to follow strict dietary restrictions and to avoid all foods containing tyramine. Patients who consume such foods when taking an MAOI experience a hypertensive episode. Antihypertensive medications, given with an MAOI, will result in hypotension. Grapefruit juice does not alter the metabolism of an MAOI. SSRIs and MAOIs, when administered together, cause a serotonin syndrome.DIF: Cognitive Level: AnalysisREF: pp. 259TOP: Nursing Process: Evaluation MSC: NCLEX Client Needs Category: Physiologic Integrity: Pharmacologic and Parenteral Therapies 198. A patient taking fluoxetine [Prozac] complains of decreased sexual interest. Aprescriber orders a “drug holiday.” What teaching by the nurse would best describe a drugholiday? a. “Cut the tablet in half anytime to reduce the dosage.” b. “Discontinue the drug for 1 week.” c. “Don’t take the medication on Friday and Saturday.” d. “Take the drug every other day.” ANS: C Sexual dysfunction may be managed by having the patient take a drug holiday, which involves discontinuing medication on Fridays and Saturdays. Cutting the tablet in half anytime to reduce the dosage is an inappropriate way to manage drug administration effectively. In addition, it does not describe a drug holiday. The patient should not take the drug every other day, nor should it be discontinued for a week at a time, because this would diminish the therapeutic levels of the drug, thereby minimizing the therapeutic effects. In addition, neither of those options describe a drug holiday.DIF: Cognitive Level: ApplicationREF: pp. 251TOP: Nursing Process: Implementation MSC: NCLEX Client Needs Category: Physiologic Integrity: Pharmacologic and Parenteral Therapies Rosenthal: Lehne's Pharmacotherapeutics for Advanced Practice Providers, 2nd Ed. Chapter 28: Drugs for Bipolar Disorder Test Bank Multiple Choice 199. A patient with bipolar disorder who wants to minimize the need for drug therapy asks the nurse what else can be done to treat the disorder. The nurse will recommend which measures? Select all that apply. a. Electroconvulsive therapy b. Moderate use of alcohol to reduce stress c. Psychotherapy d. Regular sleep and exercise e. Using a chart to monitor mood changes ANS: C , D , E BPD should be treated with a combination of drugs and adjunctive psychotherapy, because drug therapy alone is not optimal. Other measures, such as regular sleep and exercise and recognizing early symptoms of mood change, help minimize extreme mood swings. Electroconvulsive therapy is effective, but it is not the first-choice treatment; it is reserved for patients who have not responded to other therapies. Avoidance of alcohol is recommended.DIF: Cognitive Level: ApplicationREF: pp. 271TOP: Nursing Process: Planning MSC: NCLEX Client Needs Category: Physiologic Integrity: Pharmacologic and Parenteral Therapies 200. A patient with bipolar disorder who is taking divalproex sodium [Valproate] has just been admitted to the hospital. During the admission assessment, the patient tells the nurse about recent suicidal ideation. The nurse observes several areas of bruising over soft tissue areas and notes a weight gain of 10 pounds since the last admission 1 year ago. What will the nurse do? a. Ask the patient whether the bruises are self-inflicted. b. Contact the provider to report these findings. c. Give the patient information about weight loss. d. Request an order for an increased dose to help with depressive symptoms. ANS: B Divalproex sodium is used to control symptoms during manic episodes and can prevent relapse into mania. It is less effective than lithium at reducing the risk of suicide. It can cause thrombocytopenia, which results in bruising and is an indication for immediate drug withdrawal. Weight gain can be serious and chronic. All of these findings are an indication for withdrawing the drug and should be reported to the provider. Untilplatelet levels determine whether the bruises are drug induced, it is not appropriate to ask the patient if they are self-inflicted. Because weight gain is common and can be severe with this drug, information about weight loss is not likely to have an effect. Divalproex sodium is better than lithium at treating depression; however, in light of the other symptoms, it is probably not the best choice.DIF: Cognitive Level: ApplicationREF: pp. 274TOP: Nursing Process: Evaluation MSC: NCLEX Client Needs Category: Physiologic Integrity: Reduction of Risk Potential 201. A patient with bipolar disorder takes lamotrigine [Lamictal]. Which statement by the patient would prompt the nurse to hold the drug and notify the prescriber for further assessment? a. “I get a little dizzy sometimes.” b. “I had a headache last week that lasted for about an hour.” c. “I’ve broken out in a rash on my chest and back.” d. “Last night I woke up twice with a bad dream.” ANS: C Evidence of a rash in a patient taking lamotrigine requires further assessment, because this may indicate the development of Stevens-Johnson syndrome. Although dizziness and headaches are side effects of lamotrigine, they are not potentially life threatening. A bad dream is not necessarily related to the lamotrigine.DIF: Cognitive Level: ApplicationREF: pp. 275TOP: Nursing Process: Evaluation MSC: NCLEX Client Needs Category: Physiologic Integrity: Reduction of Risk Potential 202. A patient who has recently begun taking carbamazepine [Equetro] for bipolar disorder reports having vertigo and headaches. Which action by the nurse is appropriate? a. Ask the provider whether another medication can be used for this patient, because the patient is showing signs of toxicity. b. Contact the provider to request a complete blood count (CBC) to evaluate for other, more serious side effects. c. Reassure the patient that these effects occur early in treatment and will resolve over time. d. Review the patient’s chart for cytochrome P450 enzymes to see whether an increased dose is needed. ANS: C Carbamazepine can cause several neurologic side effects early in treatment, including vertigo and headaches. These resolve with continued drug use. These side effects are not related to drug toxicity. A CBC should be obtained at baseline and periodically thereafter. Carbamazepine can cause changes in hematologic laboratory values. The side effects reported by this patient are not associated with hematologic side effects. Carbamazepine induces cytochrome P450 enzymes and can accelerate its own metabolism, which would reduce the amount of drug and decrease side effects, so an increased dose is not appropriate.DIF: Cognitive Level: ApplicationREF: pp. 275TOP: Nursing Process: Evaluation MSC: NCLEX Client Needs Category: Physiologic Integrity: Pharmacologic and Parenteral Therapies 203. A patient is admitted to a hospital for treatment for first-time symptoms of mania and is exhibiting euphoric mania. Which medication will the provider order? a. Lithium [Lithobid] b. Olanzapine [Zyprexa] c. Risperidone [Risperdal] d. Divalproex sodium [Valproate] ANS: A In almost all cases of mania, divalproex sodium is the drug of choice, except for euphoric mania symptoms. Lithium is used to treat euphoric mania. Olanzapine and risperidone are used to treat other symptoms associated with BPD.DIF: Cognitive Level: ApplicationREF: pp. 270TOP: Nursing Process: Implementation MSC: NCLEX Client Needs Category: Physiologic Integrity: Pharmacologic and Parenteral Therapies 204. A patient with bipolar disorder has been taking lithium [Lithobid] for several years. The patient has developed a goiter, and serum tests reveal hypothyroidism. What will the nurse expect the provider to order for this patient? a. Administration of levothyroxine b. Increasing the lithium dose c. Iodine supplements d. Referral to an endocrinologist ANS: A Patients taking lithium may experience reduced incorporation of iodine into the thyroid hormone, resulting in goiter and hypothyroidism. Administration of levothyroxine or withdrawing the lithium will reverse both. Increasing the lithium dose will make this worse. Iodine supplements are not indicated. The provider will either order stopping the lithium or administration of levothyroxine, which will reverse this condition, so referral to an endocrinologist is not necessary.DIF: Cognitive Level: ApplicationREF: pp. 273TOP: Nursing Process: Implementation MSC: NCLEX Client Needs Category: Physiologic Integrity: Pharmacologic and Parenteral Therapies 205. A nurse is preparing to administer medications to a hospitalized patient who has been taking lithium [Lithobid] for 3 days. The patient is complaining of mild nausea and abdominal bloating. The patient’s lithium level is 0.8 mEq/L. What will the nurse do? a. Administer the dose and tell the patient that the side effects are temporary. b. Contact the prescriber to request an order for serum electrolytes. c. Hold the dose and notify the prescriber of the patient’s lithium level. d. Request an order for amiloride [Midamor]. ANS: A This patient is experiencing side effects that are common and that occur at therapeutic levels of the drug. The lithium level is therapeutic and not toxic, so the nurse should give the dose and reassure the patient that the side effects will diminish over time. In the presence of low sodium, lithium can accumulate to toxic doses; therefore, if the lithium level were elevated, evaluating serum electrolytes would be advisable. The dose does not need to be withheld because the patient does not have toxic levels of lithium. Amiloride is used if patients are experiencing lithium- induced polyuria, which this patient does not have.DIF: Cognitive Level: ApplicationREF: pp. 273TOP: Nursing Process: Evaluation MSC: NCLEX Client Needs Category: Physiologic Integrity: Pharmacologic and Parenteral Therapies 206. A patient recently was diagnosed with bipolar disorder. The patient, who has a history of seasonal allergies, is an athlete who participates in track. The nurse is teaching the patient about lithium [Lithobid], which the prescriber has just ordered. Which statement by the patient indicates the need for further teaching? a. “I can continue to use ibuprofen as needed for muscle pain.” b. “I should drink extra fluids before and during exercise.” c. “I should not use antihistamines while taking lithium.” d. “I should report muscle weakness and tremors to my provider.” ANS: A Because nonsteroidal anti-inflammatory drugs (NSAIDs) can increase lithium levels as much as 60%, they should not be used by patients taking lithium. Aspirin does not have this effect. Lithium induces polyuria in 50% to 70% of patients, so patients should be advised to drink extra fluids, especially during exercise. Antihistamines have anticholinergic effects, which cause urinary hesitancy; this can be uncomfortable when patients experience the polyuria associated with lithium use. Muscle weakness and tremors can occur with lithium; tremors can be treated with beta blockers or by altering the lithium regimen.DIF: Cognitive Level: ApplicationREF: pp. 273TOP: Nursing Process: Implementation MSC: NCLEX Client Needs Category: Physiologic Integrity: Reduction of Risk Potential 207. A patient with bipolar disorder is admitted to the hospital. The patient has been taking lithium [Lithobid] for several years and has not been evaluated by a provider for over a year. Besides obtaining a lithium level, the nurse caring for this patient will anticipate orders for which laboratory tests? Select all that apply. a. Calcium level b. Complete blood count with differential c. Liver function tests d. Renal function tests e. Serum potassium f. Thyroid function tests ANS: B , D , F Patients taking lithium can develop a mild, reversible leukocytosis, so annual CBC evaluation with differential is recommended. Chronic lithium use is associated with degenerative changes in the kidneys, so renal function should be assessed annually. Lithium can reduce the incorporation of iodine into thyroid hormone and can inhibit thyroid secretion; therefore, thyroid hormone and thyroid-stimulating hormone (TSH) levels should be measured annually. Lithium is affected by sodium levels but not by calcium or potassium levels. Because lithium is excreted by the kidneys, hepatic function tests are not indicated.DIF: Cognitive Level: ApplicationREF: pp. 273TOP: Nursing Process: Assessment MSC: NCLEX Client Needs Category: Physiologic Integrity: Reduction of Risk Potential 208. The spouse of a patient with bipolar disorder (BPD) tells the nurse that the patient will not stay on the lithium ordered by the provider longer than 1 or 2 months at a time. The nurse understands that adherence to medication regimens in patients with BPD is problematic and will tell the spouse: a. “During manic episodes, many patients don’t see the benefit of prophylactic medications.” b. “Increased gastrointestinal side effects occur over time and reduce compliance.” c. “Long-term use of lithium causes memory impairment, causing patients to forget to take their medications.” d. “Patients who are depressed do not want to take theirmedications.” ANS: A Patients experiencing manic symptoms often do not see anything wrong with their thinking and behavior and therefore do not believe they need treatment. Moreover, these symptoms are often enjoyable, and they do not want them to stop. Gastrointestinal side effects and central nervous system (CNS) effects of memory impairment subside over time and would diminish with long- term treatment. Patients are usually most uncomfortable during depressive episodes, and it is during these episodes that they often seek treatment.DIF: Cognitive Level: ApplicationREF: pp. 271TOP: Nursing Process: Diagnosis MSC: NCLEX Client Needs Category: Physiologic Integrity: Pharmacologic and Parenteral Therapies 209. A patient with bipolar disorder has frequent manic episodes alternating with depressive episodes. The prescriber orders risperidone [Risperdal] in addition to the lithium [Lithobid] that the patient is already taking. The patient asks the nurse why another drug is needed. The nurse will tell the patient that the risperidone is used to: a. elevate mood during depressive episodes. b. help control symptoms during manic episodes. c. manage tremors associated with lithium use. d. prevent recurrence of depressive episodes. ANS: B Risperidone is an antipsychotic often used in conjunction with lithium to help manage symptoms during manic episodes, regardless of whether psychotic symptoms occur. Risperidone does not elevate mood and is not used during depressive episodes. It is not used to counter side effects associated with lithium. It does not prevent recurrence of depressive episodes.DIF: Cognitive Level: ApplicationREF: pp. 275TOP: Nursing Process: Implementation MSC: NCLEX Client Needs Category: Physiologic Integrity: Pharmacologic and Parenteral Therapies Rosenthal: Lehne's Pharmacotherapeutics for Advanced Practice Providers, 2nd Ed. Chapter 29: Sedative-Hypnotic Drugs Test Bank Multiple Choice 210. A patient with a new-onset seizure disorder receives a prescription for phenobarbital. The patient reports being concerned about the sedative side effects of this drug. Which response by the nurse is correct? a. “Phenobarbital doses for seizures are nonsedating.” b. “This is a short-acting barbiturate, so sedation wears off quickly.” c. “Tolerance to the sedative effects will develop in a few weeks.” d. “You may actually experience paradoxical effects of euphoria.” ANS: A Phenobarbital and mephobarbital are used for seizure disorders and suppress seizures at doses that are nonsedative. Phenobarbital is a long-acting barbiturate. At therapeutic doses, sedative effects do not occur. Paradoxical drug effects are associated with benzodiazepines and in older adults and debilitated patients with barbiturates.DIF: Cognitive Level: ApplicationREF: p. 281TOP: Nursing Process: Implementation MSC: NCLEX Client Needs Category: Physiologic Integrity: Pharmacologic and Parenteral Therapies 211. A patient who travels frequently for business reports occasional instances of being unable to fall asleep. The patient tells the nurse that job demands require staying up late and then getting up early for meetings. The nurse expects that the provider will prescribe which medication for this patient? a. Flurazepam b. Trazodone [Desyrel] c. Zaleplon [Sonata] d. Zolpidem [Ambien] ANS: C Zaleplon [Sonata] works well for people who have trouble falling asleep and, because of its short duration of action, can be taken late at night without causing a hangover or next-daysedation early in the morning. Zolpidem [Ambien] has a longer duration and is a good choice for patients who have difficulty maintaining sleep. Flurazepam has a long duration of action. Trazodone causes daytime grogginess.DIF: Cognitive Level: ApplicationREF: p. 279TOP: Nursing Process: Assessment MSC: NCLEX Client Needs Category: Physiologic Integrity: Pharmacologic and Parenteral Therapies 212. A patient who is experiencing alcohol withdrawal is given a benzodiazepine. The nurse understands that this drug is effective because: a. the alcohol does not interact with the benzodiazepine. b. the benzodiazepine potentiates alcohol withdrawal symptoms. c. the benzodiazepine relieves muscle spasms and spasticity. d. the patient has a cross-dependence to the benzodiazepine. ANS: D Benzodiazepines are given to ease withdrawal from alcohol because of cross-dependence with these drugs and alcohol, enabling the benzodiazepine to suppress withdrawal symptoms. Alcohol and benzodiazepines can potentiate one another. The benzodiazepine does not potentiate withdrawal symptoms.DIF: Cognitive Level: AnalysisREF: p. 276TOP: Nursing Process: Assessment MSC: NCLEX Client Needs Category: Physiologic Integrity: Pharmacologic and Parenteral Therapies 213. A patient in the emergency department is given intravenous diazepam [Valium] for seizures. When the seizures stop, the nurse notes that the patient is lethargic and confused and has a respiratory rate of 10 breaths/minute. The nurse will expect to administer which of the following? a. Flumazenil [Romazicon] b. Gastric lavage c. Respiratory support d. Toxicology testing ANS: C When benzodiazepines are administered IV, severe effects, including profound hypotension, respiratory arrest, and cardiac arrest, can occur. Respiration should be monitored, and the airway must be managed if necessary. Flumazenil [Romazicon] is a competitive benzodiazepine receptor antagonist and is used to reverse the sedative effects but may not reverse respiratory depression. Gastric lavage would not be effective, because the benzodiazepine has been given IV. Without further indication of the ingestion of other drugs, toxicology testing is not a priority.DIF: Cognitive Level: ApplicationREF: p. 277TOP: Nursing Process: Implementation MSC: NCLEX Client Needs Category: Physiologic Integrity: Reduction of Risk Potential 214. A patient who has been using secobarbital for several months to treat insomnia tells the nurse that the prescriber has said the prescription will be changed to temazepam [Restoril] because it is safer. The patient asks why this agent is safer. The nurse is correct in telling the patient that temazepam: a. does not depress the central nervous system. b. shows no respiratory depression, even in toxic doses. c. mimics the actions of a central nervous system inhibitory neurotransmitter. d. potentiates endogenous gamma-aminobutyric acid (GABA) producing a finite CNS depression. ANS: D Benzodiazepines potentiate the actions of GABA, and because the amount of GABA in the CNS is finite, these drugs’ depressive effect on the CNS is limited. Benzodiazepines depress the CNS but not to the extent that barbiturates do. Benzodiazepines are weak respiratory depressants at therapeutic doses and moderate respiratorydepressants at toxic doses. Barbiturates mimic GABA; therefore, because they produce CNS depression, this effect is limited only by the amount of barbiturate administered.DIF: Cognitive Level: ApplicationREF: p. 276TOP: Nursing Process: Implementation MSC: NCLEX Client Needs Category: Physiologic Integrity: Pharmacologic and Parenteral Therapies 215. A hospitalized patient who is given one dose of flurazepam continues to show drowsiness the next day. A nursing student asks the nurse the reason for this, because the drug’s half-life is only 2 to 3 hours. Which response by the nurse is correct? a. “Benzodiazepines commonly cause residual effects lasting into the day after the dose is given.” b. “The patient is having a paradoxical reaction to this medication.” c. “This patient must have developed a previous tolerance to benzodiazepines.” d. “When this drug is metabolized, the resulting compound has longer-lasting effects.” ANS: D Flurazepam has a half-life of 2 to 3 hours; however, its metabolite has a long half-life, so giving the drug results in long-lasting effects. Barbiturates, not benzodiazepines, are commonly associated with residual, or hangover, effects. A paradoxical reaction to a sedative would manifest as insomnia, euphoria, and excitation, not drowsiness. Tolerance means that the patient would need increased amounts of a drug to get the desired effects and would not have prolonged effects of the medication.DIF: Cognitive Level: AnalysisREF: p. 276TOP: Nursing Process: Evaluation MSC: NCLEX Client Needs Category: Physiologic Integrity: Pharmacologic and ParenteralTherapies 216. A patient takes temazepam [Restoril] for insomnia. The patient tells the nurse that a recent telephone bill lists several calls to friends that the patient does not remember making. What will the nurse do? a. Ask the patient about any alcohol consumption in conjunction with the benzodiazepine. b. Contact the prescriber to request an order for a benzodiazepine with a shorter duration. c. Reassure the patient that this is most likely caused by a paradoxical reaction to the benzodiazepine. d. Tell the patient that this is an example of anterograde amnesia, which is an expected effect of the benzodiazepine. ANS: A This patient is describing complex sleep-related behavior, which occurs when patients carry out complex behaviors while taking benzodiazepines but have no memory of their actions. These actions can occur with normal doses but are more likely with excessive doses or when benzodiazepines are combined with alcohol or other CNS depressants, so the nurse is correct in evaluating this possibility. The duration of the benzodiazepine does not contribute to this phenomenon. Paradoxical effects of benzodiazepines include insomnia, excitation, euphoria, anxiety, and rage. Anterograde amnesia occurs when patients have impaired recall of events that occur after dosing.DIF: Cognitive Level: AnalysisREF: p. 277TOP: Nursing Process: Evaluation MSC: NCLEX Client Needs Category: Physiologic Integrity: Pharmacologic and Parenteral Therapies 217. A nurse is discussing the use of benzodiazepines as sedative-hypnotic agents with a group of nursing students. A student asks about the actions of these drugs in the central nervous system. The nurse makes which correct statement? a. “Benzodiazepines affect the hippocampus and the cerebral cortex to cause anterograde amnesia.” b. “Benzodiazepines depress neuronal functions by acting at a single site in the brain.” c. “Benzodiazepines induce muscle relaxation by acting on sites outside the centralnervous system.” d. “Benzodiazepines promote sleep through effects on the limbic system.” ANS: A All beneficial and most adverse effects of benzodiazepines occur from depressant actions in the central nervous system (CNS); the various effects depend on the site of action. Anterograde amnesia is the result of effects in the hippocampus and the cerebral cortex. Benzodiazepines act at multiple sites in the CNS. Muscle relaxant effects are the result of actions on supraspinal motor areas in the CNS. Benzodiazepines promote sleep through effects on cortical areas and on the sleep-wakefulness “clock.”DIF: Cognitive Level: AnalysisREF: p. 275TOP: Nursing Process: Evaluation MSC: NCLEX Client Needs Category: Physiologic Integrity: Pharmacologic and Parenteral Therapies Rosenthal: Lehne's Pharmacotherapeutics for Advanced Practice Providers, 2nd Ed. Chapter 30: Management of Anxiety Disorders Test Bank Multiple Choice 218. A patient who has been taking alprazolam [Xanax] to treat generalized anxiety disorder (GAD) reports recently stopping the medication after symptoms have improved but reports having feelings of panic and paranoia. Which initial action by the nurse is correct? a. Ask the patient if the medication was stopped abruptly. b. Instruct the patient to resume taking the alprazolam. c. Notify the provider that the patient is experiencing a relapse. d. Suggest that the patient discuss taking buspirone [Buspar] with the provider. ANS: A Long-term use of benzodiazepines can cause physical dependence, with symptoms of panic, paranoia, and delirium occurring with abrupt withdrawal. These symptoms can be confused with symptoms of relapse of anxiety, so the nurse should evaluate this by first asking about how the medication was discontinued. If the symptoms are caused by a relapse, the patient should resume taking the alprazolam. Buspirone is not indicated.DIF: Cognitive Level: AnalysisREF: pp. 288TOP: Nursing Process: Assessment MSC: NCLEX Client Needs Category: Physiologic Integrity: Pharmacologic and Parenteral Therapies 219. A patient is diagnosed with anxiety after describing symptoms of tension, poor concentration, and difficulty sleeping that have persisted for over 6 months. Which medication will the nurse expect the provider to order for this patient? a. Alprazolam [Xanax] b. Amitriptyline [Elavil] c. Buspirone [Buspar] d. Paroxetine [Paxil] ANS: C This patient has symptoms of generalized anxiety disorder (GAD) that are not acute or severe. Buspirone is as effective as benzodiazepines but without causing CNS depression or having the same abuse potential. Symptoms develop slowly, which is acceptable in this case, since symptoms are not acute or severe. Alprazolam is a benzodiazepine and would be used in the short term to treat acute, severe anxiety. Amitriptyline is a TCA used to treat panic disorder. Paroxetine is an antidepressant used as a second-line drug for GAD.DIF: Cognitive Level: ApplicationREF: pp. 287TOP: Nursing Process: Assessment MSC: NCLEX Client Needs Category: Physiologic Integrity: Pharmacologic and Parenteral Therapies 220. A nurse is preparing a patient who will stop taking lorazepam [Ativan] for anxiety and begin taking buspirone [Buspar]. Which statement by the patient indicates a need for further teaching? a. “I can drink alcohol when taking Buspar, but not grapefruit juice.” b. “I may need to use a sedative medication if I experience insomnia.” c. “I may not feel the effects of Buspar for a few weeks.” d. “I should stop taking the Ativan when I start taking the Buspar.” ANS: D Ativan should not be withdrawn quickly; it must be tapered to prevent withdrawal symptoms. Moreover, Buspar does not have immediate effects. Because no cross-dependence occurs with these two medications, they may be taken together while the benzodiazepine is tapered. Because Buspar does not have sedative effects, patients can consume alcohol without increasing sedation. Levels of Buspar can be increased by grapefruit juice, leading to drowsiness and a feeling of dysphoria. Buspar can cause nervousness and excitement and does not have sedative effects, so patients with insomnia must use a sedative. Buspar does not have immediate effects.DIF: Cognitive Level: ApplicationREF: pp. 289TOP: Nursing Process: Planning MSC: NCLEX Client Needs Category: Physiologic Integrity: Pharmacologic and Parenteral Therapies 221. A patient reports having occasional periods of tremors, palpitations, nausea, and a sense of fear, which usually dissipate within 30 minutes. To treat this condition, the nurse anticipates the provider will prescribe a drug in which drug class? a. Benzodiazepines b. Monoamine oxidase inhibitors c. Selective serotonin reuptake inhibitors d. Tricyclic antidepressants ANS: C This patient is showing characteristics of panic disorder. All three major classes of antidepressants are effective, but selective serotonin reuptake inhibitors are first-line drugs. Benzodiazepines are second-line drugs and are rarely used because of their abuse potential. MAOIs are effective but are difficult to use because of their side effects and drug and food interactions. Tricyclic antidepressants are second-line drugs, and their use is recommended only after a trial of at least one SSRI has failed.DIF: Cognitive Level: ApplicationREF: pp. 290TOP: Nursing Process: Planning MSC: NCLEX Client Needs Category: Physiologic Integrity: Pharmacologic and Parenteral Therapies 222. Selective serotonin reuptake inhibitors are known to be effective for which disorders? Select all that apply. a. Generalized anxiety disorder (GAD) b. Obsessive-compulsive disorder c. Panic disorder d. Posttraumatic stress disorder e. Social anxiety disorder ANS: A , B , C , E SSRIs have been shown to be effective in treating GAD, OCD, panic disorder, and social anxiety disorder. They are used to treat PTSD but have not demonstrated effectiveness in clinical research.DIF: Cognitive Level: ApplicationREF: pp. 292TOP: Nursing Process: Implementation MSC: NCLEX Client Needs Category: Physiologic Integrity: Pharmacologic and Parenteral Therapies 223. uring an admission history, a patient reports a frequent need to return to a room multiple times to make sure an iron or other appliance is unplugged. What does the nurse understand about this patient’s behavior? a. It helps the patient reduce anxiety about causing a fire. b. It usually is treated with alprazolam [Xanax]. c. It seems perfectly normal to the patient. d. It will best respond to deep brain stimulation. ANS: A Patients with OCD have compulsive behaviors, such as repeatedly checking to make sure appliances have been unplugged. The compulsion is a ritualized behavior resulting from obsessive anxietyor fear that something bad will happen, such as starting a fire with an overheated appliance. Alprazolam is not a first-line drug for treating OCD. Patients usually understand that compulsive behaviors are excessive and senseless but are unable to stop. Deep brain stimulation is indicated for patients in whomother treatments have failed; its effectiveness at reducing symptoms has been shown to be about 40%.DIF: Cognitive Level: ApplicationREF: pp. 290TOP: Nursing Process: Assessment MSC: NCLEX Client Needs Category: Physiologic Integrity: Pharmacologic and Parenteral Therapies 224. A patient describes feelings of anxiety and fear when speaking in front of an audience and is having difficulty at work because of an inability to present information at meetings three or four times each year. The patient is reluctant to take long-term medications. The nurse will expect the provider to order which treatment? a. Alprazolam [Xanax] as needed b. Cognitive behavioral therapy c. Paroxetine [Paxil] d. Psychotherapy ANS: A This patient is describing social anxiety disorder; the symptoms are related to performance only and are not generalized to all social situations. Because this patient must speak in front of an audience only three or four times per year, a PRN medication can be used. Cognitive behavioral therapy is used for OCD. Paroxetine must be used continuously for at least 1 year. Psychotherapy can be used but is more effective when used in combination with drugs.DIF: Cognitive Level: ApplicationREF: pp. 291TOP: Nursing Process: Implementation MSC: NCLEX Client Needs Category: Physiologic Integrity: Pharmacologic and Parenteral Therapies 225. A patient who has obsessive-compulsive disorder (OCD) has been undergoing behavioral therapy but continues to exhibit symptoms that interfere with daily life. Which intervention will the nurse expect the provider to order for this patient? a. Alprazolam [Xanax] b. Buspirone [Buspar] c. Deep brain stimulation d. Fluoxetine [Paxil] ANS: D Patients with OCD usually respond optimally to a combination of an SSRI, such as fluoxetine, and behavioral therapy. Alprazolam and buspirone are used to treat GAD. Deep brain stimulation is used when other therapies fail totreat OCD.DIF: Cognitive Level: ApplicationREF: pp. 290TOP: Nursing Process: Planning MSC: NCLEX Client Needs Category: Physiologic Integrity: Pharmacologic and Parenteral Therapies 226. A nurse is performing an admission assessment on a patient. The patient reports taking alprazolam [Xanax] for “nerves.” The nurse knows that this patient is most likely being treated for which condition? a. Generalized anxiety disorder b. Obsessive-compulsive disorder (OCD) c. Panic disorder d. Posttraumatic stress disorder (PTSD) ANS: A Benzodiazepines are the first-choice drugs for anxiety, and alprazolam and lorazepam are prescribed most often. Selective serotonin reuptake inhibitors (SSRIs) are the first-line drugs for the treatment of OCD. Panic disorder is treated with any of the three classes of antidepressants: SSRIs, tricyclic antidepressants (TCAs), and monoamine oxidase inhibitors (MAOIs). Research has not shown any drug to be effective in the treatment of PTSD, although two SSRIs have been approved for use for this disorder.DIF: Cognitive Level: AnalysisREF: pp. 288TOP: Nursing Process: Assessment MSC: NCLEX Client Needs Category: Physiologic Integrity: Pharmacologic and Parenteral Therapies 227. An agitated, extremely anxious patient is brought to the emergency department. The prescriber orders a benzodiazepine. The nurse understands that benzodiazepines are used in this clinical situation based on which principle? a. Benzodiazepines have a very short half-life. b. Physical dependence is not a risk when taking benzodiazepines. c. Benzodiazepines are known to cure generalized anxiety. d. Benzodiazepines have a rapid onset of action. ANS: D The patient is clearly in a state ofextreme, uncontrolled anxiety. Benzodiazepines are the drugs of choice for acute episodes of anxiety because of their rapid onset of action. Benzodiazepines do not have a very short half-life. Benzodiazepines are associated with physical dependence. Benzodiazepines do not cure generalized anxiety, nor do any other drugs.DIF: Cognitive Level: AnalysisREF: pp. 288TOP: Nursing Process: Assessment MSC: NCLEX Client Needs Category: Physiologic Integrity: Pharmacologic and Parenteral Therapies Rosenthal: Lehne's Pharmacotherapeutics for Advanced Practice Providers, 2nd Ed. Chapter 31: Central Nervous System Stimulants and Attention-Deficit/Hyperactivity Disorder Test Bank Multiple Choice 228. A child will begin taking methylphenidate [Ritalin] for attention-deficit/hyperactivity disorder. Important baseline information about this patient will include: a. results of an electrocardiogram(ECG). b. family history of psychosis. c. height and weight. d. renal function. ANS: C Side effects of methylphenidate include a reduced appetite, and children taking these drugs should be monitored for growth suppression. Baseline height and weight measurements help with this ongoing assessment. The value of an ECG for children has not been proved, except when known heart disease is a factor. Excessive use of stimulants can produce a state of psychosis but is not related to the family history. Renal function tests are not indicated.DIF: Cognitive Level: ApplicationREF: pp. 298TOP: Nursing Process: Assessment MSC: NCLEX Client Needs Category: Physiologic Integrity: Reduction of Risk Potential 229. A child has been taking SD methylphenidate [Ritalin], 10 mg at 0800 and 1200 and 5 mg at 1600, for 2 months. The parents tell the nurse that the child sometimes misses the noon dose while at school. The child’s appetite is normal. The teacher has reported a slight improvement in hyperactivity and impulsivity. What will the nurse do? a. Ask the prescriber whether this child could be given methylphenidate [Concerta]. b. Contact the prescriber to suggest using a nonstimulant medication. c. Reinforce the need to take all doses as prescribed. d. Suggest drug holidays for the child on weekends. ANS: A This child is showing slight improvement with the medication but has trouble taking the noon dose; therefore, a once-daily formulation would increase compliance and improve effects. There is no indication to use a nonstimulant medication, because the child’s appetite is normal. If 3 times/day dosing were the only option available, reinforcing the need to take all doses would be necessary; however, some children avoid taking medication at school because of the stigma attached to being different from their peers. The use of drug holidays is controversial; this approach is used when growth suppression is a problem.DIF: Cognitive Level: ApplicationREF: pp. 295TOP: Nursing Process: Evaluation MSC: NCLEX Client Needs Category: Physiologic Integrity: Pharmacologic and Parenteral Therapies 230. An adult patient will begin taking atomoxetine [Strattera] for attention- deficit/hyperactivity disorder. What will the nurse teach this patient? a. Appetite suppression does not occur, because this drug is not a stimulant. b. Stopping the drug abruptly will cause an abstinence syndrome. c. Suicidal thoughts may occur and should be reported to the provider. d. Therapeutic effects may not be felt for 1 to 3 weeks after beginning therapy. ANS: D Atomoxetine is a selective inhibitor of norepinephrine (NE) reuptake, and its effects probably are the result of adaptive changes that occur after uptake blockade, which can take 1 to 3 weeks. Appetite suppression is an adverse effect of this drug. Atomoxetine does not have abusepotential, and abstinence syndrome does not occur when it is withdrawn. Suicidal thoughts may occur in children and adolescents, but not in adults.DIF: Cognitive Level: ApplicationREF: pp. 298TOP: Nursing Process: Implementation MSC: NCLEX Client Needs Category: Physiologic Integrity: Pharmacologic and Parenteral Therapies 231. A child is diagnosed with attention-deficit/hyperactivity disorder (ADHD). The prescriber orders a central nervous system stimulant. Which statement by the child’s parent indicates a need for further teaching? a. “I should report insomnia and poor appetite to his provider.” b. “I will make sure he takes his medication after breakfast every day.” c. “This drug will make him less impulsive while he’s at school.” d. “This medication will help my child focus so he can learn new behaviors.” ANS: C Stimulants do not suppress negative behaviors directly and do not directly cause a decrease in hyperactivity. They act by improving attention and focus so that positive behaviors can be learned to replace negative behaviors. Insomnia and poor appetite are common side effects and should be reported to the provider, because alternate dosing regimens often counteract these effects. Taking the medication either during or after breakfast prevents morning appetite suppression at breakfast time. Stimulants improve focus and allow new, more positive behaviors to be learned.DIF: Cognitive Level: ApplicationREF: pp. 298TOP: Nursing Process: Evaluation MSC: NCLEX Client Needs Category: Physiologic Integrity: Pharmacologic and Parenteral Therapies 232. A university student who is agitated and restless and has tremors is brought to the emergency department. The patient’s heart rate is 110 beats/minute, the respiratory rate is 18 breaths/minute, and the blood pressure is 160/95 mm Hg. The patient reports using concentrated energydrinks to stayawake during finals week. What complication will the nurse monitor for in this patient? a. CNS depression b. Cardiac arrest c. Respiratory failure d. Seizures ANS: D In large doses, caffeine produces nervousness and tremors; in very large doses, it can cause seizures. This patient has been drinking concentrated energy drinks which are high in caffeine. Caffeine is a stimulant and produces CNS excitation, not depression. Although cardiac side effects are common with caffeine, cardiac arrest is not. Respiratory failure is not an effect of caffeine toxicity.DIF: Cognitive Level: AnalysisREF: pp. 296TOP: Nursing Process: Evaluation MSC: NCLEX Client Needs Category: Physiologic Integrity: Reduction of Risk Potential 233. A patient who is morbidly obese is admitted for treatment. The prescriber orders lisdexamfetamine [Vyvanse]. The nurse will be concerned if this patient shows signs of: a. anorexia. b. dyspnea. c. insomnia. d. loquaciousness. ANS: B Stimulants can produce cardiovascular effects. Any patient reporting shortness of breath needs to be evaluated for cardiovascular problems. Anorexia, or poor appetite, is an expected effect of stimulants and is the desired effect when these drugs are used for obesity. Stimulants increase alertness and can cause insomnia, which is an expected effect at therapeutic doses. Loquaciousness is an expected effect at therapeutic doses.DIF: Cognitive Level: ApplicationREF: pp. 293TOP: Nursing Process: Evaluation MSC: NCLEX Client Needs Category: Physiologic Integrity: Reduction of Risk Potential 234. A nurse is providing education to a group of patients regarding amphetamines. To evaluate the group’s understanding, the nurse asks a participant what effects amphetamines would have on her. The participant shows that she understands the effects of these drugs if she gives which answers? Select all that apply. a. “Amphetamines increase fatigue.” b. “Amphetamines suppress the perception of pain.” c. “Amphetamines increase appetite.” d. “Amphetamines increase the heart rate.” e. “Amphetamines elevate mood.” ANS: B , D , E At customary doses, amphetamines increase wakefulness and alertness, reduce fatigue, elevate mood, and augment self-confidence and initiative. Amphetamines also suppress appetite and the perception of pain and increase the heart rate. Amphetamines do not increase fatigue or appetite.DIF: Cognitive Level: ApplicationREF: pp. 293TOP: Nursing Process: Assessment MSC: NCLEX Client Needs Category: Physiologic Integrity: Pharmacologic and Parenteral Therapies 235. A nurse working the night shift begins taking modafinil [Alertec]. The nurse is telling a coworker about the medication. Which statement is correct? a. “I can take it during pregnancy.” b. “It doesn’t have cardiovascular side effects.” c. “It is safe and has no serious adverse effects.” d. “It will not interfere with my normal sleep.” ANS: D Modafinil is used to increase wakefulness in patients with excessive sleepiness, including those with shift-work sleep disorder (SWSD). It acts without disrupting nighttime sleep. It is embryotoxic in laboratory animals and therefore is contraindicated during pregnancy. It can increase the heart rate and blood pressure. In rare cases, it has been linked to serious skin reactions, including Stevens-Johnson syndrome, erythema multiforme, and toxic epidermal necrolysis.DIF: Cognitive Level: AnalysisREF: pp. 296TOP: Nursing Process: Planning MSC: NCLEX Client Needs Category: Physiologic Integrity: Pharmacologic and Parenteral Therapies 236. A nurse is teaching the parents of a child who has attention-deficit/hyperactivity disorder about methylphenidate [Concerta]. Which statement by the child’s parents indicates understanding of the teaching? a. “The effects of this drug will wear off in 4 to 6 hours.” b. “The tablet needs to be swallowed whole, not crushed or chewed.” c. “This medication has fewer side effects than amphetamines.” d. “We should call the provider if we see parts of the medicine in our child’s stools.” ANS: B Concerta tablets must be swallowed whole and should not be crushed, chewed, or dissolved in liquids. This is a long-duration preparation with effects that last 10 to 12 hours. Methylphenidate has the same actions and adverse effects as amphetamines. The tablet shell may not fully dissolve in the gastrointestinal (GI) tract; therefore, tablet “ghosts” in the stool are normal.DIF: Cognitive Level: ApplicationREF: pp. 297TOP: Nursing Process: Implementation MSC: NCLEX Client Needs Category: Physiologic Integrity: Pharmacologic and Parenteral Therapies 237. A young adult begins taking clonidine [Kapvay] to treat ADHD symptoms after suffering anorexia with methylphenidate [Ritalin]. What will the nurse include when teaching this patient about taking clonidine? a. “Avoid consuming alcohol while taking this medication.” b. “Insomnia may still occur while taking this drug.” c. “You will need to pick up a written prescription every 30 days.” d. “You may crush the tablets and put them in food.” ANS: A Clonidine causes somnolence, which is made worse by alcohol or other CNS depressants, so clients should avoid alcohol while taking clonidine. Insomnia and anorexia are not side effects of clonidine. Clonidine is not a controlled substance, so prescriptions may be refilled over the phone and may be written for more than 1 month at a time. The tablets must be swallowed whole and should not be crushed or chewed.DIF: Cognitive Level: ApplicationREF: pp. 299TOP: Nursing Process: Planning MSC: NCLEX Client Needs Category: Health Promotion and Maintenance 238. A parent thinks a school-aged child has ADHD. The nurse asks the parent to describe the child’s behaviors. Which behaviors are characteristic of ADHD? Select all that apply. a. Anxiety b. Compulsivity c. Hyperactivity d. Inattention e. Impulsivity ANS: C, D , E ADHD is characterized by inattention, hyperactivity, and impulsivity. Anxiety and compulsivity are not characteristic of ADHD.DIF: Cognitive Level: ComprehensionREF: pp. 297TOP: Nursing Process: Assessment MSC: NCLEX Client Needs Category: Health Promotion and Maintenance Rosenthal: Lehne's Pharmacotherapeutics for Advanced Practice Providers, 2nd Ed. Chapter 32: Drug Abuse I: Basic Considerations Test Bank Multiple Choice 239. The nurse is teaching a nursing student about management of controlled substances in medication administration. Which statement bythe nursing student indicates understanding of the teaching? a. “If there is a difference between state and federal laws governing a scheduled drug, the federal law takes precedence.” b. “Prescriptions for drugs in Schedules III and IV may be written to include up to 5 refills.” c. “Schedule I drugs may only be given to hospitalized patients.” d. “To reduce the possibility of abuse of a drug that is Schedule II, the prescriber shouldcall the prescription to the pharmacy.” ANS: B Providers may prescribe Schedule III and IV drugs orally by phone, written as a prescription, or electronically, and may provide up to 5 refills. When state and federal laws differ, the more restrictive law takes precedence, whether it is the state or the federal law. Schedule I drugs have no approved uses. Schedule II drugs must be typed or written in indelible ink or pencil and signed by the provider or may be submitted electronically. They may be called in an emergency but must be followed by a written prescription within 72 hours.DIF: Cognitive Level: ApplicationREF: pp. 307TOP: Nursing Process: Implementation MSC: NCLEX Client Needs Category: Physiologic Integrity: Pharmacologic and Parenteral Therapies 240. A psychiatric nurse is caring for a drug-addicted patient. The nurse knows that the ideal goal of drug rehabilitation for this patient is: a. abstinence from the drug. b. decreasing episodes of relapse. c. minimizing drug cravings. d. reduction of drug use. ANS: A The goal of treatment is complete cessation of the drug. Decreasing episodes of relapse, minimizing cravings, and reducing drug use are all steps toward achieving eventual abstinence.DIF: Cognitive Level: AnalysisREF: pp. 306TOP: Nursing Process: Implementation MSC: NCLEX Client Needs Category: Physiologic Integrity: Pharmacologic and Parenteral Therapies 241. A nurse is teaching a group of nursing students about drug abuse. Which statement by a student indicates a need for further teaching? a. “Patients who experience physical dependence will show compulsive drug-seeking behavior.” b. “People who are addicted to a drug do not necessarily have tolerance to that drug.” c. “Physical dependence means that abstinence syndrome will occur if a drug is withdrawn.” d. “Physical dependence often contributes to addictive behavior but does not cause it.” ANS: A Physical dependence occurs with prolonged drug exposure and, through neuroadaptive processes, results in abstinence syndrome if a drug is withdrawn. It does not necessarily result in compulsive drug-seeking behavior. Patients can have drug addiction, which involves compulsive drug seeking without having developed a tolerance to drug effects. Addictive behavior is the result of psychologic dependence with an intense subjective need for a drug. Because abstinence syndrome is uncomfortable, physical dependence can increase subjective feelings for a drug.DIF: Cognitive Level: AnalysisREF: pp. 304TOP: Nursing Process: Evaluation MSC: NCLEX Client Needs Category: Physiologic Integrity: Pharmacologic and Parenteral Therapies 242. A nurse is caring for four patients. The nurse would be concerned about which patient developing a substance use disorder? a. A college student who reports having experimented with marijuana in the past year. b. An older adult patient with terminal cancer who requires twice the normal dose of morphine for pain relief. c. A patient in moderate to severe pain after a total hip replacement who asks for pain medication an hour before the next dose is due. d. A patient whose history indicates the use of prescription narcotic analgesics for back and headache pain. ANS: D Patients who use narcotics for minor pains are more likely to be compulsive drug seekers. A college student who experiments with an illegal substance is not necessarily going to develop a substance use disorder. An elderly patient with terminal cancer pain has most likely developed physical dependence and tolerance to morphine but is not a substance abuser. Patients with significant pain who ask for more frequent dosing are not showing substance use disorder.DIF: Cognitive Level: AnalysisREF: pp. 305TOP: Nursing Process: Diagnosis MSC: NCLEX Client Needs Category: Physiologic Integrity: Pharmacologic and Parenteral Therapies 243. A nurse is teaching a class on addiction. Which statement by one of the class participants indicates a need for further teaching? a. “Addictive drugs lead to dopamine release in amounts similar to those released by normal reward circuits.” b. “Neural remodeling leads to decreased dopamine release, leaving users with feelings of lifelessness and depression.” c. “Over time, the brain will develop reduced responses to many addictive drugs.” d. “With the use of a drug over time, the brain undergoes synaptic remodeling.” ANS: A Drugs of addiction use the same reward circuits that are used to reward biologically critical behaviors such as eating and sexual intercourse. However, addictive drugs lead to dopamine release that can be 2 to 10 times higher than that released naturally. Eventually, neural remodeling occurs, causing the brain to produce less dopamine and to reduce the number of dopamine receptors, leaving addicts feeling depressed and lifeless. This process of down-regulation reduces the response to the drug. All of this is part of the synaptic remodeling that occurs when the brain is exposed to a drug over a period of time.DIF: Cognitive Level: AnalysisREF: pp. 305TOP: Nursing Process: Implementation MSC: NCLEX Client Needs Category: Physiologic Integrity: Pharmacologic and Parenteral Therapies 244. A patient who has been taking a medication with a side effect of drowsiness stops taking the medication after several weeks. The patient reports feeling anxious and jittery. The nurse understands that this response is due to: a. addiction. b. psychologic dependence. c. tolerance. d. withdrawal syndrome. ANS: D Withdrawal syndrome occurs when patients have developed a physical dependence on a drug and then often show symptoms that are the opposite of the drug’s effect when the drug is withdrawn. Addiction is characterized by compulsive drug seeking. Psychologic dependence is an intense subjective need for a drug. Tolerance develops when increased amounts of a drug are needed to achieve the drug’s effects.DIF: Cognitive Level: AnalysisREF: pp. 304TOP: Nursing Process: Evaluation MSC: NCLEX Client Needs Category: Physiologic Integrity: Pharmacologic and Parenteral Therapies 245. A patient is ready for discharge home from a lengthy hospital stay after a motor vehicle accident. The patient suffered multiple fractures and required large doses of morphine for several weeks. The nurse preparing the patient for discharge notes that the patient requests the maximum dose of the oral opioid analgesic at the exact intervals it is prescribed. The nurse is correct to suspect that what has occurred? a. Addiction b. Compulsive drug seeking c. Cross-tolerance d. Drug tolerance ANS: D Patients who use a drug regularly develop tolerance to the drug when a dose produces a smaller response than it did initially. This patient has been on large doses of opioids for several weeks and has developed tolerance to this class of drugs. Addiction is characterized by compulsive drug seeking, which has not occurred. A patient using narcotics for severe pain is not a compulsive drug seeker. Cross-tolerance occurs when tolerance to one drug confers tolerance to another drug. The opioid analgesic for home use is in the same drug classification, so this is not cross-tolerance.DIF: Cognitive Level: AnalysisREF: pp. 304TOP: Nursing Process: Evaluation MSC: NCLEX Client Needs Category: Physiologic Integrity: Reduction of Risk Potential 246. A nursing student is caring for a patient who is addicted to several drugs. The student tells the nurse that the patient “got this way on purpose.” Which response by the nurse is most appropriate? a. “Peer pressure and social factors determine individual choices.” b. “Physical dependence is necessary for addiction to occur.” c. “Preexisting psychopathology underlies most drugabuse.” d. “Some individuals are more vulnerable to drug abuse than others.” ANS: D Some individuals are more prone to becoming drug abusers than others for a variety of reasons, including physiologic, psychologic, social, emotional, and genetic reasons. Peer pressure, social factors, the development of physical dependence, and underlying psychologic disorders contribute to the development of addiction but are not the determining factors.DIF: Cognitive Level: AnalysisREF: pp. 305TOP: Nursing Process: Evaluation MSC: NCLEX Client Needs Category: Physiologic Integrity: Pharmacologic and Parenteral Therapies Rosenthal: Lehne's Pharmacotherapeutics for Advanced Practice Providers, 2nd Ed. Chapter 33: Drug Abuse II: Alcohol Test Bank Multiple Choice 247. A nurse is providing education to a group of college students about the long-term effects of alcohol. Which statement by a student indicates understanding of the teaching? a. “Chronic alcohol use contributes to the development of osteoporosis.” b. “Chronic use of alcohol can actually decrease the risk ofcardiomyopathy.” c. “Even small amounts of alcohol are related to the development of certaincancers.” d. “Pancreatitis is not a common problem among chronic users of alcohol.” ANS: C The risk of certain types of cancers is increased even among moderate alcohol users. The current data suggest that no amount of alcohol is safe with regard to cancer risk. Chronic alcohol use actually reduces the risk of osteoporosis. Chronic consumption of alcohol is associated with an increased risk of cardiomyopathy. Pancreatitis occurs in only 5% of heavy drinkers, but 35% of all cases of pancreatitis are related to alcohol.DIF: Cognitive Level: AnalysisREF: pp. 308TOP: Nursing Process: Evaluation MSC: NCLEX Client Needs Category: Health Promotion and Maintenance 248. A nurse is obtaining an admission history on a patient who reports daily drinking for several years. When the nurse questions the patient further, the patient reports drinking up to five or six drinks each day. The patient expresses worry about liver damage. What will the nurse do? a. Contact the patient’s provider to request liver function studies. b. Explain that hepatitis, progressing to severe liver impairment, is likely. c. Inform the patient that the history indicates that cirrhosis is likely to occur. d. Tell the patient that stopping drinking will reverse any effects on the liver. ANS: A Chronic drinking can lead to hepatitis in about 90% of heavy users. The nurse would be correct to request laboratory studies of liver function. Until the laboratory values are known, the degree of damage to the liver is unknown, so the likelihood of severe outcomes cannot be predicted. Acute drinking causes the accumulation of fat and protein in the liver, which is reversible, but this may not be the case with chronic drinking.DIF: Cognitive Level: AnalysisREF: pp. 308TOP: Nursing Process: Assessment MSC: NCLEX Client Needs Category: Physiologic Integrity: Pharmacologic and Parenteral Therapies 249. During a health history, the nurse asks a male patient about alcohol use. The patient tells the nurse that he and his wife are trying to conceive a pregnancy and he is using alcohol to lower his inhibitions. What will the nurse counsel this patient? a. “Alcohol causes increased masculinization.” b. “Alcohol may cause testicular atrophy andsterility.” c. “Alcohol will improve your chances ofconceiving.” d. “Alcohol will also help you to ejaculate.” ANS: B Alcohol may induce feminization and cause testicular atrophy, impotence, sterility, and breast enlargement. Alcohol significantly decreases the ability to ejaculate in spite of lowering inhibitions.DIF: Cognitive Level: AnalysisREF: pp. 308TOP: Nursing Process: Assessment MSC: NCLEX Client Needs Category: Physiologic Integrity: Pharmacologic and Parenteral Therapies 250. A nursing student asks a nurse to discuss alcoholism and alcohol use disorder. Which statement by the nurse is correct? a. “Alcohol use disorder can occur without the development of tolerance or physical dependence.” b. “Individuals with alcohol use disorder develop cross-tolerance with opioidanalgesics.” c. “Initial symptoms of abstinence syndrome occur within 1 to 2 hours after withdrawal of alcohol.” d. “With severe alcoholism, most alcoholics have delirium tremens when alcohol is withdrawn.” ANS: A Alcohol abuse can occur without the development of tolerance or physical dependence, although these generally develop with long-term use of alcohol. Cross-tolerance occurs between alcohol and general anesthetics and other CNS depressants, but not with opioids. The symptoms of abstinence syndrome begin to manifest 12 to 72 hours after the last drink. Fewer than 1% of alcoholics experience delirium tremens.DIF: Cognitive Level: AnalysisREF: pp. 311TOP: Nursing Process: Assessment MSC: NCLEX Client Needs Category: Physiologic Integrity: Pharmacologic and Parenteral Therapies 251. A pregnant patient in labor tells the nurse that she is afraid she may have harmed her fetus by consuming alcohol. What is an appropriate response by the nurse? a. Ask the patient how much alcohol she consumed, and at which stage of her pregnancy. b. Reassure the patient that the risk is likely to be minimal. c. Tell the patient that no amount of alcohol is considered safe during pregnancy. d. Tell the patient that the full range of outcomes may not be evident for years. ANS: A Although heavy use of alcohol has known adverse effects on the fetus, the effects of lower levels are unknown. The nurse should first question the patient about the amount of alcohol consumed at which stages of pregnancy to better determine the potential risk. Many women consume alcohol before knowing they are pregnant without any seeming ill effects. Reassuring this patient that her risks are low is not appropriate without further information. Moreover, reassuring her without getting more information only belittles her fears. Telling a patient that no amount of alcohol is safe during pregnancy would be an appropriate intervention during counseling of a woman who has just discovered she is pregnant; however, it would only intensify the fears of this patient. Telling the patient that the outcomes will not be evident for years would only intensify her fears.DIF: Cognitive Level: AnalysisREF: pp. 309TOP: Nursing Process: Assessment MSC: NCLEX Client Needs Category: Physiologic Integrity: Pharmacologic and Parenteral Therapies 252. A male patient tells a nurse that he drinks a six-pack of beer a day. When the nurse begins to question him further about his alcohol consumption, he says, “You sound like my wife. She’s always nagging me to quit. It’s only beer!” Which response by the nurse is most appropriate? a. “Because the alcohol in beer is diluted in a larger volume, it is absorbed more slowly.” b. “Have you considered switching to wine? It has chemicals that protect your heart.” c. “The amount you drink is equivalent to six shots of whiskey each day.” d. “You could try to cut the amount in half to a level that is better for your health.” ANS: C The amount of alcohol in one can of beer is equivalent to the amount in a shot of whiskey or one glass of wine. Even though the amount per volume of fluid is less in beer, the absorption of the alcohol does not change; it just may take longer to consume it. Even though red wine contains resveratrol, which may reduce cardiovascular disease, the amount is too small to have significant effects and would be offset by the amount this man consumes on a daily basis. It is not likely that someone already accustomed to consuming six cans of beer a day would be able to cut the amount in half. Moreover, daily drinking in itself may still constitute an alcohol use disorder, depending on other factors.DIF: Cognitive Level: ApplicationREF: pp. 309TOP: Nursing Process: Diagnosis MSC: NCLEX Client Needs Category: Physiologic Integrity: Pharmacologic and Parenteral Therapies 253. A nurse is discussing alcohol abuse with a group of nursing students. One student asks whether alcohol consumption has any beneficial effects. The nurse replies that, in moderate amounts, alcohol: a. helps people to sleep well. b. improves sexual responsiveness. c. may protect against dementia. d. prevents hypothermia. ANS: C In moderate amounts, alcohol helps preserve cognitive function in the older adult and mayprotect against dementia. Alcohol disrupts sleep and alters sleep cycles, reducing total sleeping time and the quality of sleep. Alcohol lowers inhibitions but diminishes sexual responsiveness. Alcohol dilates cutaneous blood vessels, which actually promotes heat loss.DIF: Cognitive Level: AnalysisREF: pp. 307TOP: Nursing Process: Evaluation MSC: NCLEX Client Needs Category: Physiologic Integrity: Pharmacologic and Parenteral Therapies 254. A college student who is unresponsive is brought to the emergency department by friends, who say that their friend drank more than half of a large bottle of whiskey 3 hours ago. Assessment reveals a blood alcohol level of 0.32%. The vital signs are BP, 88/32 mm Hg; R, 6/minute; T, 96.8° F; and P, 76/minute and weak and thready. The nurse should prepare the patient for which intervention? a. IV fluids and stimulants b. Charcoal administration c. Gastric lavage and dialysis d. Naloxone [Narcan] administration ANS: C The average rate at which a person can metabolize alcohol is about 15 mL (0.5 ounce) per hour. The patient in this scenario has consumed more than half of a large bottle of whiskey within 3 hours. Alcohol can be removed from the body by gastric lavage and dialysis. Gastric lavage “washes out” most of the alcohol if any is left in the gut, and dialysis is implemented to reduce the chance of renal failure and cardiovascular shock. Although intravenous fluids may be appropriate, stimulants are contraindicated for this patient. Charcoal is not indicated in this situation. Naloxone is indicated in opiate overdoses, not in alcohol overdoses.DIF: Cognitive Level: AnalysisREF: pp. 310TOP: Nursing Process: Implementation MSC: NCLEX Client Needs Category: Physiologic Integrity: Reduction of Risk Potential 255. A patient is brought to the emergency department after a motor vehicle accident. The patient’s speech is slurred. The nurse notes the smell of alcohol on the patient’s breath and observes hand tremors. The patient’s blood alcohol level is 0.4%. The nurse will expect to: a. find that the patient has lost consciousness within a short time. b. administer naltrexone [ReVia] and prepare for gastric lavage. c. give carbamazepine to reduce the risk of seizures. d. provide mechanical ventilation and oxygen. ANS: D A blood alcohol level that exceeds 0.4% poses a substantial risk of respiratory depression. Patients who are chronic abusers of alcohol maydevelop tolerance to other effects of increased blood levels, such as sedation, or behavioral changes, but there is very little tolerance to respiratory depression. A patient with a blood alcohol level of 0.4% must be treated for respiratorydepression, usually with mechanical ventilation. If this patient has developed tolerance, which is likely because loss of consciousness has not already occurred, the nurse cannot expect that the patient will lose consciousness. Naltrexone is not used for acute toxicity. Carbamazepine is used as an adjunct to benzodiazepines and may be used after this patient’s immediate needs have been addressed.DIF: Cognitive Level: AnalysisREF: pp. 310TOP: Nursing Process: Evaluation MSC: NCLEX Client Needs Category: Physiologic Integrity: Reduction of Risk Potential 256. A patient asks a nurse about the effects of chronic alcohol use on the heart. The nurse’s best response would be which statement? a. “Chronic alcohol use affects the liver more adversely than it does the heart.” b. “Drinking more than two drinks a day protects the heart from atherosclerosis.” c. “Long-term alcohol use can damage the heart and cause heart failure.” d. “Over time, alcohol use can lower your blood pressure.” ANS: C Chronic abuse of alcohol results in direct damage to the myocardium, increasing the risk of heart failure. Chronic alcohol abuse has a significant effect on the heart and also affects the liver. Drinking fewer than two alcoholic beverages a day potentially protects the heart from atherosclerosis. Alcohol consumption produces a dose-dependent elevation of blood pressure.DIF: Cognitive Level: ApplicationREF: pp. 308TOP: Nursing Process: Implementation MSC: NCLEX Client Needs Category: Physiologic Integrity: Reduction of Risk Potential 257. A patient who is an active alcoholic is admitted to the hospital for surgery. The nurse reviewing orders for this patient would be correct to question which postoperative medication for this patient? a. Acetaminophen b. Diazepam c. Morphine d. Thiamine ANS: A Acetaminophen poses a risk of fatal liver damage in alcoholics, because evidence indicates that even modest alcohol consumption combined with acetaminophen has this effect. Diazepam would probably be useful in this case, because it is used to aid alcohol withdrawal. However, diazepam cannot be taken with alcohol, because the central nervous system (CNS) depressive effects would be compounded. Likewise, morphine is safe as long as it is not given with alcohol. Thiamine is a vitamin that often is deficient in alcoholics, so thiamine would be indicated.DIF: Cognitive Level: ApplicationREF: pp. 310TOP: Nursing Process: Planning MSC: NCLEX Client Needs Category: Physiologic Integrity: Reduction of Risk Potential Rosenthal: Lehne's Pharmacotherapeutics for Advanced Practice Providers, 2nd Ed. Chapter 34: Drug Abuse III: Nicotine and Smoking Test Bank Multiple Choice 258. A 4-year-old child is brought to the emergency department with symptoms of nausea and vomiting and a weak, thready pulse of 120 beats/minute after ingesting several cigarettes at home. The nurse caring for this child will expect to provide which treatment? a. Gastric lavage b. Hemodialysis c. Respiratory support d. Vasoconstrictors ANS: C Nicotine is highly toxic, and the most prominent symptoms are those involving the cardiovascular, GI, and central nervous systems. This child is showing signs of toxicity. Respiratory arrest can occur, because nicotine affects the muscles of respiration. Respiratory support is the key to management; no antidote is available to nicotine poisoning. Nicotine undergoes rapid metabolism, so recovery can occur in a few hours. Gastric lavage, hemodialysis, and vasoconstrictors are not recommended.DIF: Cognitive Level: ApplicationREF: pp. 318TOP: Nursing Process: Planning MSC: NCLEX Client Needs Category: Physiologic Integrity: Reduction of Risk Potential 259. A patient reports a desire to stop smoking and asks what is available without a prescription to help with smoking cessation. The nurse tells the patient that which method is best? a. Abrupt discontinuation to shorten withdrawal effects b. Nicotine replacement and 1-800-QUITNOW c. Nicotine replacement products tapered over a year d. Support groups without the use of medications ANS: B Nicotine addiction can be treated with pharmacologic agents or counseling, but the combination of these two approaches is more effective than either one alone. Abrupt cessation is better than tapering off, because the withdrawal effects are not so prolonged; however, this approach is not recommended as the best way to quit. Nicotine replacement products should be discontinued after a few weeks to months after quitting smoking, because theycontain nicotine, which is both harmful and addicting. Support groups alone can work but are not as effective as the combination of support groups and medication.DIF: Cognitive Level: ApplicationREF: pp. 319TOP: Nursing Process: Planning MSC: NCLEX Client Needs Category: Physiologic Integrity: Pharmacologic and Parenteral Therapies 260. A patient who wants to quit smoking has begun taking varenicline [Chantix]. The patient reports experiencing mood swings and depression and a desire to cause harm to herself. What will the nurse tell this patient? a. “These symptoms are common and will disappear over time.” b. “These symptoms may indicate an underlying psychiatric disorder.” c. “You may need an increased dose to overcome these symptoms of nicotine withdrawal.” d. “You should notify your provider of these symptoms immediately.” ANS: D Varenicline can cause serious neuropsychiatric effects, including mood swings, depression, and self-injurious behavior. Because suicidality is a risk, patients experiencing these effects should contact their provider immediately. The symptoms are not common and are not likely to abate. It is not known if these symptoms indicate an underlying disorder or if the drug causes the symptoms. Increasing the dose will increase the symptoms, since they are related to the drug.DIF: Cognitive Level: AnalysisREF: pp. 322TOP: Nursing Process: Implementation MSC: NCLEX Client Needs Category: Physiologic Integrity: Pharmacologic and Parenteral Therapies 261. A patient who wants to quit smoking has a prescription for varenicline [Chantix], which will be used with a nicotine patch. The patient asks the nurse why the varenicline is necessary. Which statement by the nurse is correct? a. “It helps patients experiencing withdrawal to sleep better.” b. “It helps reduce anxiety and other withdrawal symptoms.” c. “It will help reduce the likelihood of addiction to the patch.” d. “The drug blocks nicotine’s access to ‘pleasure’ receptors.” ANS: D Varenicline is a partial agonist at nicotinic receptors and helps block nicotine’s access to these receptors. A common side effect is sleep disturbances. Buspirone is a smoking cessation agent that acts to reduce withdrawal side effects. Varenicline is not used to reduce addiction to the patch.DIF: Cognitive Level: ApplicationREF: pp. 322TOP: Nursing Process: Implementation MSC: NCLEX Client Needs Category: Physiologic Integrity: Pharmacologic and Parenteral Therapies 262. The spouse of a patient who smokes wonders why anyone would want to engage in “such a disgusting habit.” What will the nurse tell the spouse? a. Nicotine causes relaxation and helps with sleep. b. Nicotine increases alertness and promotes dopamine release. c. Nicotine lowers blood pressure. d. Nicotine settles the stomach and reduces nausea and vomiting. ANS: B Nicotine increases alertness, facilitates memory, and improves cognition. Even though users report feeling relaxed when using nicotine, it actually increases arousal and does not cause relaxation. Nicotine elevates blood pressure. Nicotine causes the gastrointestinal (GI) side effects of nausea, vomiting, and increased motility.DIF: Cognitive Level: AnalysisREF: pp. 318TOP: Nursing Process: Implementation MSC: NCLEX Client Needs Category: Physiologic Integrity: Pharmacologic and Parenteral Therapies 263. A patient asks about nicotine patches for smoking cessation and wants to know the difference between the 24-hour patch and the 16-hour patch. Which response by the nurse is correct? a. “The 16-hour patch is for patients who have trouble sleeping.” b. “The 16-hour patch simulates usual nicotine ingestion patterns.” c. “The 24-hour patch is for persons weighing more than 100 pounds.” d. “The 24-hour patch is recommended for heavier smokers.” ANS: B Nicotine transdermal patches usually are packaged in systems, with progressively smaller doses of nicotine. The 16-hour patch is designed to be removed at bedtime; this simulates the usual nicotine dosing produced by smoking. It does not necessarily affect the ability to sleep. Individuals who weigh less than 100 pounds are advised to use smaller patches. Heavier smokers are advised to begin with larger patches.DIF: Cognitive Level: ApplicationREF: pp. 321TOP: Nursing Process: Planning MSC: NCLEX Client Needs Category: Physiologic Integrity: Pharmacologic and Parenteral Therapies 264. A patient with a desire to stop smoking asks a nurse about nicotine chewing gum [Nicorette]. The patient currently smokes 30 cigarettes per day. Which statement by the nurse is correct? a. “Stop using the gum 6 months after you stop using cigarettes.” b. “Use the 4-mg strength gum and chew one piece every 2 to 3 hours.” c. “Use the gum whenever you feel a craving for a cigarette.” d. “You should start with 30 pieces of the 2-mg strength gum per day.” ANS: B Nicorette gum is available in two strengths, 2 and 4 mg. Patients who smoke more than 25 cigarettes per day should use the 4-mg strength. Dosing the gum on a regular schedule of every 2 to 3 hours has proved to be more effective than as-needed use. Use of the gum longer than 6 months total is not recommended, and the gum should be stopped 3 months after the last cigarette. PRN dosing is not as effective as regular dosing. The dose for a heavy smoker is 4-mg strength gum, one piece every 2 to 3 hours.DIF: Cognitive Level: ApplicationREF: pp. 320TOP: Nursing Process: Planning MSC: NCLEX Client Needs Category: Physiologic Integrity: Pharmacologic and Parenteral Therapies 265. A nurse is conducting a smoking cessation class in the community and is discussing the physiologic effects of nicotine. The nurse is correct to teach that these effects include which of the following? Select all that apply. a. Increased blood pressure b. Decreased gastric acid c. Vomiting d. Suppression of nausea e. Increased alertness f. Suppression of appetite ANS: A , C , E , F The physiologic effects of nicotine include increased blood pressure and other cardiovascular effects, vomiting, increased alertness, and suppression of appetite. Decreased gastric acid and suppression of nausea are not physiologic responses associated with nicotine use.DIF: Cognitive Level: ApplicationREF: pp. 317TOP: Nursing Process: Assessment MSC: NCLEX Client Needs Category: Physiologic Integrity: Pharmacologic and Parenteral Therapies 266. A prescriber has ordered nicotine nasal spray for a patient to assist with smoking cessation. Which statement will the nurse include when teaching the patient about the medication? a. “This will produce a steady level of nicotine to reduce your cravings.” b. “You should gradually reduce the dose after 3 months of use.” c. “You should use one spray in each nostril per dose up to five times per hour.” d. “You will not develop dependence on the nicotine in the nasal spray.” ANS: C Dosing for the nicotine nasal spray should be one spray in each nostril once or twice an hour, up to five times per hour. The spray causes a rapid rise in blood nicotine levels with each dose, which more closely simulates smoking. After 4 to 6 weeks, dosing should be gradually reduced and then stopped. Many people become dependent on the spray.DIF: Cognitive Level: ApplicationREF: pp. 320TOP: Nursing Process: Implementation MSC: NCLEX Client Needs Category: Physiologic Integrity: Pharmacologic and Parenteral Therapies 267. A patient who has just found out she is pregnant tells the nurse she wants to quit smoking. She asks about pharmacologic aids to help her quit. The nurse is correct to tell her what? a. “Nicotine replacement therapy is harmful, but it is safer than smoking, so it can be used.” b. “Psychosocial support is the only recommended treatment for smoking cessation during pregnancy.” c. “Varenicline [Chantix] is safe to use during pregnancy.” d. “You should try to taper off your smoking gradually, because none of the drugs aresafe.” ANS: A Because nicotine is a component of nicotine replacement therapy (NRT), NRT is not safe during pregnancy. However, it is safer than smoking, and if it can help the patient stop smoking, it is recommended with caution. Psychosocial support is more effective when combined with a first- line pharmacologic agent. Varenicline has been shown to have harmful effects on fetuses in animal testing, so it should be avoided during pregnancy. Tapering nicotine withdrawal only seems to prolong the withdrawal symptoms; abrupt cessation is better than a taper.DIF: Cognitive Level: ApplicationREF: pp. 319TOP: Nursing Process: Implementation MSC: NCLEX Client Needs Category: Physiologic Integrity: Reduction of Risk Potential Rosenthal: Lehne's Pharmacotherapeutics for Advanced Practice Providers, 2nd Ed. Chapter 35: Drug Abuse IV: Major Drugs of Abuse Other Than Alcohol and Nicotine Test Bank Multiple Choice 268. A patient arrives in the emergency department complaining of dizziness, lightheadedness, and a pulsating headache. Further assessment reveals a blood pressure of 82/60 mm Hg and palpitations. The patient’s friends tell the nurse that they were experimenting with “poppers.” The nurse will expect to administer which medication? a. Diazepam [Valium] b. Haloperidol [Haldol] c. Methylene blue and supplemental oxygen d. Naloxone [Narcan] ANS: C These findings are consistent with volatile nitrate overdose, as evidenced by the venous dilation. The primary toxicity is methemoglobinemia, which can be treated with methylene blue and supplemental oxygen. Diazepam would not be used for patients experiencing volatile nitrate overdose, but it may be used in patients who have overdosed on hallucinogens. Haloperidol would be used in patients who have overdosed on amphetamines. Naloxone would be used to treat an opioid overdose.DIF: Cognitive Level: ApplicationREF: pp. 323TOP: Nursing Process: Implementation MSC: NCLEX Client Needs Category: Physiologic Integrity: Reduction of Risk Potential 269. A patient who is an opioid addict has undergone detoxification with buprenorphine [Subutex] and has beengiven a prescription for buprenorphine with naloxone [Suboxone]. The patient asks the nurse why the drug was changed. Which response by the nurse is correct? a. “Suboxone has a lower risk of abuse.” b. “Suboxone has a longer half-life.” c. “Subutex causes more respiratory depression.” d. “Subutex has more buprenorphine.” ANS: A The combination of buprenorphine and naloxone [Suboxone] discourages intravenous abuse, because with IV use, the naloxone precipitates withdrawal; this effect does not occur with sublingual dosing [Subutex]. Suboxone does not differ from Subutex in terms of drug half-life. Subutex does not cause more respiratory depression and does not contain more buprenorphine.DIF: Cognitive Level: ApplicationREF: pp. 324TOP: Nursing Process: Implementation MSC: NCLEX Client Needs Category: Physiologic Integrity: Pharmacologic and Parenteral Therapies 270. What is the primary reason for opioid abuse? a. Ease of access b. Initial “rush” similar to orgasm c. Peer pressure d. Prolonged sense of euphoria ANS: D The primary reason for opioid abuse is the prolonged sense of euphoria that occurs after the initial rush. Healthcare professionals have easy access to opioids, which makes them more vulnerable to abuse of these drugs, but this is not the primary reason for abuse in the greater population. The initial rush lasts about 45 seconds and is not the primary reason for opioid abuse. Peer pressure is not the primary reason for opioid abuse.DIF: Cognitive Level: ComprehensionREF: pp. 321TOP: Nursing Process: Assessment MSC: NCLEX Client Needs Category: Physiologic Integrity: Pharmacologic and Parenteral Therapies 271. A provider orders clonidine [Catapres] for a patient withdrawing from opioids. When explaining the rationale for this drug choice, the nurse will tell this patient that clonidine is used to: a. prevent opioid craving. b. reduce somnolence and drowsiness. c. relieve symptoms of nausea, vomiting, and diarrhea. d. stimulate autonomic activity. ANS: C When administered to an individual physically dependent on opioids, clonidine can suppress some symptoms of abstinence. Clonidine is most effective against symptoms related to autonomic hyperactivity, including nausea, vomiting, and diarrhea. Clonidine does not stimulate autonomic activity; it is effective against symptoms of autonomic hyperactivity. Clonidine does not reduce somnolence and drowsiness. Clonidine does not prevent opioid craving.DIF: Cognitive Level: ApplicationREF: pp. 323TOP: Nursing Process: Implementation MSC: NCLEX Client Needs Category: Physiologic Integrity: Pharmacologic and Parenteral Therapies 272. A young adult patient is admitted to the hospital for evaluation of severe weight loss. The nurse admitting this patient notes that the patient has missing teeth and severe tooth decay. The patient’s blood pressure is 160/98 mm Hg. The patient has difficulty answering questions and has trouble remembering simple details. The nurse suspects abuse of which substance? a. Cocaine b. Ecstasy c. Marijuana d. Methamphetamine ANS: D Methamphetamine causes all of the symptoms shown by this patient. These are not symptoms associated with cocaine, Ecstasy, or marijuana.DIF: Cognitive Level: EvaluationREF: pp. 326TOP: Nursing Process: Assessment MSC: NCLEX Client Needs Category: Physiologic Integrity: Pharmacologic and Parenteral Therapies 273. A nurse is discussing the differences between OxyContin OC and OxyContin OP with a group of nursing students. Which statement by a student indicates understanding of the teaching? a. “OxyContin OC cannot be drawn into a syringe for injection.” b. “OxyContin OP has greater solubility in water and alcohol.” c. “OxyContin OP is not easily crushed into a powder.” d. “Patients using OxyContin OP are less likely to overdose.” ANS: C OxyContin OP is a newer formulation that is designed to reduce OxyContin abuse. The OP formulation is much harder to crush into a powder. The OC preparation can be crushed and dissolved in water or alcohol and can easily be drawn into a syringe. The OP preparation does not dissolve easily in these solutions. Despite the differences in preparation, there is no indication that either form is less subject to abuse or overdose.DIF: Cognitive Level: AnalysisREF: pp. 322TOP: Nursing Process: Planning MSC: NCLEX Client Needs Category: Physiologic Integrity: Reduction of Risk Potential 274. A college student tells the nurse that several friends have been using synthetic marijuana to get high. What will the nurse tell this patient about this type of substance? a. “These substances are fairly safe because they are derived from herbs.” b. “They can cause hypertension, nausea, vomiting, and hallucinations.” c. “These substances do not have mind-altering affects.” d. “These substances produce a high and they are not illegal.” ANS: B Synthetic marijuana can produce severe symptoms including hypertension, nausea, vomiting, and hallucinations. Although once thought safe, it is no longer considered safe. It produces a high and can cause hallucinations. Many types of synthetic marijuana are now illegal.DIF: Cognitive Level: ComprehensionREF: pp. 330TOP: Nursing Process: Planning MSC: NCLEX Client Needs Category: Physiologic Integrity: Pharmacologic and Parenteral Therapies 275. In discussing the rationale for using methadone to ease opioid withdrawal, the nurse would explain that it has which pharmacologic properties or characteristics? a. Methadone can prevent abstinence syndrome. b. Methadone has a shorter duration of action than other opioids. c. Methadone is a nonopioid agent. d. Methadone lacks cross-tolerance with other opioids. ANS: A Methadone is used to ease opioid withdrawal and can prevent abstinence syndrome. Methadone does not have a shorter duration of action. Methadone is not a nonopioid agent. Methadone does not lack cross-tolerance with other opioids.DIF: Cognitive Level: ComprehensionREF: pp. 322TOP: Nursing Process: Diagnosis MSC: NCLEX Client Needs Category: Physiologic Integrity: Pharmacologic and Parenteral Therapies 276. A patient who is agitated and profoundly anxious is brought to the emergency department. The patient acts paranoid and keeps describing things in the room that do not exist. A cardiac monitor shows an irregular ventricular tachycardia. Which medication will the nurse expect to administer? a. Anticocaine vaccine b. Diazepam [Valium] c. Disulfiram [Antabuse] d. Vigabatrin [Sabril] ANS: B This patient is showing signs of acute cocaine toxicity. Diazepam can be given to reduce anxiety and suppress seizures which may occur. Anticocaine vaccine, disulfiram, and vigabatrin are drugs under investigation for treating cocaine addiction.DIF: Cognitive Level: ApplicationREF: pp. 326TOP: Nursing Process: Planning MSC: NCLEX Client Needs Category: Physiologic Integrity: Pharmacologic and Parenteral Therapies 277. A nurse is teaching a drug prevention class to a group of parents of adolescents. Which statement by a parent indicates understanding of the teaching? a. “Compared with alcohol, marijuana has little or no long-term adverse effects.” b. “Ecstasy causes reversible damage to serotonergic neurons.” c. “LSD does not cause an abstinence syndrome when it is withdrawn.” d. “Most individuals who abuse opioids began using them therapeutically.” ANS: C Although tolerance to LSD develops rapidly, there is no abstinence syndrome with abrupt withdrawal of the drug, and tolerance fades rapidly. Many adverse behavioral, subjective, and long-term effects are associated with chronic use of marijuana. MDMA [Ecstasy] can cause irreversible damage to serotonergic neurons. Most people who go on to abuse opioids begin their drug use illicitly; only an exceedingly small percentage of those exposed to opioids therapeutically go on to abuse these drugs.DIF: Cognitive Level: AnalysisREF: pp. 331TOP: Nursing Process: Implementation MSC: NCLEX Client Needs Category: Physiologic Integrity: Pharmacologic and Parenteral Therapies 278. A nurse is caring for a patient who is addicted to barbiturates and who will begin receiving phenobarbital. The nurse discusses the care of this patient with a nursing student. Which statement by the student indicates understanding of the teaching? a. “Phenobarbital acts as an antagonist to barbiturates and prevents toxicity.” b. “Phenobarbital has a long half-life and can be tapered gradually to minimize abstinence symptoms.” c. “Phenobarbital can be administered on an as-needed basis to treat withdrawalsymptoms.” d. “Phenobarbital prevents respiratory depression associated with barbiturate withdrawal.” ANS: B Phenobarbital has a long half-life and can be given to ease barbiturate withdrawal and suppress symptoms of abstinence. Phenobarbital is not an antagonist to barbiturates. It is not used on a PRN basis. Phenobarbital does not prevent respiratory depression associated with acute toxicity.DIF: Cognitive Level: AnalysisREF: pp. 325TOP: Nursing Process: Planning MSC: NCLEX Client Needs Category: Physiologic Integrity: Pharmacologic and Parenteral Therapies 279. Which factors make meperidine an opioid of choice among nurses and physicians who abuse opioids? Select all that apply. a. Easy access to syringes for administration of the drug b. Highly effective oral dosing c. Increased effects on smooth muscle function d. Less pupillary constriction than other opioids e. Shorter half-life than other opioids ANS: B , D Meperidine is often abused by medical personnel because oral dosing is highly effective, so telltale injection marks are unnecessary. Also, the drug causes less pupillary constriction than other opioids. Access to syringes is not necessary with oral dosing. Meperidine has fewer effects on smooth muscle function, causing less constipation and urinary retention.DIF: Cognitive Level: ApplicationREF: pp. 321TOP: Nursing Process: Assessment MSC: NCLEX Client Needs Category: Physiologic Integrity: Pharmacologic and Parenteral Therapies 280. A college student is brought to the emergency department by a group of friends who report that they had been dancing at a nightclub when their friend collapsed. The patient has a temperature of 105° F and shows jaw clenching and confusion. The nurse will expect to administer which medication? a. Dantrolene [Dantrium] b. Haloperidol [Haldol] c. Methadone d. Naloxone [Narcan] ANS: A This patient shows signs of Ecstasy toxicity. Dantrolene can be given to relax skeletal muscle to reduce heat generation and prevent the risk of rhabdomyolysis. The other medications are not used to treat Ecstasy toxicity.DIF: Cognitive Level: ApplicationREF: pp. 332TOP: Nursing Process: Assessment MSC: NCLEX Client Needs Category: Physiologic Integrity: Pharmacologic and Parenteral Therapies 281. A patient arrives in the emergency department acutely intoxicated and difficult to arouse. The patient’s friends tell the nurse that the patient took a handful of diazepam [Valium] pills while at a party several hours ago. The nurse will expect to administer which drug? a. Buprenorphine [Subutex] b. Flumazenil [Romazicon] c. Nalmefene [Revex] d. Naloxone [Narcan] ANS: B Flumazenil can reverse signs and symptoms of benzodiazepine overdose. Buprenorphine, nalmefene, and naloxone are all used to treat opioid addiction or toxicity.DIF: Cognitive Level: ApplicationREF: pp. 325TOP: Nursing Process: Implementation MSC: NCLEX Client Needs Category: Physiologic Integrity: Reduction of Risk Potential 282. A patient who has a long-term addiction to opioids takes an overdose of barbiturates. The nurse preparing to care for this patient will anticipate: a. a severe abstinence syndrome when the effects of the barbiturates are reversed. b. minimal respiratory depression, because the patient has developed a tolerance toopioids. c. observing pinpoint pupils, respiratory depression, and possibly coma in this patient. d. using naloxone [Narcan] to reverse the effects of the barbiturates, because cross-tolerance is likely. ANS: C A patient who is tolerant to opioids does not have cross-tolerance to barbiturates, so this patient will show signs of overdose such as pinpoint pupils, respiratory depression, and coma. Because there is no cross-tolerance, a patient addicted to opioids will not have an abstinence syndrome when the effects of the barbiturates are reversed. Respiratory depression will be severe. Naloxone cannot be used to reverse the effects of the barbiturates.DIF: Cognitive Level: EvaluationREF: pp. 322TOP: Nursing Process: Evaluation MSC: NCLEX Client Needs Category: Physiologic Integrity: Pharmacologic and Parenteral Therapies 283. A college student admits frequent use of LSD to a nurse and reports plans to stop using it. What will the nurse tell this student? a. Flashback episodes and episodic visual disturbances are common. b. Tolerance to the effects of LSD will fade quickly once use of the drug has stopped. c. Withdrawal symptoms can be mitigated with haloperidol [Haldol]. d. Withdrawal from LSD is associated with a severe abstinence syndrome. ANS: B Tolerance to the effects of LSD develops rapidly but fades quickly when the drug is stopped. Flashback episodes mayoccur but are not common. Haloperidol may actually intensify symptoms associated with an acute panic reaction; it is not indicated for LSD withdrawal. Abstinence syndrome does not occur when LSD is stopped.DIF: Cognitive Level: ApplicationREF: pp. 331TOP: Nursing Process: Implementation MSC: NCLEX Client Needs Category: Physiologic Integrity: Reduction of Risk Potential 284. A school nurse is teaching a high school health class about the effects of marijuana use. Which statement by a student indicates a need for further teaching? a. “Chronic use of marijuana can result in irreversible brain changes.” b. “Higher doses of marijuana are likely to produce increased euphoria.” c. “Marijuana is unique in that it produces euphoria, sedation, and hallucinations.” d. “Marijuana has more prolonged effects when it is ingested than when it is smoked.” ANS: B With higher doses of marijuana, euphoria may be displaced by intense anxiety. Chronic use may cause irreversible brain changes. Euphoria, sedation, and hallucinations can all occur with marijuana use. Ingesting marijuana causes prolonged effects.DIF: Cognitive Level: ApplicationREF: pp. 328TOP: Nursing Process: Evaluation MSC: NCLEX Client Needs Category: Physiologic Integrity: Pharmacologic and Parenteral Therapies 285. A pregnant patient reports using marijuana during her pregnancy. She asks the nurse whether this will affect the fetus. What should the nurse tell her? a. Children born to patients who use marijuana will have smaller brains. b. Neonates born to patients who use marijuana will have withdrawal syndromes. c. Preschool-aged children born to patients who use marijuana are more likely to be hyperactive. d. School-aged children born to patients who use marijuana often have difficultywith memory. ANS: D School-aged children born to patients who use marijuana may show deficits in memory, attentiveness, and problem solving. Chronic marijuana use alters brain size in individuals who use marijuana but not in children born to parents who use marijuana. Newborns will not show withdrawal symptoms. Preschool-aged children have difficulty with memory and sustained attention.DIF: Cognitive Level: ApplicationREF: pp. 328TOP: Nursing Process: Diagnosis MSC: NCLEX Client Needs Category: Physiologic Integrity: Pharmacologic and Parenteral Therapies 286. A patient who is a heroin addict is admitted to a methadone substitution program. After administering the first dose of methadone, the nurse notes that the patient shows signs of euphoria and complains of nausea. What will the nurse do? a. Administer nalmefene [Revex]. b. Contact the provider to obtain an order for naloxone [Narcan]. c. Question the patient about heroin use that day. d. Suspect that the patient exaggerated the amount of heroin used. ANS: D Patients entering a methadone substitution program must be carefully questioned about the amount of heroin used; patients may exaggerate the amount used to obtain higher doses of methadone or may minimize the amount used to downplay the extent of their addiction. In patients who exaggerate use, the amount of methadone given may cause euphoria, nausea, and vomiting. Nalmefene and naloxone are used to treat overdose and are not indicated. A patient receiving methadone along with a usual heroin dose would be likely to have signs of toxicity.DIF: Cognitive Level: AnalysisREF: pp. 323TOP: Nursing Process: Implementation MSC: NCLEX Client Needs Category: Physiologic Integrity: Pharmacologic and Parenteral Therapies Rosenthal: Lehne's Pharmacotherapeutics for Advanced Practice Providers, 2nd Ed. Chapter 36: Review of Hemodynamics Test Bank Multiple Choice 287. A nurse is teaching a nursing student how blood can return to the heart when pressure in the venous capillary beds is very low. Which statement by the student indicates a need for further teaching? a. “Constriction of small muscles in the venous wall increases venous pressure.” b. “Negative pressure in the left atrium draws blood toward the heart.” c. “Skeletal muscles relax to allow the free flow of blood.” d. “Venous valves help prevent the backflow of blood.” ANS: C Skeletal muscle contraction, along with one-way venous valves, helps create an “auxiliary” venous pump that helps drive blood toward the heart. Constriction of small muscles in venous walls helps increase venous pressure. Negative pressure in the left atrium sucks blood towardthe heart. Valves, which are one-way, work with the contraction of skeletal muscles to create a venous pump.DIF: Cognitive Level: ApplicationREF: pp. 338TOP: Nursing Process: Assessment MSC: NCLEX Client Needs Category: Physiologic Integrity: Physiologic Adaptation 288. A nurse is caring for a patient who is receiving a drug that causes constriction of arterioles. The nurse expects to observe which effect from this drug? a. Decreased stroke volume b. Increased stroke volume c. Decreased myocardial contractility d. Increased myocardial contractility ANS: A Constriction of arterioles increases the load against which the heart must pump to eject blood. Increased constriction of arterioles would decrease, not increase, the stroke volume of the heart. Myocardial contractility is determined by the sympathetic nervous system, acting through beta1- adrenergic receptors in the myocardium.DIF: Cognitive Level: AnalysisREF: pp. 339TOP: Nursing Process: Assessment MSC: NCLEX Client Needs Category: Physiologic Integrity: Physiologic Adaptation 289. A patient with hypertension is admitted to the hospital. On admission the patient’s heart rate is 72 beats/minute, and the blood pressure is 140/95 mm Hg. After administering an antihypertensive medication, the nurse notes a heart rate of 85 beats/minute and a blood pressure of 130/80 mm Hg. What does the nurse expect to occur? a. A decrease in the heart rate back to baseline in 1 to 2 days b. An increase in the blood pressure within a few days c. An increase in potassium retention in 1 to 2 days d. A decrease in fluid retention within a week ANS: A When blood pressure drops, the baroreceptors in the aortic arch and carotid sinus sense this and relay information to the vasoconstrictor center of the medulla; this causes constriction of arterioles and veins and increased sympathetic impulses to the heart, resulting in an increased heart rate. After 1 to 2 days, this system resets to the new pressure, and the heart rate returns to normal. The blood pressure will not increase when this system resets. Increased potassium retention will not occur. Over time, the body will retain more fluid to increase the blood pressure.DIF: Cognitive Level: EvaluationREF: pp. 341TOP: Nursing Process: Evaluation MSC: NCLEX Client Needs Category: Physiologic Integrity: Pharmacologic and Parenteral Therapies 290. A nurse is assessing a patient who has heart failure. The patient complains of shortness of breath, and the nurse auscultates crackles in both lungs. The nurse understands that these symptoms are the result of: a. decreased force of ventricular contraction. b. increased force of ventricular contraction. c. decreased ventricular filling. d. increased ventricular filling. ANS: A In the failing heart Starling’s law breaks down, and the force of contraction no longer increases in proportion to the amount of ventricular filling. The result is the backup of blood into the lungs and the symptoms of shortness of breath and crackles caused by fluid. Increased ventricular contraction would not result in a backup of blood into the lungs. Changes in ventricular filling are not the direct cause of this symptom.DIF: Cognitive Level: ApplicationREF: pp. 340TOP: Nursing Process: Assessment MSC: NCLEX Client Needs Category: Physiologic Integrity: Physiologic Adaptation 291. A patient is taking a drug that interferes with venous constriction. The nurse will tell the patient to: a. ask for assistance when getting out of bed. b. expect bradycardia for a few days. c. notify the provider if headache occurs. d. report shortness of breath. ANS: A A drop in venous pressure reduces venous return to the heart, and as blood pools in the extremities, orthostatic hypotension can occur. Patients taking drugs that reduce venous constriction should be cautioned to ask for assistance when getting out of bed. Bradycardia, headache, and shortness of breath are not expected effects.DIF: Cognitive Level: ApplicationREF: pp. 341TOP: Nursing Process: Diagnosis MSC: NCLEX Client Needs Category: Physiologic Integrity: Reduction of Risk Potential 292. A patient is taking a beta1-adrenergic drug to improve the stroke volume of the heart. The nurse caring for this patient knows that this drug acts by increasing: a. cardiac afterload. b. cardiac preload. c. myocardial contractility. d. venous return. ANS: C Beta1-adrenergic agents help increase the heart’s stroke volume by increasing myocardial contractility. Cardiac afterload is determined primarily by the degree of peripheral resistance caused by constriction of arterioles; increasing afterload would decrease stroke volume. Beta1- adrenergic agents do not affect afterload. Cardiac preload is the amount of stretch applied to the cardiac muscle before contraction and is determined by the amount of venous return. Beta1- adrenergic agents do not affect cardiac preload. Venous return is determined by the systemic filling pressure and auxiliary muscle pumps and is not affected by beta1-adrenergic agents.DIF: Cognitive Level: AnalysisREF: pp. 339TOP: Nursing Process: Evaluation MSC: NCLEX Client Needs Category: Physiologic Integrity: Physiologic Adaptation 293. A patient with a history of hypertension is admitted for a procedure. If the patient’s arterial pressure decreases, which clinical manifestation would the nurse expect to see? a. Decreased heart rate b. Increased heart rate c. Decreased blood pressure d. Syncope ANS: B When arterial pressure decreases, the vasoconstrictor center causes constriction of nearly all arterioles, leading to an increase in peripheral resistance, constriction of veins, increasing venous return, and subsequent acceleration of the heart rate. A decrease in arterial pressure would not cause a decrease in the heart rate or blood pressure, nor would it cause syncope.DIF: Cognitive Level: ApplicationREF: pp. 341TOP: Nursing Process: Assessment MSC: NCLEX Client Needs Category: Physiologic Integrity: Physiologic Adaptation Rosenthal: Lehne's Pharmacotherapeutics for Advanced Practice Providers, 2 [Show More]

Last updated: 1 year ago

Preview 1 out of 949 pages

Add to cart

Instant download

document-preview

Buy this document to get the full access instantly

Instant Download Access after purchase

Add to cart

Instant download

Reviews( 0 )

$30.00

Add to cart

Instant download

Can't find what you want? Try our AI powered Search

OR

REQUEST DOCUMENT
122
0

Document information


Connected school, study & course


About the document


Uploaded On

Jan 24, 2021

Number of pages

949

Written in

Seller


seller-icon
securegrades

Member since 4 years

117 Documents Sold


Additional information

This document has been written for:

Uploaded

Jan 24, 2021

Downloads

 0

Views

 122

Document Keyword Tags

Recommended For You

Get more on TEST BANK »

$30.00
What is Browsegrades

In Browsegrades, a student can earn by offering help to other student. Students can help other students with materials by upploading their notes and earn money.

We are here to help

We're available through e-mail, Twitter, Facebook, and live chat.
 FAQ
 Questions? Leave a message!

Follow us on
 Twitter

Copyright © Browsegrades · High quality services·